You are on page 1of 350
Study Manual for SOA Exam MLC Life Contingencies Eighth Edition by Abraham Weishaus, Ph.D., F.S.A., CFA, M.A.A.A. Note: NO RETURN IF OPENED TO OUR READERS: ( Please check ASM’s web site at www.studymanuals.com for errata and updates. If you have any comments or reports of errata, please I e-mail us at mail@studymanuals.com, | ©Copyright 2009 by Actuarial Study Materials, 276 Roosevelt Way, Westbury, NY 11590. ‘All rights reserved. Reproduction in whole or in part without express written permission from the publisher is strletly prohibited. Contents 1 Probability Review 1 Probability Basics 1.1 Functions and moments . 1.2 Percentiles. . . 1.3 Conditional probability and expectation 1.4 Probability distributions Exercises een Solutions... - - 2 Variance 21 Additivity 2.2 Normal approximation 23 Mixtures ae 24 Bernoulli shortout . « 2.5 Conditional variance Exercises. Solutions sae ee I Life Contingencies 3. Survival Disttutions: Probability Fnetions Te Tales 3.1 Probability Functions . . 32 Life Tables - 4 Survival Distributions: Force of Mortality Exercises Solutions... 55+ 5. Survival Distributions: Mortality Laws 5.1 Mortality Laws for Exam Questions 5.11 Exponential Distibution, or Constant Force of Mortality 5.1.2 Uniform Distribution, or de Moivre's Law 5.1.3 Beta Distribution, or Generalized de Moivre's law 5.2 Morality Laws that May be Used for Human Moraty 52.1 Gompertz’sLaw ... «+ 7 5.2.2. Makeham’s Law 5.2.3 Weibull Distribution Exercises 7 Solutions 6 Survival Distributions: Moments SOA MLC Sy MaranlBiheion Copa 200 ASNT 35 35 37 38 2 48 50 58 o 67 67 68 68 6 69 9 9 n iv CONTENTS 6.1 Complete ee te eaeeeece tee : 8 6.2 Curate. eee ae B Exercises. : spree seit ae eee renee Se auat Solutions me : i Sees cee 7 Survival Distributions: Percentiles, Recursions, and Life Table Concepts 95 71 Percentiles eee eran 95 7.2 Recursive Formulas for Lite Expectancy... || 1. 9S > 7.3 Central Death Rate, bnT at) aSeieHor Exercises : : 100 Solutions : i ee 406 8 Survival Distributions: Fractional Ages 113 8.1 Uniform Distribution of Deaths... 6... . ee Secrets 8.2. Constant Force of Mortality 7 8.3 Hyperbolic Assumy ; 119 84 Summary eae : 122 Brereises ees 123 Solutions Sete 132 9 Select Mortality 143 ae cee : ee 146 i vee 153 10 Insurance: Payable at Moment of Death—Moments—Part 1 161 10.1 Definitions and General Formulas papery : 161 10.2. Constant Foree of Mortality aaa eee eel Bxercises .., . . : : aus fee 168 Solutions... Pieper ien re aan a ae 176 11 Insurance: Payable at Moment of Death—Moments—Part 2 183 I1.1 de Moivre’s ee ‘ ae 183 11.2 Other Mortality Functions Deere ie See 113 Integrating ce IT! 184 tat ferevae || . : eee aee 32 fear : 185 11.4 Increasing and Decreasing Insurances foes ‘ pees 185 11.5 Variance of Endowment Insurance... . . : : 186 11.6 Normal Approximation : a 187 Exercises... ee Seaeees sora etree oe a age ane Solutions. eee ee st eee +e 195 12 Insurance Payable at Moment of Death: Percentiles 205 Brerelies oe eee erie ere oO Solutions : nae 212 13 Insurance Payable at End of Year: Moments 224 Exercises aticeaa: been iene Solutions oe ee ‘ see 235 14 Insurance Payable at End of Year: Recursions, Varying 245 SOA MLC Stay Manual eon eps cnc09 ask 2NTENTS 14.1 Recursive Formulas 142. Increasing and Decreasing Insurance - Exercises Solutions 15 Insurance: Discrete to Continuous Bxercises oe ee Solutions +16 Annuiti 16.1 Whole Life Annuity . . 16.2 Temporary Life Annui 16.3 Deferred Life Annuity 16.4 Other Annuities...» « 164.1. m-year certain and life annuity. 164.2 Accumulated value 1643 Increasing and decreasing anus Exercises Solutions 17 Annuities: Diserete, Expectation V7.1 Annuities-due . . « 17.2. Annuities-immediate 173. Accumulated value Exercises Solutions. Solutions 19 Annuities: Percentiles, Recursive Calculations 19.1. Distribution function of annuity random variable 19.2. Percentiles thly Payments Bxercises Solutions ‘ontinuous, Expectation 21 Premiums: Fully Continuous Expectation 211 Loss at Issue Exercises Solutions 22 Premiums: Diserete Expectation 22,1. Three Premiuin Principle Exercises Solutions ‘SOK MLCSty Banosa0 eon Conytahi ©2009 ASM 245; 246 249 285 263, 264 267 mm Mm 274 275 276 216 2716 216 211 281 287 287 289 291 293, 306 3i7 321 328 339 339 341 343 344 351 361 362 365, am 373, 314 318. 383 386 387 402 vt 23 Premiums: Percentile of Loss at Issue 24 Premiums: Variance of Loss at Issue, Continous Bxerci 25 Premiums: Vi Exercises Solutions iance of Loss at Issue, Discrete 26 Premiums: ‘True Fractional Premiums Exercises... eee Solutions 2... 27 Reserves: Prospective Formula Exercises 7 Soluti 28 Reserves: Retrospective Formula 28,1 ‘Three Premium Principle Exercises Solutions... . 29 Reserves: Annuity-Ratio and Insurance-Ratio Formulas 29.1 Summary of Reserve Formulas. . Rrercises Solutions 30 Reserves: Variance of Loss Exercises Solutions 31 Reserves: Recursive Formulas Exercises aH Solutions... . ee 32 Reserves: Advanced Topics 32.1 General Insurances. 32.2 Refund of Reserve 32.3 Fractional Premiums 324 Fractional Durations 32.5, Accounting Loss Exercises Solutions 33 Multiple Lives: Joint Life Probabilities 33,1 Introduction . Hebe 33.2. ‘The Joint Life Distribution Function 33.3 Independence . . SOA MLC Suaty Monst—Bh edtion Copyright 2009 aSht CONTENTS 49 eee 420 421 425 21 431 439 see 440 bere 445 451 452 454 459 sees 462 sees 467 473 415 Seale a vii CONTENTS 7 _ Exercises ae : ‘i sn Solutions ae 576 34 Multiple Lives: Last Survivor Probabilities 581 Exercises epee : 586 Solutions 595 597 35 Multiple Lives: Moments 35.1 Expected Value. . fees 135.2 Variance and Covariance. . « Bxorcises Solutions 36 Multiple Lives: Insurances ot Exercises 614 Solutions. . ; org 37 Multiple Lives: Annuities 625 37.1 Introduction ee 625 37.2 Three Techniques for Handling Annui a ; 626 37.3 Other Techniques. : 29 374 Variance ae 630 Exercises... see ae tease 6B Solutions eee GAD 38 Multiple Lives: Contingent Su 649 38.1. Contingent Probabilities ae : ee 649 38.2 Contingent Insurances : ees 654 Exercises... « ie et eerie 656 Solutions i ee) 39 Multiple Lives: Common Shock ont Brercises .. . « ede fee See 615 Solutions. i on 40 Multiple Decrement Model 683 40.1 Introduction 683 402 Life Tables : cH IEE 684 40.3. Examples of Multiple Decrement Probabilities : ‘ CEs LORS: 40.4 Discrete Insurances ede 687 Exercises 689 Solutions 698 41 Multiple Decroment Models: Forces of Decrement 708 Bxercises c os 708 Solutions ns 42 Multiple Decrement Models: Associated Single Decrement Tables m3 Exercises a se 6 Solutions 732 SOA MLC Sey Manual eon eps @2009 aS vii CONTENTS 43. Multiple Decrement Models: Going Between Probabilities and Rates 43.1. Uniform Distribution Assumed in the Double Decrement Table... es... ss 43.2 Uniform Distribution Assumed in the Associated Single-Decrement Tables - |» 43,3 Discrete Decrements .... 4... en aa Bxercises : Solutions 44 Multiple Decrement Models: Continuous Insurances Bxewises . 2.2.2... : ‘| See eee Solutions ee eee eae ae 45 Expenses: Expense Premium and Reserve 45.1 Expense Premium... .. . eee : REE 45.2 Expense Reserve |. . eae R aaa ait 45.3 Variance of Loss 7 Beer Brereises oy See Solutions. . nae se oie 46 Expenses: Policy Fees Exercises Solutions 47 Expenses: Asset Shares Exercises Solutions IM Stochastic Processes 48 Multi-State Models (Markov Chains): Probabilities Bxercises Sate . Solutions 49 Multi-State Models (Markov Chains): Premiums and Reserves 49.1 Prototype Example... 4... . a eee 49.2 Insurances . . . 49.3 Annuities 49.4 Premiums and Reserves Brerises 0... . 7 Solutions cee 50 The Poisson Process: Probabilities of Events 50.1 Introduction... . : Case eeeeeeet teresa 50.2 Probabitities—Homogeneous Process... eee, 50.3 Probabilities—Non-Homogeneous Process. Exercises . . oe ‘| Solutions... : eben aie 51 The Poisson Process: ‘Time To Next Event ree re Solutions ‘Coppa 2000 ASME 739 739 ma m4 748 755 763 165 7m 785 785 786 787 799 197 805 807 810 815 818 823, 827 829 836 84) 84s 845 845, 855, 861 861 862. 865, 869 873 876 877 CONTENTS fea 52 The Poisson Process: Thinning 52.1 Constant Probabilities . 52.2 Non-Constant Probabilities Exercises tee : Solutions. . + 153 The Poisson Process: Sums and Mixtures 53.1 Sums of Poisson Processes. . « 53.2 Mixtures of Poisson Processes 54 Compound Poisson Processes 54.1 Definition and Moments. - a 542, Sums of Compound D Distributions Exercises Solutions IV Practice Exams 1 Practice Exam 1 2. Practice Kxam 2 3. Practice Exam 3 4 Practice Exam 4 5 Practice Exam & 6 Practice Exam 6 7 Practice Exam 7 8 Practice Exam 8 Appendices ‘A. Solutions to the Praetice Exams Solutions for Practice Exam 1... Solutions for Practice Exam 2 Solutions for Practice Exam 3 Solutions for Practice Exam 4 Solutions for Practice Bxam 5... Solutions for Practice Exam 6 nee Solutions for Practice Exam 7. Solutions for Practice Exam 8 B_ Solutions to Old Exams B.L_ Solutions to CAS Exam 3, Spring 2005 SOA MLC Sey Maou 80 epg 2008 ASM 879 879 881 882 886 891 891 892, 895 998 903 903 905 905 oui 917 919 927 937 943; 951 959 967 975 983 985 985 994 + 1008 1014 1024 1035 1097 1056 1069 = 1069 B2_ Solutions to SOA Exam M, Spring 2005 B.3. Solutions to CAS Exam 3, Fall 2005 . B.4. Solutions to SOA Exam M, Fall 2005 B.S_ Solutions to CAS Exam 3, Spring 2006 B.6 Solutions to CAS Bxam 3, Fall 2006 . , B.7 Solutions to SOA Exam M, Fall 2006 B.8 Solutions to CAS Exam 3, Spring 2007 B.9 Solutions o SOA Exam MLC, Spring 2067 B.10 Solutions to CAS Exam 3, Fall 2007 B.1] Solutions to CAS Exam 3L, Spring 2008 | | || 3.12 Solutions © CAS Bxam 31, Fall 2008, B.13 Solutions to CAS Bxam 31, Spring 2009 | C Exam Question Index ‘SOA MLC Sty Mana sh eon Cenygh 20 Ashe CONTENTS = 1073 tos2 + 1086 + 1094 1100 seen es HOS sees UB vee 1UB + 1126 ++ 1130 1133 1137 141 Preface Welcome to Exam MLC! Teas ML is where life actuaries lear the traditional craft. The issue dealt with by Life Contingencies, which ie 800% ot the course, is how to price and reserve for an insurance whose benefits wil not be paid for along time ‘This requires dealing with both probes of evens and interest—the topics of exams P and FM. cieftional anodes included inthis course are Markov chains and Poisson processes, Life insurance is simple in the sense thatthe insured event can only happen once. But other insurances aren't so simple, With disability insur ance, person ean become disabled, den recover, cc, Markov cheins are a simple model which allow movement between an arbitrary number of states. Voisson processes aren't really closely zlated to insurance, bat are @ general-purpose model wih the exam committee thought you should know. Trudiional insurance and annuity reserve calculations still use traditional Life Contingencies methods, but mod xn nuaet hnked products are more based on asset behavior. For this reason, you must also study Financial Hoo™ cree hat material has been spit off into an unrelated exam, MFE, There i absolutely no relationship between MLC and MFE, and you may take them in either order, This manual ‘The only way you're going to earn this subject is by working out lots of problems. And problems are the most important thing this manual provides. You ae given easy to read explanations of every topic, with Jos ofsumnmaty errand illustrative Aigures, However the idea is not to memorize formulss, but to understand hem well enough otha you can reproduce them, And after working ou Tos of problems, you will be abe to do that. For addtional practic, I recommend working out the examples inthe manual by yourself before reading the answers Order of toples ‘The manual starts with probability review which you may scan quickly or ead slowly, depending on your back- ground, ‘This is purely background material, nothing you will be directly tested on, but if you don’t know what aetatliy density functions, moments oF percentiles are, youl hae serous problems with this cours. ae re fe Contingencies, the heart and overwhelming majority ofthe cours. ‘The material is covered in alogial onder, avd there are very fw paces where you ean stody out of order, aa veers Stochastic Provestes, which is largely independent of Life Contingencies, Moreover, Markov chains at independent of Poisson processes, So you can study those in either order. Old exam questions “There are many original problems in the manual, but most of the problems are from old exams, They come fom cold Part 4, Part 4A, Course 150, and Course 151 exams, as well 25 2000-syllabus Exam 3. SOA Patt 4 in 1986 had morning and afternoon sessions. 1 indicate afternoon session questions with "A. The monning session had the more baste topics (Uhroogh reserves), while the aftemoon session had edvanced topes {anultiple lives, multiple decrements, etc.) Both sessions were multiple choice questions ‘SOA Course 150 from 1987 through 1991 hed multiple choice questions in the morning and essay ques the aftemoon, In most eases, the essay questions are not suitable for modern exams. I've Kept only & few, and even these few riay not be that appropriate. So ifnecessary, you may skip the “A" questions from those Years, “The CAS Part 4A exams awarded varying numbers of points to questions; some are 1 point and some are 2 points, The point questions are probably too easy for a modem exam, but they'l giv you a ite prac, THe pre 1987 exams probably were stil based on Jordan (ie ol textbook), but the questions I provided, while ancien SOA MLE:Sidy Manual eon xt oprah ©1009 AN xi PREFACE sill have value, Similarly, the cluster questions on SOA Course 150 in the 1990's generally were awarded 1 point per question, ‘The fitst edition of Actuarial Mathematics, which was used until around 1997, had commutation functions, In some cases, I've adapted such questions for use without commutation functions,” You will see some “based on” uestions where I made this adaptation. Even though these questions still have a commutation function feel, they ate still legitimate questions. You should work out both SOA and CAS questions, regardless of whether you are an SOA or a CAS student Mite styles may difr between organizations and between pre-2000 and post-2000 question, these questions are based on the same textbook and syllabus. The fact s, some of the pre-2000 questions have shown up on pos 2000 ‘exams, The CAS expects less with regard to Life Contingencies than the SOA does, s0 CAS Life Contingencies ‘Questions ore easier than what an SOA student should expect onthe exam, Stl, they are a good stating point. Course 151 Is the leas relevant to this subject, 've only included a small number of questions from 151 in the background lesson on variance, Back in 1999, the CAS and SOA created a sample exam for the then-new 2000 syllabus, This exam had some uestions from previous exams but also some new questions, some of them not multiple choice. This sample exam Wwas never a real exam, and some of its questions were defective. This sample exam is no longer available on the ‘web, Thave included appropriate questions from it. Whenever a problem is labeled C3 Sample, it refers to the 1999 sample, not the current list of sample questions. Questions from exams Spring 2005 and later are not included in this manual. You may use those asa final test St your knowledge, Check Appendix C fora listing of which questions are not onthe syllabus. For the period since Spring 2005, the SOA released Spring 2005, Fall 2005, Fall 2006, end Spring 2007. The CAS releases all exams, put dors not provide worked-out solutions. However, Ihave provided worked-out solutions to all relevant questions fromm the released SOA and CAS exams in Appendix B Questions from old exams are marked xxx:yy, where xxx i the time the exam was given, with $ for spring and F for fal followed by a 2-digit year, and yy is the question number, Sometimes xxx is preceded with SOA or CAS Heats the sponsoring organization. From about 1986 to 2000, SOA exams had 3-diglt numbers (like 150) and GAS exams were a number and a letter (like 4A). From 2000 to Spring 2003, the exams were joimly sponsored, [There was « period in the 1990's when the SOA, while it allowed use ofits old exam questions, did not want people {o reveal which exam they came from. As a result, I sometimes had study notes for old exatns in this period and could not identify the exam they.came from. In such a case, I mark the question aaa-bb-ce:yy, where asa-bh-ce is {he study note number end yy is the question number. Generally aaa is the exam number (like 150), and ce is the 2-digit year the study note was published Tables Download the tables you will be given on the exam. They will often be needed for the problems. Go to vont, s0a. "9, click on EDUCATION on the top bar, then choose Exams and Requirements, then click on ASA, then on one Pf the Exam M buttons (hey both go to the same place), then on the first item in the frst eolimn, “Syllabus with Learning Objectives/Outcomes and Readings", then go to page 3 of the PDF and click on Exam MLC Tablos. ‘The direct address at this writing is http://mwa.soa.org/files/pdf/edu-2008-spring-nlc- tables. pdf ‘The tables include the Ilustrative Life Table, the Illustrative Service Table, some interest functions, andthe standard ‘normal distribution function, ‘The Tilustative Life Table included in the packet is somewhat different from the one in Actuarial Mathematics. ‘The values are rounded and the table has @ couple more columns, such as sE,, which are useful for calculations, In {iis manual, al exercises are worked out with the tables you get athe examn, not the tables in the textbook. The SOA hes specified rules for using the normal distribution table they supply: Do not intenpolae nthe table Simply use the nearest value, If you are looking for (0.0244), use (0.02). If you are given the cumulative Probability (x) = 0.8859 and need x, use 1.21, the nearest x available, SOA BILE Say Manes! ein Copia 2009 Aste PREFACE xi “The examples and exercises in this manual use this rounding method. On real exams, they will try to avoid ambiguous situations, so borderline situations won't occur, but my interpretation of the rules (used for problems in this manual) is that if the third place is 5, round up the absolute value. So I round 0.125 to 0.13 and ~0,125 to ~0.13, Note certain percentiles, such as the 95th percentile (1,645) are given in the tables, and should not be rounded to ‘ovo places, Practice Exams Eight practice exams are provided, Exem MLC will be 30 questions; accordingly, these exams are 30-question exams and should be completed in 3 hours if you want to go at exam speed, Other Useful Features of This Manual ‘The SOA site has a set of 282 sample questions and solutions (MILC-09-08). ‘These are mostly questions from released exams which are readily available, with a couple of additional questions from Markov Chains, expenses, Balducei, and contingent survival; nevertheless many students prefer to use this list since non-syllabus material hhas been removed (or at least that's what the SOA claims). Appendix C has a complete cross reference between these questions and the exams they come from, as well as the page in this manual having either the question or the solution, ‘This manual comes with an index. Whenever you remember some topic in this snanual but can't remember where you saw it, check the index. If it isn’t in the index but you're sure it’s in the manual and an index listing ‘would be appropriate, contact the author, New in This Edition Annuity distribution graphs were added to lesson 19. ‘Moltiple-life and raultiple-decrement lessons (33-44) were revritten for greater clarity. There is more explana- , more step-by-step instructions, more examples, and complete formula summaries for almost each ‘Solutions to relevant Spring 2009 CAS 3L exam questions were added About Exam MLC Breakdown of Questions {A breakdown of the number of questions on relevant exam topics appearing on recent SOA exams is in Table 1 For Markov chains, I only counted questions which can be answered in the current syllabus; there was more on the syllabus before 2005. Jin reading this table, Keep in mind the following changes to the syllabus: ‘© The exam was jointly sponsored by the SOA. and CAS from Spring 2000 through Spring 2003. Probably the SOA had most ofthe say on what appeared on the exam, but the CAS had some control. ‘+ From Spring 2000 through Fall 2004, the exam included frequency and severity models, ruin theory, and some simulati ‘* From Spring 2005 through Fall 2006, the exam included frequency and severity models, and some simulation, but not ruin theory. ‘© Spring 2007 represents the current syllabus. SOX MLC Say Manat eon epyigarqr008 ASHE PREFACE xiv Table 1: Number of questions in released exams ‘ointly sponsored Topic soo | Fo0 | sot | FoI | Fo2 || Fo3 | Fos | sos | Fos | Fos | so7 Survival aul ees uleeaul peat tae | aeaur tae stia| tia ty a iia: Insurances 4{/ a} a2} 3a] afoa] 3] 4] 2] 3] 3 Ann 3] 4} 2/1 3] 2f 3} 3] a] 3] 3] 2 Premiums paler oul ee Sul ee@E eae M aust iat lace at Reserves milee Sul eemat eae (ta: gaan retod tad c| tela lata a9: Maltilife 2] 2} 3] 2] 3} 2} 4] a] 4] 2] 3 Malti-decrement 1] a] 2} 2fafe2} a] a} af ata Expenses Se ieee tele ia Markov chains 1] of of rf rf of a} 2} af 2} 3 Poisson process 2| 4] a] 4] 3q4 a} 5] 3}4 Total life contingencies | 21 | 21 | 20 [ 20 | 21 ff a | at | a6 [a3 | 24 | 23 Total stochastic processes | 3] 4] 3] S| 41 4] 4] s| 8] 5s] 7 Total 24 | 25 | 23 | 25 | 26 || 25 “29 [30 ‘The SOA will not release any more exams, so Spring 2007 is the only released exam in the current syllabus. However, the table indicates that about six questions will be from stochastic processes, and the life contingencies questions will be evenly distributed among the topics. Changes to Syllabus Since Last Sitting There have been no changes to the syllabus since last siting. Other characteristics ‘There is no penalty for guessing, Fill in all questions regardless of whether you have time to work out the problem or not—you lose nothing and you may be lucky! ‘The answer choices on SOA exams are usually specific answers, not ranges. ‘The SOA maintains a set of formal rules. Allegedly, the correct answer choice is never be the same for 4 questions in a row. (At least on the published exams it has never happened.) 3-way true-false questions are out; however, there may be 4-way true-false questions witha fifth choice “AI statements are true (False)". ‘The SOA did not release all exams, ‘They will no longer release exams in the Future, as they accumulate questions. for a question bank for computer based testing. Acknowledgements 1 would like to thank the SOA and CAS for allowing me to use their old exam questions. These exam questions. constitute the bulk of this manual. I'd also like to thank Hatold Cherry for suggesting this manual and for providing three of the pre-2000 SOA exams and all of the pre-2000 CAS exams I used. Kevin Blackman deserves special thanks for checking the solutions to most of the problems in the first edi of the manual. ‘Many readers have provided errata lists; their contributions are appreciated, whether they consisted of multiple ‘e-mails with large errata lists or simply pointing out a single typo. Particularly notable contributions were provided by S08 MLC Stdy Moneta eon Copyabt @2009 ASH ACE see xy Don Neville, who acted virtually as a copy editor; about 90% of the errata for the first edition came from him, YYitay Lowy, who similarly provided the vast majority ofthe Life Contingency errors ofthe third edition, ‘as well as useful shortouts T was not aware of, and which are now incorporated. Rick Sutherland, who provided about 20 errata lists. [A pata list of other readers who submited errata is Carter Angell, Rich Astuillo, Yim Balogh, Michac! Betts ein Brown-Jowett, Ken Burton, Anna Buzueva, Michael Castellano, Christina Chambers, Kenneth Charette, Michael Chion, Jessica Culhane, Lindsey Daniels, Robert Dohner, Bryan Donkersgoed, Tim Dudziaky Matthew ce Yushong Feng, David Fernandez, Mathew Flanagan, Brad Foxa, Mordechai Golaburd, Mamuk Gomartel William Gornal, Lake Grady, Sana'a Hayat, Steve Hoagy, Scat Hickner, Martin ochler, Thomas Hopkins, Kathy Howard, Matthew Teer, Negi Jffery, Matthew Jewezyn, Derrick Kaufman, Geo Kini, Frank Kirby, Boris Krant Towvein Krasner, Anna Keylova, Takehiro Kumazawa, Xie Kun, Viedmir Kustov, Charles Lee, Bva Lee, Tommy Teng, Steven Ling, Nicole Li, Pljuan Liv, Joanne Loh, Andrew Magee, Rusty Maw, Kyle Milles, Anions Nic jason Owen, Jobn Peres, Amanda Popham, Rebecca Pulsifer, Ronald Rephael, Toff Raven, Amado Criet Fiver, Vikas Sata Megan Seot, Sarah Shitk, Cari Simon, Jason Thomas Smith, Stephen Smith, Brica Stead, John Smeirect, Yang Su, Susan Szpakowski, Stephen Taubel, Shelly Tilman, Harrison Weaver, Mendy Wenger, Jon ‘Winn, Wilson Wong, Janghyeon Woo, Xue Ying, Sandy Zhang. Thcetectors of TX, RS, and its malttude of packages all deserve thanks for making possible the professional typesetting ofthis mathematical materia Errata Please report all errors you find in these notes 10 the author You may send them to the publisher at mal1@ ctadynanuals.com or direcly to me at errata@aceyourexans.net. Please identify the manual end etion the ervoris in, ‘This is the eighth edition of the SOA Exam MLC manual. ‘An errata list will be posted at errata .aceyourexams.net, Check this errata list frequent ‘SOA MUCSmly Mana}—Bh een Copyieh GIO ASM. xvi SOA MUC Sindy Manesl_at editon Copy e009 ASK PREFACE Part I Probability Review PARTI, PROBABILITY REVIEW sp) | {SOA MLCStaty Mnun—Sin eon Copig @n000 ASH Lesson 1 Probability Basics Life Contingencies and Stochastic Provesses both use probability models, soars a brief summary of probability concepts you will need inthe cours, You wil not be directly tested thes topes, but they are essential background. IF you find this review too brief, you should review your favorite probability textbook for more details. 1.1. Functions and moments ste cumulative csribution faction of «random variable X, usually just called the distribuvon Snton, the vobabity Px) = Px(X 3 3. TtdefinesX, andi ight-coninuous, meaning Hn 108 +1) = F(Q) for h positive. one anon variables are disret (ihe points a which they are non-zero are isolated) and some e098 uous caning F(a) continuous, and differentiable except aa countable numberof point). Some are mixed—they are ‘continuous except at « countable number of points. ‘Some probability funetions are: «(9s the survival function, the complement of P(), the’probabilty of surviving longer than % Prk > 2). Jn Aetwarial Mathematics, a smalls (s(%)) is used for the same concept, «Fora continuous random vasiable, 2) i the probability density fonction, (0) = SC) the probability mass funtion, pla) = PrX = 2). Often, f(s) satis les as p(3) does for discrete variables. 4 Fora discrete random variable, p(x) the same relations for continuous var «na the hard ate fonction, Hn) = 423 = ~£82, Ta Itermationl Atal Notation, 1 He or He Me condional density fare, the conditional density given stival time ‘Why do we bother differentiating F to obtain 2 Because the density is noeded fer calculating moments. Mo- mene andom variable measure it center and dispersion. The expected value of X is defined by EIX= | xfGoxr and more genrily the expected vale ofa function of random variable is defined by nigco]= [ acosore For diserete variables, the intgrals are replaced with sums. ‘Thcn rar moment of Xis defined asp, = (XM). = fs the mean. The a central moment of X(n # Dis defined as jin = E{(X ~ 1)").' Usually 1 is a positive ‘integer, but it need not be, Expectation is linear, so the cena moments cen be calculated from the raw moments Py binomial expansion. In the binomial expansion, the last 2 terms always merge, so we have woe instead ofp — 2H an Wy ~ Bagge a? nstead of wy ~ 3ygue + ee? 1? a2) ig= ayy Gig? 3H stead of — Aun + Gge — Ani "sas wo sonneaton tos he Force of moral wsed in Aetarat Mathematics. SOA MLC Sty Maral eion 3 Copy 108 AS ea cae 1. PROBABILITY BASICS Special functions of moments are: * Tho variance is Var(X) = jz, and is denoted by 2, *+ The standend deviation othe postive square root ofthe variance. + The stewness is = 8, * The kurtosis is yo = Ms. * The coeficient of variation is £, Var(X) = £02) - B00? ‘Many times this is the best way to calculate variance, For 2 random variables X and ‘* The covariance is defined by Cov(X, Y) = EL ~ px)(¥ ~ py), + The conelationeveficen i died by pyy = SMD, As with the va ance, another formula for eavatiance is Cov(X, ¥) = EEKY] - ELXYELY) For independent random variables, Cov(X, ¥) = 0. A 100p% percentile isa number ny such that Pomp) = p and Fonz) 2, you are given (i) The hazard rate function: h(x) = £., for x > 2 Gi)__A value of the distribution function: 1°(5) = 0.84 Caleutate 2, Aya @)3 oa ) 5 SOA MLC Study Manas eon Copy 2000 ASK 1, PROBABILITY BASICS ® te 6 ‘Bxarcites continue on the nest page EXERCISES FOR LESSON ul 410, {CAS3-S04:28] A pizza delivery company has purchased an automobile liability policy for its delivery drivers from the same insurance company for the past five years. ‘The number of claims filed by the pizza delivery ‘company as the result of at-faull accidents caused by its drivers is shown below: Year | Claims 2o0r| 4 CT a 2000 [3 1999 | 1998 | 15) Calculate the skewness of the empirical distribution of the number of claims per year. (A) Less than 0.50 (B) Atleast 0,50, but ess than 0,75 (©) Atleast 0.75, but less than 1.00 (D) Atleast 1,00, but less than 1.25 (B) Atleast 1.25 L.11, You are given for the random variable X: @ Bx Calculate the skewness of X. (A) Less than =1 B) Atleast -1, bat ess than -0.5 (©) Atleast ~0.5, butless than 0 () Atleast O, but less than 05 ©) Atleast 0S 1.42, You are given the following: ‘+ Xisarandom variable with probability density function fe) (ey x2f,a>0,p>0 ‘+ mis the median of X. Determine the value of fn). (A) Less than 0.00020 (B) Atleast 0.00020, but less than 0.00025 (©) AtJeast 0.00025, but less than 0.00030 {D) Atleast 0.00030, but less than 0.00035, (B) Atleast 0.00035 ‘SOA MLC Sly Manuel eon Breciescontinwe on the nex page. ony 2008 ASM 2 1. PROBABILITY BASICS 1.13. [4-1°00:32} You are given the following for a sample of five observations from a bivariate distribution: 0 auane sauna i) 8, ‘Ais the covariance of the empirical distribution #. as defined by these five observations Bis the maximum possible covariance of an empirical dist Determine B~ A, 36,5 tion with identical marginal distributions to Fr, (a) 09 @®) 10 Out @) 1.2 ® 13 4, (CAS3-F04:24) A pharmaceutical company must decide how many experiments lo run in order to maximize profits, The company will receive a grant of $1 million if one or more of ‘© Bach experiment costs $2,900, ‘+ Bach experiment has a 2% probability of success, independent of the other experiment, experiments is successful, * All experiments run simultaneously, © Fixed expenses are $500,000, © Ignore investment income, ‘The company performs the number of experiments that maximizes its expected profit, Determine the company's expected profit before it starts the experiments. (a) 7.818 (B) 77,829 © 77,840 D) 77.851 (B) 77,862 1.18. Claim sizes for Kevin follow an exponential distribution with mean 6, Claim sizes for Kira follow an ‘exponential distribution with mean 12, Kevin and Kira submit one claim apiece, Calculate the probability that the sum of the two claims is greater than 20. Additional released exam questions: SOA M-F0S:17, CAS3-S06:25,30 Solutions Ld. SOA MLC Study Manesl-2thedlen (Copyieh @2009 ASM EXERCISE SOLUTIONS FOR LESSON 1 HE B 1.2. Fora gamma distribution, we have B{X!] = #(a + k= 1)---a. So = 200 4, = 60000 ‘60000 - 200* = 20000 1224-108 p= 120: 10° jua= 10[120 420) + 6642) ~308)] = 24-10" 24-108 ne For 1.3, ‘he abn is symetio, cite skewness [0] BI his not obvious o you, alu the mem, stich 30. Then ate tat 1s = 0.05(100 ~ 300) + 0.20200 ~ 300) + 0.20(400 ~ 300) + 0.05(500 ~ 300)" = 0 s0 the coeficient of skewness, which is py divided by 0°, is O- ‘id. Skevness is dimensionless; doubling a random variable has no effect on skewness, since it multiplies the numerator ad denominator by 2° = 8 So 1 is false. ‘Negatng a mndom variable nepitesthe numerator, without afecting the denominator since cis always posi tive, so 2is true, Gaz ul expest 3 10 be true, since as more identical random variables got added together, the distribution ecomes more and more normal (hich has skewness 0), To demonstrate 3, without Joss of generality ssue E(X) = 0, since we can use X! = X ~ px which has the same skewness as X; subtracting & constant has no effect on skewness, Var(k + ¥) = Var(X) + Var(¥) = 202, 80.0}, = 29/0. In the numerator, E(k 4 YP = EPS) + SELEY) + SEXTET?) + BD") 1x9) ae 2E skaen= See making 3 rue, ) 1.5. From the cosficient of variation, we have Zan x o=4 o =16 EQ?) = 16 + 0? = 64 136 — 3(20)(2) + 28) oe 64 SOR MLC Study Manas eon Coppa n0m a3 _ 1, PROBABILITY BASICS 1.6. ‘The vaianceis eae (a += 3% 44-37 5-37 _ : 2 ‘The fut cena moments mye CVAD BGI og Kurtosis is (c) 1.7, We differentiate, We don’t need the denominator, since all we want to dois set it equal to zero, mamerator (2) = 432°) ~ XOX + 224) =U +2 F((1 + 2 -6)) 5P=0 x= V0l = [0872] «© 1.8. Using the means and coefficients of variations, we have = Var(B) = 25 B(A?) = E(B?) = 3? 4-25 = 50 oe= 10 Var(C)= 100 So EA + BYA + C)] = E[A?] + E[AC] + ELBA) + E[BC] 50 + E[A] E[C] + E[5] [A] + E[B)E[C] = 50+ 25 + 1004 100 = 275 Cov((A + B),(A + C)) = 275 RIA + BRIA + C1 oan = VEC = V5 ome = VIETTO = VIS 25 25 215 (5 + 5S +20) = 25 ° 19, The survival function i (5) = exp(~ facade, s0 7 »-0sese(- "2a 2 2 0.16 = exp(-ZtIn5 ~ 1n2] = © nays} 2 2 SOA MLC: Sety Morel 8 on Coppa once She EXERCISE SOLUTIONS FOR LESSON 1 40 we calculate 1.10, For the empirical disuibuton, each observation is treated as having probability 5 = 358 eanpivcal fest, second, nd third raw moments: Aa le3424 15 5 164149444225 # he ’ 51 3375 = SALAM HES I. gs ‘Then the skewness is, 695 ~ 3(51)(5) + 265°) n a 7} ©) Lai. = 3(100 + 39@)428%) _ 897 Too" @) 42. So we have Dividing the second expression by the square of the fist =F De oF = dale =2) SOA MLC Sty Manat eon {pian no ASH 16 — _1. PROBABILITY BASICS And werejeet=1 since « > 0, Then B= 3(7500) = 5000, The median is determined from F(n) = 03,0 000, m= DO ~ 6299.61 Ws en) = (500 \ (o:0002381] By ! distribution is the one that assigns a probability of ! to each ofn observations, The covariance roducts of the observations minus the products of the means times the proba Ma aie Lo maxinize the covaciance, the ys shouldbe in he same onder as here's Toe products is (IG) + (2)(2) + (4)(3) + (5)(6) + (6)(4) = 74. If the y's were ordered in Increasing order, the sum would be (IQ) + (203) + (4)C4) + (5)64) + {6)(6) = 80. We must subtract 39 from cach and divide by 5, but since we're just Comparing A and B, we won't bother subtracting 59. Then 0.2(80 74) = [£3] (D) O.oaey ne probability of success fr n experiments is ~ 0.98%, s profit, ignoring fixed expenses, is 1,000,000(1 ~ 0.98") — 2900n, Differentiating this and setting it equal to 0: 5000} (6299.61, ies, which are ~10°(0.98")(In0.98) ~ 2900 = 0 0.98" a 1050.98 = 96.0815, 1n0.98 ‘are needed. Plugging those numbers into the otiginal expression ‘Thus cither 96 or 97 experiments 8(8) = 1,000,000(1 - 0,98") - 2900n gels 2(96) = 577,818.4 and g(97) Erste X_ = 10° - 0.98%) — 2900n — $00,000, the ineremental profit generated by experiment i is 577,794.0, s0 96 is best, and the expected profit is $77,818.4 ~ 500,000 = Xa Xie 108(098" 0.98) ~ 2500, ‘We wan! this difference to be greater than O, which occurs when 10°(098" 098") » 2500 0.98""'(0.02) > 0.0029 96» 20028 4 (~1)1n0.98 > ino.145 — pg NOtds _ 1.93102 *in0.98 ~ 0.02020 * te inequality got reversed because we divided by In0.98, a negative number. We conclude thatthe ‘th experiment increases profit only when n < 96.582, or n < 96, the same conclusion as ature SOA MLC Stdy Manas eon Cope o2009 ASK EXERCISE SOLUTIONS FOR LESSON 1 7 115, By the Law of Total Probability, the probability that the sum of two claims is greater than 20 is the integral ‘over all xof the probability tht the sum is greater than 20 given that Kira's claim is x times the density of Kira’s ition, (Ehe problem ean also be done integrating over Kevin's distribution.) If X is Kira's claim and Y is \'s claim, then noctroane [necer> oLoote £ [irre mod LiCX > 20 since X can't be negative, IfX < 20, then ¥ > 20 — X, and the probability of 1 distribution with mean 6 is, Now, Pro + ¥ > 20) that under an exponent So we split the integral up into x = 20 and O < x < 20. 1 1 Px a= [dear ervanyeh pnat EHV > D0) | gears yeas ‘The first integral, the probability of an exponential random variable with mean 12 being greater than 20, is 1 ~ Fx(20) = e?9!"?, The second integral is Maya ie 0-01 a= |e ‘dx Layette? 12 - ona 1 Lf” gama, 12 Jo =e So the final answer is 188876 + 0.188876 ~ 0.035764 = [04207 Pu(X + ¥ > 20) SON NLC Sty Merast—B0 een enya 008 ASH 18 ‘SOA MLC Stay Mania ih eion Copygh exon ase 1, PROBABILITY BASICS Lesson 2 Variance ‘You leaned everything in this lesson in your probability couse, Nevertheless, many stadents miss a It of these points. They are very important. "There won't be any examn questions testing you directly on this material, Rat ‘pround needed for the rest of the course, 3, this material is basic back- 2.1 Additivity pected vale ner meaning that #(0X + DY) = aX) +BECD, regs of wheter X and Yar independent area eI PTs EU] + 20IXY] + EUY?), for example, This means that + ¥9?] is nor equal to E(X?) + E[Y?] (unless E[XY] = 0). ‘As, it is nor in gneral that E030] = (CX), So B13") # (808) iene defined interns of expected value, Var X) = E(X}—E(XT this allows uso develop a formula for Vin(aX + bY), Ifyou workout, you get Var(aX 4 bY) = a? Var(X) + 2ab Cov(X, ¥) +B Var(Y) 1) In particular, if Cov(X, ¥) =0 (which i true ifX and ¥ ate independent, then Var(x 4 ¥) = Var(8) + Varl¥) ‘and generalizing to n independent variables, ve( Sox] Srwen fai ‘fall the X's ate independent and have identical distributions, and we sot X = forall j then vu | = nVar(X) fa However, Var(#X) = n? Var(X), not n Var(X). You must distinguish between these two situations, which are quite Wieeat ‘his ofthe following example. The stock market goes up or down randomly each day. We will assume deceth day's ehange is independent ofthe previous day's, and has the same distribution. Compare the variance of te following possibilities: 1. You pit Sin the market, and leave it there for 10 days 2, You put $10 in the marke, and leave it here fr 1 day. In dhe ft ose, there are going (o be potential ups and downs exch day, and the variance of the change of your rant ill be 10 times the variance of one day's change because ofthis averaging, n the second case, however, sore ary eulpyingthe volt of a singe day's change by 10—thre's no dampening ofthe change by 10 diferen, J Mlepondent random evens, the change depends ona single random even, ere, you are multiplying the wlatity fa single day's change by 100. Parana ener cane wher the variables ae not independent, you need to know the covatianee, This am De provided inacoverance mtr. Ifyou have tandom vtablesX,-» Xa thismn matt: A has ay = CovCKi 3) eect Poel Ja = VarX). This mati is symmettc end non-negative definite, However, covariance may be negative SOA MLC Sty Maenl—sin eon » Copyreh 2009 ASM 2. VARIANCE Exaupit 2A For a loss X on an insurance policy, et X; be the loss amount and X» the loss adjustment ex that X = Xy + Xo, The covariance matrix for these random variables is cc 3 ; 3 2) Calculate the variance in total cost ofa loss including loss adjustment expenses ‘Avswwen: In formula (2.1), = b= 1, 30.25 + 2(5) +2 = [34] a A sample is a set of observations from n identically distibuted random variables. The sample mean Z is the sum of the observations divided by . The variance of the sample mean of X,...,Xy Which are observations from the random variable X, is ‘Var(X) (2 Xi) _ marx) _ VantX) n a n hase 2.2. Normal approximation ‘The Central Limit Theorem says that for any distribution with ite variance, the sample rhean of a set of indepen lent identically distributed random variables approaches a normal distribution. By the previous section, the mews of the sample mean of observations of X is B[X) and the variance is o2/n, These parameters uniquely determines the orm distribution that the sample mean converges to. A random variable ¥ with normal distribution with mean jt and varlance o? can be expressed in terms of a standard normal random variable Zin the following way: Yeytoz ‘and you can look up the distribution of Z exam, ‘The normal approximation consists of calculating percentiles of a random variable by assuming that It has @ normal distribution. Let (x) be the cumulative distribution function of the standard normal distribution, (The Standard normal distribution has w= 0, 0 = 1. @ is the symbol generally used for this distribution function.) Suppose we are given that X is @ normal random variable with mean p, variance o; we will write X ~ n(u.02) to describe X, And suppose we want to caloulate the 95th percentile of X; in other words, we want a number x such that PrOX sx) = 0.95, We would reason as follows: 4 table of the standard normal distribution function that you get at the Pr(X < x) = 0.95 22H 2-H) 095 of) 0.95 = 2 0-00.95) x= pf +00"(095) Note that °1(0.95) = 1.645 is a commonly used percentile ofthe normal distribution, and i listed at the bottom of the table you get at the exam, ‘You should internalize the above reasoning so you don’t have to write it out each time. Namely, to calculate a Percentile of a random variable being approximated normally, find the value of x such that (x) is that percentile, ‘Then scale x: multiply by the standard deviation, and then wanslate x: add the mean, ‘This method will be used repeatedly throughout the course, SOA MLC Sty Manual eton Cepriah 2609 A 2.3, MIXTURES _ a Exanmve 2B A big fire destroyed a building in which 100 of your insureds live, Each insured has a fre insure policy. The lsses on ths poliey follow distribution with mean 1000 and variance 3,000,000, Bet though all the Poly vie in the same building, the losses are independent, You are now selting up & reserve forthe cost ofthese losses. ‘Using the normal approximation, calculate the size of the reserve you should put up if you want 0 have 8 95% probability of having enough money in the reserve to pay all the claims. Answer: The expected tol Loss isthe sum ofthe means, or (100K1000) = 100.000, ‘The variance of the wi} rae the sum of the vacanees, or 100(3,000,000) = 3 108. a = 3x TOF = 17,320.51. For a standard toss istebution, the 95th percentile is 1,645, We scale ths by 17,320.51 and wanslate it by 100,000: 400,000 + 17,320.51(1.645) = [128,492.24 a 2.3 Mixtures ‘A Gite) mintoredistebution is «random variable X whose distibtion function ean be expresse #5 & weighted fqverage of n distribution function of random variables Xi, f= 1a... In other words, Fo)= DywkGd fa vith tho weights adding up to 1. Since the density funtion 3s the derivative of the distribution function, it aan Nitumne weighted average ofthe individual density functions. If discrete variables are being mixed, the probabilities will have the same property of being weighted averages, For example, suppose X is a mixture of an exponential distribution with mean 100 and vicight 60% and an al distribution with mean 200 and weight 40%. Then the probability that X < 100 is as 0.4889] 6(0.6321) + 0.4(0:3935) Pr(X < 100) = 0.6(1 ~ €"!'") 4 0.4(1 ‘A mixture is not the same as a sum of random variables! ‘The distribution funetion for sum of random a en wnen they are identically distributed —is usually dificult o calculate tis important not to confuse the two, Let's consider the situations where each would be appropriate, sr eun of random vatiables isan appropriate model fora situation whore several distinet events occur. and you are Nerewed in the sum, Bach event may have the same distribution, oF may not. Examples of sums of random variables are: 1. Arandom sample isa sum of random variables, since m events occur, and you want to add up the vals, pos: ‘toy dividing te sum by no ealeulat te sample mean, For arandom sample al the evens are independent and identically distributed, 2. Aggregate oss on a policy with multiple coverages is a sum of random variables, one for each coverhae see etcovners policy has coverage for fire, windslorm, and thet, the aggregate loss for each of these trae coverages could have its own disibution X, and then the aggregate oss fr the entire poliey wou! be 1X, # y+ X5. These distributions would be different, and may or may not be independent ‘Armixture distribution is an appropriate model fora situation where a single event occurs, However, the single event nay be of many different tyes, andthe type is random. For example, let X be the cost ofa dental cain, his sc cingle claim and X has a éistibution function. However, this claim could te for preventative work (Cestiog 1 can basi secvces filings, or major services (crowns). Hach typeof work a distbution X16 405 of are mare for proventaive work, 35% for basi services, and 25% for major services, then the distribution of Ati bea weighted average of the distributions ofthe 3 Xs: P(x) = 04M) +0.35F(2) + 0.257), 1s no) ete dane ats + 0.35%, + 0.25%). In fact, the type of work that occurred is randoms its not tae that every cli i gots preventative, 35% basle, and 25% major. If that were true, there would be less variance in claim size! ‘SOA MLC Sty Mana on Sey 009 ASE 2 ___2. VARIANCE Since « mixture distribution i a single distribution function, it can be used as a model even when there is no ation as given in the last paragraph, if it fits the data well. For calculating means, the mean of a mixture is the weighted average ofthe means of the components. Since the sonsities ate weighted averages and the expected values are integrals of densities, the expected value of a mixture 5 the weighted average of the expected values ofthe componenls, This i true for any moment, not just the fist moment, But this immediately implies thatthe variance of a mixture isnot the weighted average ofthe variances of its Components. You must compute the variance by computing the second moment and then subletting the quste of the mean, Just Brant 2C Losses on an auto liability coverage follow a mixture discbution. Each dstibution inthe mixture bes equal weight, One dstribation has parameters mean 500 and variance 750,000. The other distribution has mean ‘5000 and variance 75,000,000. Calculate the variance of a loss. Answer: Let X be loss size, We have E(X) = 0.5(500 + 5000) = 2750 Ne the second moment, we use E[X?] = Var(X) + BIXP, 30 (x?) = 0.5{(750,000 +. 500%) + (75,000,000 + 5000*)] = 50,500,000 ‘The variance is then calculated from Var(X) = E{X?} — ELX}2, or Var(X) = 50,500,000 — 2750? o Exauris 2D On an auto collision coverage, there are 2 classes of policyholders, A and B. 70% of drivers are in lass A and 30% in class B. The means and variances of losses for the drivers are: (Chass | Mean” [Variance] A | 300 |” 30,000 B_| 800 | 50,000 ‘claim is submited by a randomly selected driver, Caleulate the variance of the sizeof the claim, Answent ‘This is a mixture situa size, ‘single claim, with probabilities of being one type of another, Let X be claim E(X) = 0.7(300) + 0:3(800) = 450 E(X) = 0.7(80,000 + 300%) + 0.3(50,000 +. 800%) = 291,000 ‘Var(X) = 291,000 ~ 450? = [RB SOU a X is also possible to have continuous mixtures. ‘This means thatthe distribution flnction is an integral of parametric distribution functions of random variables, and a parameter vaties according to a distribution function, ‘The latter distribution is called a mixing distribution, Here isan example. Exanseue 2E The number of losses on a homeowner's policy is binor 4. @ Vaties by policyholder uniformly between O and 0.4 Calculate the probability of 2 or more losses for a policyholder. ly distributed with parameters m = $ and SOA MLC Study Manali Cepysieh 62009 24, BERNOULLI SHORTCUT 7 _ Awsiver: For a single policyholder, Pre Pax = In=U-9 19) = 5901-9 "To calculate the probability for a randomly selected policyholder, we integrate over using the uniform density function, which here is i, asthe weight, P(X = ‘This is easier to integrate by substituting 1 1 - 0.6%) 1 - 0.65 ing 222) 12-29) 2.5(1 -065)— BC - 06%) PAX = 0) + Pr(X = 1) = 2.5(1 -0.6°)~ BC - 0.65) = 2.3056 — 1.5889 = 0.7167 ‘Therefore, the probability of 2 or more losses is the complement of 0.7167. 1 - 0.7167 2.4 Bernoulli shortcut ‘A Bernoulli distribution is one where the random variable is either O or 1. Tt is 1 with probability g and 0 with probability 1g. Its mean is q, and its variance is g(1 ~ 9). However, any random variable which can only assume 2 values is a scaled and translated Bernoulli, If X is Bernoulli and Y can only assume the values @ and B, with a having probability q, then Y = (a~D)X +b. This means that the variance of Y is (a ~ bY? Var(X) = (a ~ b)Pq( ~ q). Remember thi ‘To repeat, for any random variable which assumes only 2 values, the variance is the squared difference between the 2 values, times the probabilities ofthe two values. ‘This is faster than slogging through calculation of B(X) and BQ), OK, 80 quickly—if a random variable is equal to 20 70% of the time and 120 30% of the time, what’ its variance? (Answer below!) ‘This shoricut will be especially useful when we study Hattendort's ‘Theorem (Section 32.5). core = (0!) Cox 0 sane) SOW MC Sady Matus esa0 Copyigh ©2009 ASK 24 __2. VARIANCE, Exauix 2F For one-year term life ‘The premium is 30. ‘The probability of death during the year is 0.02. (iil) The company has expenses of 2. Gv) I the insured survives tothe end of the year, the company pays a dividend of 3. Ignore interest. Caleulate the variance in the amount of profit the company makes on this policy. surance policy of 1000: Answer: There are only two possibilities—either the insured dies or he doesn’t—so we have a Bernoulli here, We ‘can ignore premium and expenses, since these don't vary, so they generate no variance. Bither the company pays 1000 (probability 0.02) or it pays 3 (probability 0.98). The variance is therefore (1000 - 3)°(0.02)(0.98) = [19}482.5764 cy ‘A random variable which can only assume one of two values is called a avo point mixture, 2.5 Conditional variance Let's go back to example 2D, Let's solve it the wrong way. ‘Wronc answer Take the weighted average of the variances in each class: ‘Var(X) = 0:7(30,000) + 0:3(50,000) = 36,000 WRONG! ‘What's weong with this answer? Ifyou had 7 claims from drivers in cass A and 3 claims from drivers in class B, the variance would be 7(30,000) + 3(50,000) = 360,000. However, that's not what you have. You have a single claim, which could be from either type of driver. In addition to the variance in elaim size which would exist even if you knew the class the driver came from, there is variance because you don't know which class the driver came from, Using second moments, you can derive the following general formula fora situation like this. Namely, you are calculating the variance of a random variable X. You have one random variable, let's cal tf (For Indicator), which indicates which class the event is coming from, and then you are given the variance of X given the class, / may be Aiscrote (like here, where it has one of two values) or continuous. The formula is: [Conditional Variance Formula Van(X) = Vary(ExEX (10) + Eif Vary(X | 0] @3) Let's apply this equation to example 2D. 1 in this ease can be elther A or B, and has a probability of 0.7 of being in class A and 0.3 of being in class B. is Bernoulli! So we'll apply the previous section's shortcut. We already calculated £[Var(X | 1), the expected value of the two variances, above, and it came out 36,000. Now let’s caloulate the Var/[(X | )], The expected value of X given the cless is cither 300 or 800. So E(X | 1) is 4 Bernoulli random variable with possible values of 300 (probability 70%) or 800 (probability 30%). Its variance is (600 ~ 300)*(0.7)(0.3) = 52,500. So the variance of claim size is 36,000 + 52,500 = 88,500. ‘This matches the answer (o Example 2D. SOA MLC Sud Manual8 eon oprah @n009 ASN 25 EXERCISES FOR LESSON 2 Exercises 2.4. {4-893:9] (point IF X and ¥ are independent random variables, which of the following statements ae true? Le Verlx +7) = Vax) + Var 2. Var{X — ¥} = Var{X] + Var) 3. Varlax + BY) = a? B[X7] — (BEX? + FP ELY?) - DEEL? wi ®) 1.2 ©13 @) 2,3 (B) 1,2,3 4.2, [48-F95:28) (2 points) Two numbers are drawn independently fom a uniform distibaion om (0,1. ‘What is the variance oftheir produat? Oe ®) ie Ow Ow oe 2.3, [418-F99:7) (2 points) A player in a game may: select one of two fair, six-sided dice. Die A has faces marked with 1, 2,3, 4, 5 and 6. Die B has faces marked with 1, 1, 1, 6, 6, and 6. If the player selects Die A, the payoff is cae o the reault of one roll of Die A. I the player selects Die B, the payoff is equal tothe mean ofthe rests of rolls of DieB. “The player would like the variance ofthe payoff o be as small as possible Determine the smallest value of for which the player should select Die B. wl B)2 ©3 ©) 4 @s ‘ua {051-82-92:4) A company sells group tavel-accient life insurance with m payable in the event of = covered individual's death in a travel accident “The gross premium for # group is sl equal to the-expected value plus the standard deviation of the group's aggregate claims. “The standard premium is based on the following assumptions: ) all individual claims within the group are mutually independent; and mng( ~ q) = 2500, where qis the probability of death by tavel acident for an individual Ina cettan group of 100 Ives, the independence assumption fis becmse three specific individuals always iavel together. Ifone dies in an aceident, all three are assumed to dic, Determine the difference between this group's premium and the standard premium. wo (By 15 © 30 @) 45 ©) 60 2.5. You ste given the following information about the random variables X and ¥: Vax) = 9 Gi) Var(Y) = 4 Gil) Var2x-¥) Determine the correlation coefficient of X and ¥. (ayo (B) 025 (©) 050 @) 0.75 @1 SOA MLC Sty Manst—Bth an Enercses come onthe net page. Cegymast 2009 ASM. _ _ 2. VARIANCE 26. {151-82-93:9} (1 point For heath insurance poliey, wended claims will be equal tthe Product of the claims random variable X and a trend random variable ¥. ‘You are given: @ BX) =10 i) Var(x) = 100 E[Y) = 1.20 Gv) Var() = 0.01 (W) Xand ¥ are independent Determine the variance of trended claims, (A) 144 (B) Mas © 146 (©) 147 ® 148 27. X and ¥ are two inde and Var(XY) = 7500, Determine Var(?). Pendent exponentially distributed random variables, You are given that Var(X) = 25 (A) 25 @) 50 © 100 (®) 200 (®) 300 2.8. You are given the following information about a portfolio of insurance risks: % There are thres classes of risks: A, B, and C. + The number of risks in each class, and the ‘mean and standard deviation of claim frequency for each class, are given in the following chert: Number — Claim Frequency of Standard Class | Risks Mean Deviation A | 3000.10 0.20 B | 300 012 02s Cc {| 200 015 03s Determine the standard deviation of claim frequency fora risk r (A) Less than 0.240 B)_Acteast 0.240, but tess than 0.244 (©) Atleast 0.244, bat less than 0.248 (D) Atleast 0.248, but less than 0.252 (B) Atteast 0.252 randomly selected from the portfolio. 22: , ou are given a portfolio of 100 risks in classes, A and B, each having 50 risks. The losses ofthe risks in class A have @ mean of 10 and a standard deviation of 5, Fe ‘or the entire portfolio, the mean loss is 20 with a standard deviation of 15, ‘What is the standard deviation of losses for risks in class B? (A) Less than 9 (B) Atleast 9, but ess than 13 (© Atteast 13, but less than 17 @D) Atteast 17, but less than 21 (B) Atleast 21 SOA MLC Stuy Maral eon Exercises continue on the next page. Copy ©2000 ASA EXERCISES FOR LESSON 2 2 20-11, Use the following information for questions 2,10 and 2.11: Fora group of 1000 insureds in 3 classes, you ae given: ‘Number of insureds] MeanToss Siandard deviation i of loss 300 aocae0 12 300 20 30 200 | 30 Beecee ot) 2.10. The number of claims submitted by each insured is identically distributed for all insureds, 1000 claims are submitted from this group. Using the normal approximation, calelate x such that ther is 95% probability thatthe sum ofthe clsims is less than x. 2cL1. Each insured submits one claim, Using the normal approximation, calculate x such that thore is a 95% probability thatthe sum ofthe clams is Jess than x. 2.12, (CAS3-F04:29} High-Roller Insurance Company insures the cost of injuries to the employees of ACME Dynamic Manufacturing, Ine. ‘+ 30% of injuries are “Fatal” and the sest aro “Permanent Total” (PT), There are no other injury types. 4 Fatal injuries follow a log-logistic = 400and-7 + PY injuries follow « log-lopistic distribution with @ = 600 and 4+ There is a $750 deductible por injury. Cateulate the probability that an injury will result ina claim to High-Roller, (A) Less than 30% B) Atleast 30%, but less than 35% (C) Atleast 35%, but less than 40% (D) Atleast 40%, but less than 45% Atleast 45% 2413. Losses for an insurance coverage follow a distribution which is a mixture of an exponential distribution with, ‘ean 5 and an exponential distibution with mean @. The mean loss size is 7.5. The variance of loss size i 75. Determine the skewness of the toss distribution. ‘SOW MLC Seay Manan) eon ‘Bxerclescondnus onthe net page {Cops 2009 ASH 28 _ 2, VARIANCE 2.14, (4B-92:23) (2 points) You are given thé following: A portfolio of risks consists of 2 classes, A and B. For an individual risk in either class, the number of claims follows « Poisson distribution. ‘Distribution of Claim Frequency Number of Standard Class Exposures Mean Deviation A "500 0.050 0227 B 500 0.210 0561 “Total Portfolio 7,000 Determine the standard devi (A) Less than 0.390 (B) At feast 0.390, but less than 0.410 (©) Atleast 0.410, but less than 0.430 (D) At Tesst 0.430, but less than 0.450 @) Atleast 0.450 tof the claim frequency for the total portfolio. is not 2.18. (C3 Sample:10) An insurance company is negot reached, the claim will be decided in the courts 3 years from now. You are given: ‘© There is a 50% probability that the courts will require the insurance company to make a payment, The amount of the payment, if there is one, has a Jognormal distribution with mean 10-and standard deviation 20. hhave to pay 5 in tegal expenses, * Incither case, ifthe claim is not settled now, the insurance company ‘which will be paid when the claim is decided, 3 years from now. ‘© The most that the insurance company is willing to pay to settle the claim is the expected present value of the claim and legal expenses plus 0.02 times the variance of the present value, ‘+ Present values are calculated using i = 0.04, Calculate the insurance company's maximum settlement value for this claim. (A) 889 @) 9.93 © 1245 ©) 1289 (©) 13.53 26, {181-83-94:6] (2 points) For number of claims WV and aggregate claims S, you are given: @ PAN =D= $f =0,1,25 @) RSI Gi) BS | N= Gv) Var(S |W = 1) (¥) Varts |N=2) Determine Var(S). 19 B) 21 O23 (D) 25 27 Additional released exam questions: SOA M-S05:10,34,39, CAS3-F05:32, SOA M-F06:39 SOA MUC Study Menaal8ieddon Cepynicht 2009 a8 :XERCISE SOLUTIONS FOR LESSON 2 : a 29 Solutions 2, ‘The frst and second are tue by formula (2.1), The third should have squares on the second a and second b, since Vor(aX) = B{(aX)) ~ Bfax}? = a ELC] = EXP for example, (B) 22, ‘The mean ofthe unifoum distribution is} and the second moments, So ‘VantX¥) = B(X2Y?) - BYPEYY =e) Ge 23, The variance of Die A is Die B is Bemoull, only 2 possibilities with probabilities } and values 1 and 6, sp the variance of one tes I js the variance of one toss over n (equation (2.2)). So 32)’ =. The variance of the mea 25 38 in “12 140n > 300 n>? ‘The answer is(3} ©) aah "The numberof accidents for one life, NY, has @ Bernouli distribution with mean q and variance 40° 0). He aec ies aggregate claims isthe variance of mN. Van) = m? Var) = mig(L = 9) = 2500, For 100 independent lives, aggregate claims is LOOmN, with varianes 100 Va(mN) = 100(2500), race ee always traveling together, agaregate claims are Sm with variance 3 Var(n) = 92500), Mf We add tate the ner 37 independent Ives, the variance is 9(2500) ++ 97(2500) = 106(2500). The expected values however, is no different from the totally independent group's expected value. “The difference in premiums is therefore {1663500} - -f100¢2500) = [547815] «B) 28. 22. = Var(2X — ¥) = A(9) +4 ~ 22) CoveK, ¥) Covi, 1) = 4.5 pxr os) ) SSOAMLC Sty Manan eon epynent G10 AS |e 2. VARIANCE 26, E[XY] = (10)(1,20) 007) = (HCx*) (BEY) = (10° + 100)(1.20° +001) = 290 Varo) = 290-12? = [7%] © 2-7. For an exponential variable, the variance isthe square of the mean, Let 9 be the parameter for ¥ ‘Var(XY) = BOVE) ~ BOP EC 7500 = (25 -+ 25)(26*) ~ 256% = 150 o=10 Var(r) = @ = [700] (c) 2.8. This is a mixture distribution, The first moment is BCX) = 0.5(0.10) + 0.3(0.12) + 0.2(0.15) = 0.116 ‘The second moment is BOR") = 0.5(0.10? + 0.20%) + 0,300.12? + 0.25% +.0.2(0.152 + 0.35%) = 0.07707 116? = 0.063614, The standard deviation is VO.053614 = [D:382518]. (2) 2.9. The frst moment is 20 = 4(10 + j1), 50 jty = 30. ‘The second moment is ‘The variance is 0.07707 ~ 208 + 15% = 3 (10° 45? 30? +08) i 625 = £ (1025.09) from which it follows that op = [IE} (C) 2.10, "Since there are 1000 random claims “from the group", this sa sum of 1000 mixture random variables, ‘The Fandom variable fora single claim isa mixtuce with metn 0.5(10) + 0.3(20) + 0.2(30) = 17 ‘and second moment 0.5(244) +0,3(1300) + 0,2(4500) = 1412 = 1123. ‘The mean and variance are multiplied by 1000 for 1000 claims, and the ¥000C17) + TO00CT2B}.645) = [FTI] 80 the variance is 1412 — 172 normal approximation requites SOA MLC Sup Mamit—sthetion Copyngbt Gate ASHE EXERCISE SOLUTIONS FOR LESSON 2 siete 34 dat. Now we have just # sum, not a mixture. ‘The mean ofthe 1000 claims is the same os before, elthoush technically i's calculated as E(X) = 500(10) + 300(20) + 200(30) = 17,000 ‘The variance is (he sum of the variances, oF ‘Var(x) = 500(122) + 300(30%) + 200(60") = 1,062,000 ‘The normal approximation requires x= 17,000-+ -V1,062,000(1.645) = [18,695.23] “The vatiance is lower than in the previous problem (where itis 1123 per elim, er 1,123,000 for 1000 elsims) ‘pecause the uncertainty on who is submitting the claims has been removed. Bin, This i a mixture disiution, and we want PrX > 750), Probabilies are weighted probabites of (he amgrnentdstbutions. For @~ 400, P(X; > 750) = 1~ yippge = 0225483, For @ = 600, Pra > 750) = 1 = {SQANE, = 0.390244, Then 0,3(0:221453) + 0:70:390244) = 0.339607} (B) 23, Let w be the weigh ofthe exponential with mean 5, The second moment is 75 + 7.5? = 131.25, We are given Sw 4 Ow) = 75 Sw + 2C1 — we? = 131.25 ‘50w = 206? + 26? = 131.25 “We solve for w in tems of @ from the fist equation and plug into the second equa (-Ow40=75 wa =e “3-0 is solution 9 = 5. ‘As we plug in, we multiply through by 5 ~ 0. This will introduce the spuri (15 ~ 650 ~ 26%) + 264(5 ~ 8) = 131.25(5 ~ 6) 315 — 500~ 150% +26? + 109? ~ 26° = 131.25(5) - 131.250 50? — 81.250 + 281.25 = 0 81.252 V8i95*= 2008195) _ 81.25 4 31.25 10 10 O=1125 w=06 this also posible to solve for @in terms of w in the frst equation and plug int the second in fac, hat would avoid the quadratic since the terms with 6 cancel "To calculate skewness, we only need £(X*), since we already ‘variance, For an exponential, B(X*) = 60°, 80 know the first and second moments and the LX) = 6(0.6(5") + 0.4(11.25%)) = 3867.1875 BU) - SEL + 0 n= ee 3867.1875 ~ 3(131.25(7-5) + 2(7.5°) 75a 3867.1875 ~ 2953.125 + 843.75, a een oe ‘SOA MLC Stay Mua Bh don {Copyrg @200 ASM 32. 21d, LetJ = the random variable indicating the class, Ver) = Var(IE(N | 1) + B{Var(Nv 1) BIN 11) sa random variable which assumes the value 0.050 half the time and 0.210 half the time, The probabiltes 9F 0.050 and 0.210 are each 0.50, and the difference of the wo values is 0.210 ~ 0,050 = 0.160, so by the Bernoulli shortcut, the variance of EW |) is (0:50)(0.50X0.160%) = 0.0064, Similarly, Var(W | 1) is @ random variable which assumes two values, 0.227? and 0.5612, each one half the lime. ‘The expected value ofthis random variable by the Bernoulli shortcut is $(0.227? +.0.5612) = 0.183125, Putting i all together Var(I¥) = 0.0064 + +0,183125 = 0.189525 o = VOI89525 3] ©) 2:18, |The expected present value ofthe claim is 0.5(10/.04%), and the present value of legal fees is $/1.04%, fora total of 1071.04 = 8.89. We will compute the variance using the conditional variance formula. The legal expenses ae no gancom and have no variance, 80 We'll ignore them. Let / be the indicator variable for whether a payment is required, and X the settement value, Var(X) = Var(8{X | 1) + Bf Varcx |] fre exPected value ofthe claim i 0 50% of tho time and 10/1.04° the other half, Thus the expected value can only have one of two values, tis a Bernoulli random variable, The Bernoulli shortout says that its variance is 2 Yelaern) = 0330-9.) = 9500 ‘The variance of the claim is 0 50% of the tr the variance is therefore and (20/1.04%Y the other 50% of the time. The expected value of 2 | merin]=03(04 (i) }- 158.0629 ‘Therefore, Var(X) = 19.7579 + 158.0629 = 177.8208. The answer is 8.89 +-0.02(177.8208) = [F2AFES] © (C) 2.16, Naturally, if W = 0, both mean and variance are zero. ‘Using the conditional variance formula, efvags jay] = 2428 9 efuts xy] = 2434? Bumts 1p) = 245481 9 Var(BIs | W}) = 30-4? = 14 Var(S) = 941 ‘SOA MLC Sry Marat edon Coppa 200 ASK Part Il Life Contingencies PART, LIFE CONTINGENCIES ‘There are now 2 textbook options for stadying life contingencies: Actuarial Mathematics and Models for Quan- tifying Risks, Many students will use this manual's coverage exclusively and not study cither textbook. However, the following will help you choose whieh textbook to use, if you wish to use one. Actuarial Mathematics is the more mathematically rigorous one. It has much additional material no longer on ‘he syllabus in its tater chapters, which may be handy if you ever need it Models for Quantifying Risks is down-to-earth, and limits its life contingencies coverage to what is on the syllabus, wrote this manual in approximately the same order as Actuarial Mathematics, and with Actuarial Mathemat- ics's notation. Models for Quantifying Risks has somewhat different notation, ‘There isa study note available from the SOA or CAS website discussing the differences in notation, Please note: All references to Models for Quantifying Risk are for the second edition, SO MLC suady Mareat—B0 eon Copy 209 ASH Lesson 3 Survival Distributions: Probability Functions, Life Tables Reading: Actuarial Mathematics 3.1-8.4 or Models for Quantying Risk (2nd 3d eition) 34.1-3.5.2, 3 3d 33.2 3.1 Probability Functions In tis part of the course, we will be studying survival time. Although almost always this wil refer 40 har Heine ican also refer fo any othe station where an object ean “al”, where “ail” ean mean almost anything, racer smething postive. For example, we could be studying the amount of time until student becomes @ Fellow! Lark be the random variable indicating age at death; n other words, the amount of time a newly born baby, age 6, will ive, A probability model wll provide answers to questions like "the probability that X is between 70 and $80" (taslation the probability tat age of death is between 70 and 80). As you leamed in probability, the Basle fuvaton in the probability model isthe cumulative dstibution funtion, P(x) = Prk 2). Using this function, the probability X is between 70 and 80is Pr(10 < X ¢ 80) = F(80) ~ F(70) nce the random variables are continuous, 50 We can be sloppy about whether the inequalities are strict or not, ‘ous ages by differentiating F to obtain the Pr(X = 80) = 0, We can get some idea of the weight of mortality atv density fantion f(3) Te, distribution function isthe probability of not surviving past age x. The opposite coneept isthe probability “of surviving past age x, or Pr(X > 3). This concept is ealled the survival function, oF (x): 5(3) = Pr(X > 2) Jn the loss distributions part ofthe course, a capital S is used instead of @ small s forthe sarie concept, ‘We thus have that s(x) = 1 ~ F(). Ni et the above ate unconditional probability functions, meaning that they ate probabilities as of bith. ‘We usually are interested in conditional probability functions—functions which relate the future lifetime for sown sii aleady age x. To fustrate the difference, imagine that X, age at death, had a uniform distribution on (0,100). “Thos scans that for someone at bith, there ix a 1/100 probability that he will de in any speific year. fn patinlen, Feeatt/1O0 chance tha he will io at age 99, somewhere between his 99th and 100th birthdays. However, for Someone age 99, the probability of dying at age 99 1, And for someone age 98, the probability of dying a age 99 ioe The general formula eating the conditional probability of surviving past age +f given that a persons xis P(X > x40) Prk >») “The conditional distibution is a random variable in its own right. 7) will be the random variable for the future Titslaseot someone age 3,7 stands forme”. We will omit the (x) and just calli? when the meaning is cleat. 7 seer onibuton funtion, adonsity function, and survival function, just like X does. We wil almost always onky talk about 7, not X. Pax > x41) X>2)= [SOA MLC Sty Manat ion 38 Cepehs 200 ASE 36, 3. SURVIVAL DISTRIBUTIONS: PROBABILITY FUNCTIONS, LIFE TABLES 0 os we have stuck to ordinary function notation, In function notation, funetions ae written as letters—many of mich have special meanings—followed by arguments. Thus F(x) isin ordinary function notation, Now let SiadyTnernational Actuarial Notation (TAN). TAN writes functions as letter, lof which have special meaning, but decorates the letters with subscripts and superscripts, both before ad after the letter, many of which have decoretone themselves. You have already encountered some ofthis notation in Theoty of interest, suchas dq, where the lever 4 had a specific meaning, the angled subscript meant something, and the double-dot meant something, You vill fredually lean actuarial notation as we introduce the needed symbols as they come up, igh! now, we introduce »p, and gs. Ps is the probability that someone age x lives # years, or P(X > x+ | ha itis itis «conditional probability, with x being the condition. gz isthe complemest of the probability {hat someone age xdies within ¢ years. When is 1, itis usually omitted. Ie alto have a notation for deferred probabilities. ys isthe probability that someone age x lives 1 years and then dies in the subsequent u years: jugs = Pe(x+< X'S x+t+u|X'> a), When u= Litisusualy omiied. The Aefered probability yup. isn’t too useful, since it would be the same a8 soup, ‘The notation (x) means a life age 2" "Note thatthe subseript xis dhe eondition, You wouldn't want to add up mixed conditions, so an expression lke Pa be aide mobably useless. On the other hand, when multiplying probabifties fora single life, the subserips Nill be dierent. The right subscript on a feetor will equal the sum ofthe right and left subserits of the previeus aor. For example, oPz0 oz would represent the probability of (20) living 10 eats to 30 followed by (30) ying in the next 10 years. This would be the same as qiogao Another way of expressing the same concept would be znra— toa, the probability of (20) dying inthe next 20 years, minus the probability of (20) dying inthe nox 10 years: Notice that by Keeping the contition, (20), the same in both terms, we obtain a meaningtl expression wen To, Ubwaet one from the other: Yet a third way to'express the sme concept is opm ~s9Paz the probability of living 10 years minus the probability of living 20 years. Exaseue 3A. You are given the following mortality table: = & 60 | 0.008 61 | 0.002 62 | 0.003 63 | 0.004 64 | ooos Caleulate the probability that a person age 60 will die sometime between 2 and 5 years from nove Anawexi ‘The actuarial notation for what we are calculating say gzo. One way to calculate this is 38 2pep span. We calculate: 2Pa = PooPar 1 ~ qaX ~ got) = (1 0.001)(1 — 0.002) = 0.997002 Seo ™ 2760 Pa Pea Pos 0,997002(1 — ge2)(1 — gaa) ~ ger) 0.997002(1 —0.003)(1 — 0.004)(1 ~ 0.005) = 0.985085 angio = 0.997002 ~ 0985085 = [ATTA] a Exam 3B You are given that Calculate 510 SOA MLCStty Manes -Biheon opyrig 62009 ASN 3,2, LIFE TABLES é i - 37 into function notation: Answer: Let's anslate actuarial notati sigan = PS < X $461 X 2 40) F(46) ~ FAS) (40) 345) ~ 5(46) sO) 8)" — 0.475624, and (46) = (if) = 0.469131, to obtain 0510204, s(45) We evaluate £40) = ( 0475624 ~ 0.469131 sigs = 9.510204 3.2 Life Tables ‘Acconevete way to Jook at the random variable X isto build life tables. Ina life table, we start with a certain number of lives at age 0, called the radix. We then apply probabilities to compute the expected! number of lives at any other age. In other words, if we lt J; be the expected numberof lives at age x and lo is the radix, ly ~ lgs(x). The number of deaths at each age iid, = fy lyr} Using life able, we can compute p's and q's by taking ratios, For example [Notice that the subscript of p or qis the condition, which isthe subseript of the denominator of the rato. ‘Conversely, one could use p's and g's to go from one ! to another (. ‘An example of a life table isthe Illustrative Life Table in Actvarial Mathematics, which you get @ copy of at the exam. This table is somewhat redundant in providing 1000. as well a f, and d,, but for computing mortality probabilities spanning more than 1 year, the [, column will usually be the one that you use. ‘There are 2 ways to interpreta fife table: 1. We may interprot fy asthe expected number of survivors at age x, and ds a8 the expected mumber of deaths at ‘age x. This is the random survivorship group interpretation ‘2, We may interpret the life table as a decrement table, similar to an interest table (Which isan increment table). In an interest table, one would stat with the principal A and on line x there would be the number A [T}-o(1 + 11) where i, isthe interest paid in ycar j-+ I. Similarly, in aife table, on starts with a radix fp which serves the same purpose as A, and line has f FT} py- This i the deterministic survivorship group interpretation "Both interpretations Jead tothe same life table, The deterministic interpretation is easier to understand and more conetete, and if you ever become an expert witness and must explain your work to jury, you will probably use this interpretation. However, the random interpretation is more flexible sine i allows you to calculate not just expected value but also variance, medians, and other probability measures. Continuation of Example 3A. Redo this problem using life tables. ‘Answer: We will arbitrarily use a radix of 1,000,000 at age 60. Then we recursively calculate ly, x = 61,...,65 wsing levi = bx(1 ~ du) SOA MLC Say Menast—s eon Copa Quo ASHE 38. 3. SURVIVAL DISTRIBUTIONS: PROBABILITY FUNCTIONS, LIFE TABLES a dp 60 | 0.001 | 1,000,000 61 | 0.002 | 999,000 62 | 0.003 | 997,002 63 | 0004} 994,011 64 | 0.005 | 990,035 65 985,085 ‘The answer is (997,002 ~ 985,085)/ 1,000,000 = [6.0117] Exawei 3C You are given the following life table: [e[ Tape 0 50 1 0.98 2 | 890 Calculate apo ‘Answer: We back out lo y= 8. o08.16 098 4a = 908.16 + 50 = 958,16 Hence app = 890/958.2 = [0.02886] Exercises 3.1, [CAS4-$87:16] (1 point) You are given the following survival function: (10000 ~ x%)/10000 0s x < 100 0 x> 100 Caleulate 932 (A) Less than 0.005 (B) Atleast 0.005, but less than 0.006 (©) Atleast 0.006, but less than 0.007 . At least 0.007, but less than 0.008 (2 3.2. ae sts. st) At least 0.008 {CAS4-$88: 16] (1 point) Which of the following are. equivalent to ,px? A) aes ~ uP SB) aude ~ ite + vePs (de rats + Pas DO) ide nude Prvw ©) The comect answer isnot given by (A), (B) (C), oF). SOA MLC Sty Mans! eon Copyright 200s ASM ‘Brerisescomnue on the next poe. EXERCISES FOR LESSON 3 39 33, [CASAA-893:2} (I point) You are given the following information: 1, = 9700 Determine the expected number of survivors to age 5. dus Qy-Qar Leary Se Ad 8 Ago 00? 4506 as 88 4 00% 4083.0 (A) Less than 8,845 ‘Av east 8,845, but less than 8,850 ay {) Atleast 8,850, but tess than 8 855 cae pe tate (D) _AtTeast 8,855, but less than 8,860 er eec) (E) Atleast 8,860 () Atleast ee 3.4, (CASHA-F93:I] (1 point) You are given the following morality able: aaa : =e [ke Te) aPare Sor Pra 30 | 004 | 30,000 | 1,200 Lage Sas tae) aM ae Peta 4] a1 | 0.0803 | 2600 Das: Ror 4-422) 22 | nosed | 27,350 ae bas te 4a) 23 | 0.0700 | 56184 Bay = Avo ta 20) 24 | 00790 | 23,900 |_ doo hoot hin wo years. Determine the probability that a life age 21 will die wi “ @) © ® ® 35. “ @) © ya oy Jack enters a mortality “Which of the following does not express the likelihood of this event? Less than 0.0960 ‘At Teast 0,0960, but less than 0.1050 ‘AtTeast 0.1010, but less than 0.1060 ‘Atleast 0.1060, but Jess than 0.1110 At Teast 0.1110 ops "2965, (65) = 5667) 525) 4ops~ des + den hs sons Pas ‘SOA BILC Stay Manual eon Copyaghn @2000 ASH ads + Aste (a bey ( study at age 25. He dies between ages 65 and 67, las} sia 3) soles (4. stead sts av tas 2465 sob og 1Mer fey her 1 POr- Leo deg 9 O66 Qos ho S166) - St68) 569) 10) Bxercites continue on he nest poe 40 3. SURVIVAL DISTRIBUTIONS; PROBABILITY FUNCTIONS, LIFE TABLES 3.6. (3-800: Fora morality sty on college tots (Suds eerd te sty onthe ithe in 16 ; Youtanoifomaon bot meri fre ithaysn 1962, yy ay «asd ste) Gi) Diek, who turned 20 in 1963, died between his 32” and 33 birthdays, sok (%) Jane, who turned 21 in 1963, was alive on her bitthday in 1998, at which ine she left the study, (¥) All lifetimes are independent, (vi) Likelthoods are based upon the Illustrative Life Table, aha = stsd sti) Calculate the likelihood for these two students, (A) 0.00138 B) 0.00146 ©) 0.00149 ©) 0.00156 (®) 0.00169 3:7. Morality follows the Mlustative Life Table, Jack and Jil are two independent lives of ages 25 and 30 respectively. ayples sy hu Cates the probability of ack and both ng to atleast age 65 but not to age 90. NO> G8). ta.ss6c131 Youare gi Calculate spas. that «as = 0.10 for ¢= 0,1,...,9. ‘ (A) 040 ®) 0.60 oon ©) 080 ®) 081 39. [180-82-94:10] You are given the following: (The probabitity that a person age 20 will survive 30 years is 0.7. Gi)__‘The probabil years is 0.0475, Gi) The probabitity that a person soso © robo 16 Bae that a person age 45 will die within 5 years and that another person age 40 will survive 5 Hote cag) 6 ‘ge 20 wll survive 20 years and that another personage 40 will 5 = 0 oF within 5 ‘years is 0.04, afi -gflas <0 0115 Calculate the probability tht a person age 20 will survive 25 years. «fro : eaiie od soto. § tao Scene (a) 0.74 ®) 075 © 076 0) 0.77 ®) 0.78 340. (CAS4A-892:4) (2 poins) You are given the following mortality table Ps 50s G50 - Oe )ee [apie arg ffs ‘ i = ) 50 | 1,000 [0.020 [70 sresteetecaitecis 51] avo foo33 | 32 dere Bsr Os 32] ve | 0032 | 30 Jon La sy 53| gn. |oost | 38 Rees bev (44s) | 54 | sao | 0.028 | 2 ea Broup oF 800 people age 50, determine the expected number who wil dic while age 4, (A) Less than 21 @) Atteast 2, bat ess than 24 QQ Atteast 24, but less than 27 (D) Atleast 27, but less than 30 ®) Atleast 30 tea) = 0 SOA MLC Stdy Manton Exercises continue on the next pose. Copyigh @2008 ago EXERCISES FOR LESSON 3 _ 3.11. [CAS4A-S98:13) 2 points) You are given the following information: 1. The probability that to 70-year-old ae bath alive in 20 years is 16%, 2, ‘The probability that v0 80-year-olds are both alive in 20 years is 1%, 3, There isan 8% chance of «70-year-old living 30 years. 44_Allfivesare independent and have the same expected morali Determine the probability of an 80-year-old living 10 years. yo = 9 5 (A) Loss than 0.35, (B) At east 0.35, but less than 0.45 (GJ_At eas 0.45, but less than 0.55 (D) Atleast 0.55, but less than 0.65 a aePto = 04 hte = 0.4 wofro = 008 roto 10 Pao = 0-08 rwFeo ww Pay 08 ig life table: or 00100 10 120, 100 Atleast 0465 3,12, (CAS4A-F98:15] (2 points) Light bulbs bur out according tothe followin, Jo 1,000,000 lo 10004 oa 500 1 "800,000 : 600,000 300,000. — 4 0 ‘A new plant has 2,500 light bulbs, Burned out light bulbs are replaced with new Tight bulbs at the end of each year, “What is the expected number of new light bubs that will be needed at the end of year 37 (A) Less than 800 (B) Atleast 800, bu es than 860 (©) Atest 860, but ess than 920 {98 At leas 920, but less than 980 (@_Atteast 980 3.13. [CASAA-$99:12} (2 points) Given the following portion of life table: Siecle tier ee Paeee ae 1,900 85 0875 $US gas us osm 074 750 WS 835 925 aaa er 2» fa oe 200 120 ee to fo pas » 20 1.00 Determine the value of pr paps: pa-psge si Aes 5] 4 (A) Less than 0.055 {B) Atleast 0.055, but less than 0.065 (QS. At Jeast 0.065, but less than 0.075 (D) Atleast 0.075 (B) The answer cannot be determined from the given information. ‘Additional released exam questions: CAS3-S05:29, SOA M-F05:31, CAS3-S07: CAS3L-FO8:14, CASSL-S09:2 ‘SOA MUC Sty Manel eon opyraht 2009 ASM fans Bata ad dye ta Rae cs SS b- a 200 Deby W028. 0.068 rs 0s , CAS3L-S08:14, 2 . 3. SURVIVAL DISTRIBUTIONS: PROBABILITY FUNCTIONS, LIFE TABLES Solutions 3.1, Translate gy into survival functions, aoa = Pr(32- 32) 3.2. We want ps = P(X > x41 | X > x), We can ofiminate (C) and (D) immediately because of mixed conditionality, (A) translates into Pr(x4+1 x)—PX > x44-+u | X > 2), which doesn’t make uch sense cither—why would you subtract the second probability when it isn’t included in the first probability? ‘That leaves (B), which translates into Pe x) P< X sxe t[X> x4 PX > etre X> x) = PUK > xt teulX> 2+ Pree Xs xtra n|X> x) = PUK > x4t1X>x) which is exactly what we want, (B) 3.3, We reeursively compute through x = 5. h (1 ~ gx) = 9700(1 — 0.020) = 9506 In(1 ~ ga) = 9506(1 ~ 0.020) = 9315.88 Wy = by dy = 9315.88 ~ 232 = 9083.88, Js = 1a(] ~ qa) = 9083.88(1 ~ 0.026) = [8847,70] — (B) 3d. We need tay and fy 30,000 — 1,200 = 28,800 3,900/(1 = 0.0700) = 25,698.92 28,800 ~ 25,698.92 28,800 2 ©) 35. All of these expressions are fine except for (D), which should have des + dg in the numerator. 3.6. For independent lifetimes, we multiply the likelihood for each life together to get the likelihood ofthe joint event. For David, the condition is age-20, and death occurs at age 32, so we need fit = For Jan, the condition is age 21 and she survived to age 56,80 we need Looking up the Iustrative Life Table, we find S08 MLC Sty Momat—Bekdon ‘Copa 2009 ASM EXERCISE SOLUTIONS FOR LESSON 3 HEE 43 ea 20 9,617,802 21 9,607,896 32 9,471,591 33 9,455,522 56 8,563,435, 22)/8.563.435) _ reapag0] '\Gisot as) (CT no) fit. For Jac, we ned and fr J we need 2. From the seat Life Table, we ave ‘The answer is Pan 259,565,017 30 9,501,381 65 7,533,964 901,058,491 oid 7,533,964 — 1,058,491)/7,533,964 — 1,058,491) 9,565,017 9,501,381 » probabilities of death in 2 con- ‘Frtve yeas of 5 consecutive years, a8 needed. Here, we want the probability of survival 7 years, given Su". val 5 years, Les use the survival function conditional on living to OF ry (we'll omit the subscrip). "Then (GB) ‘the qs are all contin on x and this cn be added together 0 ob 565) = 1 9Qs = ngs “aga = 0.5, and (7) = 03,80 She 9) Late 28) (1) _ 03 sexvay ~ 86097 cone yn PEG)" 08 6) ay sees 39. In aproblem like this, your first step should be to determine what the variables are and what the equations are, Thero should be the same number of equations and variables. Tn this problem, If we analyze the 3 statements and the probability were looking for, we notes that. a probs: bitten stator end at ages 20, 40, 45, or 50. ‘The variables are therefore opan, sP4, snd spas, and everything can Be expressed in terms Of these. Let w= 29Pz0. = Pao and W = spas, Then the 3 statements give ws these 3 equations: : ww 07 a ud = w) = 0.0475 (1 =v) = 0.04 ‘We must find ww. I's therefore reasonable to substitu w = SZ into the second equation: 0 ye xt |X > x)= e(ro) > 4). The function iis the comulative distibution Function for T(x), ot F7y9(). The version of equation (4.3) for Pci enol fel) as By a change in variables, we can write this as 43) ‘What you should remember about these two equations is if you integrate p, from a lower bound to an upper bound, ‘and then exponentiate the negative of the integral, you get the probability of survival to the age represented by the upper bound, conditional on survival to the age represented by the lower bound. "Te random vara Xl a apecal ease of (a; i8 TO). ‘SOA MIC Study Manual eon 45 Capen 209 AS 4, SURVIVAL DISTRIBUTIONS: FORCE OF MORTALITY Exauus 4A You are given? that ps() = 1/(100 +1) 1, Calculate sopas. 2, Caleutate 209s. 3. Calculate pogo ‘Awswer: 1. We want the probability of survival {0 45 given survival to 35, so we'll integrate from age 35 to age 45, 1ys(0) represents age 35 and p35(10) represents age 45, 1opss = exp{— if st as) = ds EMPL J, Toor, = exp(~(In 110 ~ In 100)) = exp 2. We calculate the probability of survival to 65 given survival to 45 and then take the complement, ° sors «o0(- [ nstoas) ho ie ds ) ' sexp(- [> tt PL io TOFS, = exp(~(In 130 ~ In 110) 110 Pin 39 * 13 200 3. This is not survival to age y given survival to age x rather iis survival 1 age y but not to age z given survival to age x. We need two integrals to do this. We already saw in the previous two solutions that for this force of ™moriaity, the probability that (2) survives for ¢ years isthe logarithm of 100-+ x-+ minus the logarithm of 100 + x, 20940 = s0P40 ~ 30240 1s s =exn(- sts) exo(~ f soso} Is Is ‘exp (In 105 ~ In 115) ~ exp (In 105 ~ In 135) 105 _ 105 1B is 5266) o this 3 Porto, a poor model for human moti, ad is used only to lastate the concep of calculating probabilities fom the free of mortal. SOA MLC Seay Manu —2m etn Ceppsigh O08 ASHI ) 4, SURVIVAL DISTRIBUTIONS: FORCE OF MORTALITY 47 4 02! a6] 0.12} 0.08 0.04] ® ED 7 eas Eo) 40 Figure 4.1: Graph of ax ‘Quiz dt ? The force of mortality sx = 0.001 Vs. wane = whe 40 Fo rive orbit of some age 20 surviving 30 yeas and then dying in he TEX, 10 years. By equation G1), the density fanetion for X iss pe enmierics a ik ful@) = SCM Mesias | q Let's caleuate the probability density function fr T(x) in terms of lO) “The probability density function for T(3) Here tive of he dervalie wih respect tof the survival function for 7) “The survival fonction for T(x) is PAK > xe) _ sxOrH) epiray> =P > 241X299" Borey 7 ax) arent) = Po((a) > #) = Pr > 241K) = “BGS a) x) “the nogatve ofthe derivative ofthis with respect tts f(s + 9/sx(2). On te nent Jine, the second equality is by equation (4.1): LGD _ Sx + Ol y= HED fro) = Gy Sxl) “The distribution fonction isthe integral ofthe density function, s0 10 get the ‘unconditional probability that 72) is in (a, 6), we would write: f PaBigut ar mee fort be ein fres of ment, the val fvetion mis 30107028 nn ee eh tm iniy 2080 nay. a dos en eae mor uncon wl Heep nsening sense or ene DSN 30 or 0. = west) (4.6) Pr(a< T(2) © ssuetoe aoa uae) memati , MMi 2 6 toes ° CaleulateaP ¢ re subject 1 extra moe Bors, age 45, is recovering fom SBE jer the py for & ‘ag, For a standad fife, ps = O98. 1 ty Therefore, the apying Lo BOCES Boni oa for x beeen 45 and 50. Te n=rase rromerd ies 0.002 at = 45, desressing straight fine 100 at age 50. Calculate spas for BOF ie er a 9a tai pias 1 $0002. ‘son MLC Sty Mansa 8 _enecses comin on the 2 POBE Seppe ash 4 a force of mortal 0 De (A) (B) «© ® 43, wo Gii) Dete 20 -in m1, (A) Land (©) The correct answer is not 4. SURVIVAL DISTRIBUTIONS: FORCE oF wey TALITY 4.10, Determine gy, (4-886:26) You ar given y, = Tomi for 0 = x < 100, nat ter eh Ae 1 @ tno 4 he 2x41 10,000 — 33 4x42 10,000 ~ 3? 6x43 10,000 — 37 2x41 29,999 3,2 Orth 29,999 323 au (A) 3 ®) oem 42 Ad nes © 1 = wo. - Ue 2 000. ¥2 ©) ® 1. [4886:31] A mocalty table has 8 force of morta HY Hin and mortality rate 9°, ity and monty rate g You are given A second mortality table has - Sosmende ae ee 13 Aay?® = a 1~ yixq (B) var (C) 0.59, termine the value of ‘Less than 0,150 Atleast 0.150, but le At least 0.155, n tho revised table, 8 than 0.155 i less than 0.160 but less than 0.165 6 aa an Atleast 0.165 wR Coa 4 Staaten Ben = Hoy -kO sts 1 9: = 0, where 4. is based o mek 1150-S88:1) You are given, ae Pe ett n the Force of mortality iy, Vee ted ate tnd tate Ps following functions can serve as a B>00O.x20 &> 0050220 Bore 1-05 ve ty) X i ° Ace In Leonor, 7 Tonly ®) Yand 1 ony (C) Wand tt ony (©) 1, Wana my * given by (A), (B), (C), or ee samt | SARS Sht Connigh 2am ashe Prertes conn onthe next page. EXERCISES FOR LESSON 4 4s. [CAS4A-199:13} (2 points) Which ofthe following equations define valid mortality functions? 4 ; 1 wae (4a? 720 mone e: jiale < ¢™ * a ajmonsonr 720% g- Seoeenenl te se easea | so 3 hkl an o oy MD abs oe 0) ne peas 9 tebe : wt 2 © 12 13 @ 23 suas, (CASB-FOAT Which ft flowing formulas OPT force of morality? aie Wctas ace AG amie aaa ait 2 pezadtay!, a7 0b>0 v ek elbO\S - ¢ ns ays ape Peer anny? )* s(n? a meena, BO % es Mea () Lonly @) 2only (© only (pf Land 2only— &) Land only darn, 150-286:15] You ae ven FO) =~ sar FFE {Which ofthe following are true? 1 pede oY nn wys0a Yt a: te Th wrens Mal: Bs os seal Se (A) Land IT only * qa) Tand 1 only (C) Hand Hi only (ph 1, Wend (B) The correct answer is mot given by (A), B), sor D)- 4.18, {150-590212) You are given ee Sey i) pe Atel for x20 afo= Setar erat ag - i) spo 050 - 0 ete ea cdeeecal eabined ee LO cen eee a4 «0 OBRE (A) -0.26 (B) -0.09 (©) 0.00 IR) 0.09 (B) 026 4.19, [150-81-94:48} ‘You arc given: w= L22* osxs00 Caloulate the difference between the foree of mortality at age 1, an te probability that (1) dies before age 2 «oy 0007 (B) 0010 poor ©) 0016 @ 0024 ‘308 MLC Say Maral e800 Bzercses continue on the wo PORE ‘Coat 1009 ASM 420. (150-82-94:18) You are given: © Re tee kme, te Sasa, GS = te fvrna, ve MUM ia Gil) kisa Positive constant Determi exude ea ne an expression fork such that § = Ko he (A) In paves = 299) . B) lA ~ Faay/ct = p,y} © ~ 2p ~ Po ©) tl a/c ~ 393) GX Inlet ~39a70 49) 421. [CAss.ss6: force of mortality 7, 71 point) Standord Me = “are subject t force of mortality. Substendard lives are subject to Let as and 4 be mortality rte for standard and substandard fives respectively. Express 4, in terms of q, ee ee @ ax Med elkecditi eds as lo ® Go ~29, Bee es tes a is att 24 -Gy ode a the ay? D) 2004)? 9 ©) 29. + (Gy 422, {CASHA-S92:16} (2 points) You an #50005 5 I. For a preferred, Determine ¢ such that g, e given that psp insured, the force of mortality i ‘8 reduced by 25%, 0.95 for a standard insured wi th force of mortality is fans for 0 Sts 1, AS Less than 0.014 Poe 9 Whe x +! |X > x) = PAX > 0. In other Words, «ps. the probability of surviving ¢ years if ene is age (2), is independent ofthe aget Clearly an ‘unrealistic ‘sumplfon-—mortaliy nereases by 98% aa ih leds oan exponent station of vial ime is Const of Mortality. In a me an forall 5 he spol ofthe exponential ame other words, aie with ater ors He ony has expected fore iim with variance yo eRNaSS ofthe lifes eurent age. tore ae tne probability fnetions fora constant force of mortal H- o ‘504 MC Sy Mannion Cepia 200 ASHE "5. SURVIVAL DISTRIBUTIONS: MORTALITY 5.1.2. Uniform Distribution, or de Moivre's Law {he uniform distitution on 0,6) has a mean off, Tosa) ae Aer instead of 0 in survive model, so the survival lw saps ea na {0.4 de Moivre proposed this distribution a8 a survival ine distibotea While de Moivre's law does have memory-—afer al, ‘ou havo uni certain death—it has a similar property de Moivre's law with parameter wand one ts medion and midrange. Is vatince is. tis wadiona to is uniformly disteibuted on called de Moivie’s law, You can’t live beyond 6, 50 the older you are, the less time h simplifies calculation, Namely, if survival time follows is ago x, remaining lifetime follows de Moivre’s law: Parameter, ding tn fea Fontions fr de Mole’ aw wth parame en Youshen ke ee them by Ina on aga,” 2 MOURN ar werking ou several probly sould seeps ae gee force of mortality on sight (5.1) 6.2) Exanetn SA ‘The force of mortality for (30) is Wee Calculate the probability of dying in his 70's, Answii We recognize this force of mortalit, x>30 1 100 = 'y a8 de Moivre's law with es = 100, For), remaining tine is de Moive with w= 70. We need pan ape _ 20-403 a= BE# 3 502 nes SF So the answer is } — © 5.1.3. Beta Distribution, or Generalized de Moivie's law A generalized version of de Moivre’ law has Mee a (53) Where a is a postive real number. (The ungeneralized for fn of de Moivre's aw has a = 1,) Multiplying the original form by a constant raises the probabilities to that power, 8 we learned in the previous lesson, So fo~x~ 1" od ae 54) pen (treat ¢ ratte ice property that for (2) fare lifetime follows the same lw with {onetion is no longer a constant, making computations of insurance at impossible todo in closed form, Thus, exam questions using generalaed de \e questions in the previous lesson, SOA BEC stady Meo 2h ion enya 200) ashe 5.2. MORTALITY LAWS THAT MAY BE USED FOR HUMAN MORTALITY 6 5.2 Mortality Laws that May be Used for Human Mortality §.2.1 Gomperiz's Law Gompertz proposed the force of mortality, By = Bet ‘with appropriate parameters B and e, He based this law on studies in which the logarithm of the force of mortality over a wide range of ages above 20 appeared to fit well to a straight line, ‘You can derive the distribution and density functions using the formulas from the previous lesson, Memorizing the force of mortality should be more than adequate for the exam; in fact, just being aware that this law exists should be adequate. 6.2.2. Makeham’s Law Makeham improved Gompertz’s law by adding a third parameter A: Me = A+ Be* ‘A ropresents the constant part ofthe force of mortality, mortality that is independent of age and is due to accidental ‘causes, Bc* represents mortality resulting from deterioration and degeneration, which increases exponentially by ‘age, Makeham's law provides a good fit for ages above 20, and was used to build the Illustrative Life Table. ‘Makeham's and Gompertz's laws have nice properties for simplifying joint life functions, However, that has been removed from the syllabus. So reference to these laws on exams will be rare. Just be aware that they exist. 5.2.3 Weibull Distribution For the Weibull dstrbution, the cumolativedistibutionfanetions Fa)= 1-2 If you log, negate, and differentiate the survival function, you will obtain ts ‘Actuarial Mathematics vses a different, more taditional, parametrization of the Weibull, # = kx", You can then derive probability functions inthe usual manner, “The Weibull distribution is flexible in that n (or r) can be set 10 allow mortality to have @ reasonable pattern. It n = 0 (r = 1) the Weibull reduces to an exponential distribution (which, as we discussed, is nor a reasonable ‘mortality assumption). But if you make n greater than 0 (or r > 1), the force of mortality increases with age, so it becomes more reasonable. Rather than using it for human mortality, it is more commonly used for other types of failure, ike machine breakdown, Exercises Sk. [CASH-S85:17] (1 point) Morality follows Makehan’s lav, te = A+ Be Which ofthe following represents Psst? ® Ps ®) a or Mo ) He 52 [CAS4-S86:16] (1 point) Mortality follows de Moivie’s lw with « = 105. Calculate on0925: = wl, sobs pee eae etree ray ace @ UB @) 1/6 © us (5) 14 &) 38 HOA MLC Sly Manat eon Exerclses comine onthe ax page Capps 200 ASM $. SURVIVAL DISTRIBUTIONS: MORTALITY LAWS 5.3, [CAS4A-S98:14] (1 point) Calculate the probability that a 40-year-old will survive to age 42 if mortality follows Weibul’s law: ue = Ax" with k= 1/100 and w= 1. yt fe (A) Less than 020 Sasa. uta te) At least 0.20, but less than 0.30 rs @ Se Beer ea At least 0.30, but less than 0,40 Atleast 0.40, but less than 0.50 (B) Atleast 0.50 54. Mortality over age 80 follows de Moivre's law with «» = 100, Below age 80, the force of mortality is constant, HO} woe = EMM 4. BonOH Calculate the Force of mortality below age 80, 88, Mortality follows de Molvre’s law with w= 120, Sf" VE ek Caleulete qs990. = aPy an 86-4 9 08s 5:6, Mortality follows de Moive's lav. Youare given hat ge = 1/45. 1s 4s Determines. 4 oe 5.7. [CAS4A-196:5] (2 points) A population of 20,000 lives has two subpopulations. ‘The fist subpopulation has 10,000 fives, all age 30, with their mortality described by a de Moivre survival function where w = 100. ‘The ‘other subpopulation has 10,000 lives, all age 40, with a de Moivre survival function where « = 90, Determine the expected number of people from this population of 20,000 who will die between the ages 5O aid 60. (A) Less than 3,300 a He wl BL Atleast 3,300, but fess than 3,500 sti itiqg ele 8Fe XO Ww NO (©) Atleast 3,500, but less than 3,700 (D) At least 3,700, but les than 3,900 (2) Atleast 3,900 38, {CAS4-F82:16) In acerain population, the force of mortality is constant If the probability that «life age 60 will survive to age 80 is 0.10, what i the force of mortality? (A) Less than 0.10 robo 2040 = OM"? GP) Atleast 0.10, but less than 0.12 (C) Atleast 0.12, but less than 0.14 (D) Atleast 0,14, but less than 0,16 (BE) Atleast 0.16 5.9. Yolanda's motality follows de Moivre’s law with w = 100, sy i the force of mortality for Yotanda. Zinny’s moutlity is subject toa force of mortality He, where 1, = pet A, Zinny’s mortality rate a age 15, qs, is twice Yolandy’s. avy. 4. c= : Determine A. iw. 4 np ES aban de) a 5.10, Mortality follows Gomperte's law with B= 0.001 and c Dotermine x such that yi, is maximized, We. 0% 105, 1. eo Merde gn Mee hs am, ale gah ween, . oe ee bh MOY gs coment Ie ct Meta pease aaeneas Copy 2009 Asst eects EXERCISES FOR LESSONS $.11. [150-82-94:24] You are given: {iu denotes the fore of moti under Gompertz’ aw at ge here = 9.8 ande= oR, get Gti Gets the oe of moi under Weil! aw at age wit Poce of marty Hy where k= O.1 and n> 0. Gi) p= aceasta cot Calculate m (A 05 10 Obs @) 20 (B) 25 Notes the following problem es key o be on a current exam, since slicing has been moved to Exam C4. 5.12, {SOA3-03:17) 7 the four ifetime of (0), has a spliced distebotion, @ fil follows the Ilustrative Life Table, Gi) fA) follows DeMoivte’s law with «= fk, 081550 ee ene so kus nes ~ mye 1 f0+79 025) - [g] More generally, as we will lean in lesson 8 (page 115), when uniform distribution of deaths is assumed, nb} cahaz x < 100. 2 ys 8 constant, 2. Once again we use formuta (6.7). exnsa = > kaytae + 50:0P20 & 2 e+ sof) sen). ~ [34.0828] ‘SOA MLC Swdy MansslBh eon Copyign 2009 ASK EXERCISES FOR LESSON 6 Table 6.1: Formula summary Expected value (ronan) =2 [ane (Kooany)]= Sor Dips Second moment tn this cas, the slationshp to the complete temporary life expectancy is a8 follows. We stw above tht Biygy = 34375, For ages at death between 20 and 70, the curate temporary Tie expectancy is } lower, 2 ee nouiference forages at death above 70, Therefore the difference between the complete and aonee ectacts it ssn = (G) = se which ste diference between 34.375 and 34,0625, 3, We will use (6.7) o calculate the second moment. : eleoo']= 34° wen ‘i wyP 1 (@sy80x159) 42009) ons because Sif 2 = MeeYRHE 1c douifl hat an exam wold expe you to know this formula “The variance is then 2093.5 ~ 39.5? = [$3825], o "The formulas in this lesson are summarized in table 6.1. Exercises Gc. A-porson age 70 is subject to the following force of mortality: sox Soo owt oon rs5 0H 7 . Pe {mating g Boonte §; cera Gas cont 1s00r , gt? Calculate pp forthis person, thy. fA cteat = JF enmbars Je enesemh ay 62. Alife is subject to a constant force of morality. You are given tht en = 24 Determine te ae ol Mo 0a0t SOA LC Say marl -eries continue om he es page py 02009 ASM 6.3, [4-88 { (Morality follows de Moivre’ taw, ! Gq =30. > HI 4 wegen ; Calentate gy, 4. 3 ‘ (&) 1/80 68) 160 © 161 ©) 1402 © 170 ‘ 6.4, [150-S88:11] You are given: : he = (100 19°8,0 5. < 100 ° * nal’ | O) Mam 2887 (ecar. Peete aca al nate Caleutate [PF pre sisirde ros reso es te (A) 3.67 (B) 5.00 ©) 11.33 (D) 19.67 (8) 24.67 I 6.5, [150-891:23) A survival function, (x), is defined as follows: ‘ w=(I-2) osxsurro vee ONY \ For age y, 0 < y <, you are given: te on : e,. { Lyd @O wy 201 n geen eee Geo ( G) yeas Hee wey Caleulate r. anaes aE 0, : is we B)3 os pT @s a ‘ 6.6, [150-81-94:8} You are given: t s@)=(1- 2)", 0.22 <0, where wis postive constant ‘atculate, by Bw Calculate pr,» os i @ aw a aw x) Ron O or O Oe © Cite 67. [150-83-96:25) You are given: : @ s@)= oc re eho o GD eo = Peo Stag tety acu ! Calculate dp os \ (@) 10 ®) 20 © 3 ~ 40 & 50 § ng SOA MLE Stay Me Cops Gatos ASHE elses continue an he nest page EXERCISES FORLESSON6 _ 83 6.8, (150-F97:1] For the current type of refrigerator, you are given: @ sQ)=)-g0sxs0 Gi) %=20 wo 208 For a proposed new type, with the same c, the new survival function is: 1 Osxs5 (wal ~5), S rity Een bane oy trees: Sr he E020 4 ants Cr Gi) EEN) =02 “ (iv) E[minr, 9] 0.0057, 0 <1 < 60 Caleulate the complete expectation of lie at 40. ws wy 35 ow was Mo 6.16, (3-FO1:1] You ere given: eat Cet oa (9% O<2<4 tts 0 eect MON YO05, x>40 yyy ge Sy Pardl -j,, oor Caleulate dagen. f° BSA = PS yaom ge FE mee wong, © CREME EPTS erg wos @) 140 ® 144 © ws () 152 98, 15.6 6.17. Fora life whose survival function is “ 7 wo Lome Fa Yyou are given that ejay = 18 cae 2 1 ao Determine « an ons SOA MLE Sey Monet ain eres conto the nes page opps One ASM EXERCISES FOR LESSON 6 ec = Jnts) You are givon the following information about two lives: Future Lifetime Random Variable = Constant force of mortality Hs F Constant force of mortality = 0.20 Determine the rato of (as expacted future lifetime between ages xand x + 10 to (y's expected future Wfetime 6.18, (CAS4A-F97:13) (2 pe § osoat between ages y and y + 10. fain» fata. (reetmal « Euae (A) Less than 1.00 bys te p020, a Wana = | eres ee (B) Atleast 1.00, but less than 1.25 Brent fy are’ ra ‘Avieast 1.25, but less than 1.50 ote ) A east 1.50, but less than 1.75 See Net (G) Atleast 1.75 6.49. [CAS4-890:2] (1 point) Morality follows ly = 10001 ~ 35) for 0 = x $ 100. Caleulate e99 stg) te Mote 100 (A) Less than 4.2 : a : (B) Atleast 4,2, but less than 4.4 ao Cao, tedOrt 2 as (GQ) Atleast 4.4, but less than 4.6 (D) Atleast 4.6, but less than 4.8 @) Atleast 4.8 bution for the amount of time until burning out: | Time ¢ in hours FO a (0-4800 0 NY, ety | gcastyniaon | 92000 Le 1 6000 6.20. ‘Light bulbs have the following di ach bulb uses 0.015 kilowatt-hours of electricity per hour. Calculate the expected nuinber of kilowatt-hours used by 50 bulbs in ther first 5000 hours. 6.2. (CAS4-S90:1)@ pn Yon weve ec () Morally flow deMoimsston, (etek EY A) (i) dm=as wh Coteuate the variance ofthe fue lifetime ofa personage 20, Var T(20), to the nearest integer (A) 108 (B) 215 (©) 350 ¥Q 615 (&) 700 6.22, (150-F87:11} You are given: (Mortality follows de Moivre’s law. Gi) Va(60)) = 192, wD Catelate i) 98 (B) 100 (C) 107 @) 110 (&) 114 on cs a en res conto te Pt Copygs 2005 ASM fae 6. SURVIVAL DISTRIBUTIONS: MOMENTS 6.23. (CAS4A-894:3} (2 points) You are given: 10) = 1-4 foros wwe tone 0) = 1 ig for 0 < x's 100 Determine Var(7(20)). ia00 -20}" 9% Less than 600 . (B)_Atteast 600, bt tess than 800 (©) Atleast 800, bat less than 1,000 (D) _Atteast 1,000, but less than 1,200 (®) Atleast 1,200 (624) Fora ite with survival unetion ce i nce you are given i: 2h carear ("Ped @ w>40 Sosa Gi) For a lite age 30, the variance of the number of years lived between 30 and 40 is 3.5755. Determine ea, 625; (150-889:A1} You ate given the survivel function s(x) = e-® for x > 0, Calculate each of the following, @ soto chy eerie sto Cb 30 Gy) va(r@o) 6.26, (150-F89:2] You aro gi (Mortality follows de Moivre’s law, (i) Yax(7U5)) = 675 wr W105 Calculate 295. 05-1, cans 9) 400 © 25 @ 450 @ 45 6.27. (3-800:1) Given: @ & OP 4 we se Gi) k=w-1 0x50 Gil) 7G) is the future tifetime random vorinble, Caleuate Varf7UD), <0" (A) 65 (By 93 W133 ©) 178 ®) 333 6.28, Mortality follows the Iltustrative Life Table. Caleutate the variance of the number of complete years lived by (67) before reaching age 70, Cora) fea a aPor s aVor SCAMLC Saal Manest_€h con ercses continue onthe net page. Copyngiecuany asa EXERCISES FOR LESSON 6 6.29, (SOA3-F03:28) For (x): (i) Kis the curtate future lifetime random variable. GI) quae = OGD, BON 250009 Caleulte Van A), « J veenates (Ft) Lt @ 12 1s 30, You ae given that y = 03, det = 0.5, 2 = 07 Cathe vance of uta foe etme, Vx( C2) bebe Pe Rear Pan BO Ven) rant on re Qayead 09 1 Vomoe esoGogar- (04+ 0% OU L0G 08-07) @) 4 ©) 15 id ga03 (02,4 serosa Sox 0504]. [07107 05109.65:.097" oat 631. You are given: Gi) ‘The foree of mortality is the constant yz. exponencial ; Gi) e249. = ky 3 weet Hmong tee et Calculate the revised value of eas. Additional released exam questions: CAS3-807: Solutions 6.1. Use equation (6.1). eo ‘To calculate ,pyo, we consider two cases: 1 < 5 and ¢ > exponentiated integra of so), That integral is "9° for # < $ and & example of how to calculate zor.) So aon ferme = 100(1- a) the force of mortality is changed to the constant + 0.01. OA M-805:21, CAS3-FOS:10, SOA M-FO5:13,14, SOA M-FO6:2,23, SOA MLC-S07:21, CAS3L-S08:13, CAS3L-SO9:1 {ea For! <5, Px 291, For 1 > 5, :pro is negative the -005-000U-9) for ¢ > 5, (See exercise 4.1 for an 85-0000-, seo [eam s = 100(1-<*%).e%%50) = 48771 447.5614 =| 6.2, Use formula (6.9). ‘Summing up the geometric series with rat ‘SOA MLC Sey Manus—Bh eon Cepyash 2009 ashe 33.d385, (01940822 88 6, SURVIVAL DISTRIBUTIONS: MOMENTS 6.3, We know that for de Moivre's law, life expectancy is the midrange. If life expectancy at 30 is 30, then w 64, By equation (6.4) the differenes beiween Byam and {2 ,pycpant Is 28oapys- We calculate: =e Hence pss tts d 167], (A), 65. For the generalized de Moivre, wy = ;£5. Let's calculate the life expéctancy aty ae [728 C0 ‘Therefore, we are given these 2 equations together: 6.6. We savy in the last problem that st, and &, = 82. Multiplying together gets (A). 6.1, This is a generalized de Moivre with « = and exponent a = 4, From the previous problems, we know Sa. Using (i), 40 _ 2(K2 ~ 80) wa” 3p = 120 that for such a mortality law, Then 2g = 190 = [AG] (Dy 68, The current ype is deMolvre, so w = 2éy = 40. The new type is sre to survive 5 years, then deMoivre over 35 years thereafter, so the conditional expectation after surviving, 5 years is 4(35) = 17.5, fo total expected survival time of + 17.5 = 22.5, which is [25] years longer. (R2) SOA MLC Smy Manual etn {Copren x00 ASM EXERCISE SOLUTIONS FOR LESSON 6 an ie 89 69. inst we must solve fork fi xo ar) fP@-oian een _-4@-oet! Sa _ Heo = Gaye = fie =» B= 40 = 20 — 80) B= 120 We are given that 240 = 22en, 50 "Then deo 60) = (2).60) = [45] ®) Pia ana de Mote with «y= 720 months. Remaining tine tow (72) i 6,90 ble mea 30, The waver of est ten the narnber of hens times eggs per month mes lif expectancy, ‘or (100,30330) = [905000 © Ga. Forde Moive's law, life expectancy is hal the limiting age increases by 8, [108] ®) G12, Under deMoive's la, life expetation is al of remalning life, 900 = 26-200 fast 2) (Eig) Wewil do tis problem using the wapezoidal method inconuneson wih eat (1), This means we \ gift sum up the areas of trapezoids from 21 to 25, 25 to 30, 30 to 35, and 35 to 40. “pn at 25, 3, 35, and 40 is the number of patients remaining hospitalized at 25, 30, 35, and 40 divided by the a eat patents tne 21, shih is 08053488) + 0.217.384) = 46,2672. Lers hold off dividing by 46,267.2 ‘50 if expectancy increases 4, the limiting age 80. Then qao = aaia6 = ‘iy uti thee We sum up the areas ofthe trapezoids (times 46,267.2)- ca 4 Range of Average hospitalized Length of interval Product wg bmn 21-25 31,825.6 4 127,302.4 25-30 11 366.5 5 36,832.5 oe 30-35 3,343.0 5 16,715.0 ES 35-40 668.5 5 3,342.5, Som 204,192.4 “The average time is 204,192.4/46,267.2 = [4341883] (A) Gad, Toemenn residual i or Part, 2, sf for a wo-parameter Paco, From is orm 1 immediately follows that 230 "you didn't kno this formula, though, you could use equation (6.1) and nie From Table 1.1: ‘SOA MLC Sy Maus —t0 es, Copy 2009 AME 90 a 6. SURVIVAL DISTRIBUTIONS: MOMENTS so (er) Where xi8 age, Now, ,pao = “280, go tba). (én) 70 3 tax) From equation (6.1), t= [peat sf "« oe we ea en alee 70° “wy 6.13. The full exy Pectation of life i the expectation bounded at plus the probability ofsurvvel to ¢ times mean residual life att, ECT) = Efmin(7, 40)] + 0 (anp0) 62 = (40 0.005(40%) + 2(06) = 324 849(0.6) 2-32 t= Fy -B] © 80 we have to compute ,pas, 6.16. We want to use equation (6.5), vrs we het ae 1315 20 = fst e015 1s Now we calculate és, 1s s deen = if ipodee £ Past 5 2s = fC ettans [i veaweing hb hs = 25(1 ~ 6) + 206"08(1 — @P5) =25~- 5°06 - 996°) = 25 ~ 5(0.548812) - 20(0332871) ®) S08 MLC Staly Mapua don opi 2009 ashe EXERCISE SOLUTIONS FOR LESSON 6 ae om 6.17. We use equation (6.7) for this de Moivre distribution. 1% onan EMG a (409019) + 2010 ~30) _ w—10 aa 190 + 201 ~ 600 = 18%9~ 180 2w = 230 6.8, We will use equation (6.1). For (2), 0 aero «f omar 6.3212 For (y), ‘The ratio is $322 = [MEAT] (C) 6.19. This is de Moivre’s law with w = 100. We use equation (6.7). ee =p © Gee ees a0, “Theres uniform dstbuton between 4800 and 5000, s0 we use the apezoidal method to caleue 20 vector funtion nth ang. Po = | spo 30 the ae of the teapens fom 00 S00 is 20()(2) = 1834. Te suv fin below 4800 Land the aren fe reine 0 0 4800 is “aoo. The wl amp temporary lie expectancy to $000 1s 4800+ 1835 = 4983}. The numberof Kowal: hous used is 50(0.015)(49834) = [37375 Can, since bpp = 45, c= 20+2(4) = 10, and frre ittime fs ifm over (0,90), Then the varanee is he Jength ofthe interval squared over 12, or 3 = [675] @) 622, Variance is #2 = 192,40 ~ 50 = 48,0 = (55) (A) e 623, Future lifetime is uniformly distributed on [0, 80), so the variance is "y ied moments of future lifetime for (30). ‘The first moment is 50 MLC Sly Mansion apeh @a00) ASM 2 _ __6. SURVIVAL DISTRIBUTIONS: MOMENTS 50 nee a f° @~ nae and the second moment, using formula (6.6) on page 76 is 2[(ra0 a 107] =2 [sina oe [° 10 -Nae a 2 100) 6 3 (s00 2000 = 100 ° (10 410) = 00-222 ap 08250 e 2500 1000 “ete ‘his eset equal to 35755 and the quadratic i solved, ‘An altemative way to gt the same quadratic which may be easier isto us the condiinal variance for tua (2.3) on page 24. We condition on surviving 10 years, Given that (30) survives I0 youn he aia (20) survives inthe next 10 years i extlly 10 years, 80 the oxpeced value ofthe amoustof tine eee next 10 years is 10 and the variance is 0. If (30) does not survive 10 years, death time uniformly distributed on te ate bs vale ofa uniform dsttion i the midrange or 5, athe aunt te sea aca sivided by 12,0" HF = 2. The probability of dying within 10 years. There + Tho expected vac of the 2 variances is (1) ‘The variance of the expected values (using the Bernoulli shortcut, So the variance of 10-year future tifetime For (30) is, Eto) section 2.4 om page 23) is, 230 2500 | 250 aa 8 Ogee Gy = 3575: 2500 20 108 57520 Either way yields this quadrati, Using the quadraie formula for 3, we get the solutions 1-1 35.755 6” 2(2500) j= 0011765,0.121569 = 85.00126, 8,22579 Since wo ate given that > 40, the answer is 30+ 85.001726 = [RSTOOTE Hot that i were less than 40, then the vance in number of years lived between 30 and 40 is variance in complete future lifetime. The variance of a uniform distribution on (0,01 is §. Since future lifetime is uniformly Aisributed, the variance is the maximum number of future years squared over'12. So we'd Ieee 2 2 5755 30) MLC Sty Mane tthetidon Copyright e200 ASME EXERCISE SOLUTIONS FOR LESSON 6_ HE 93 and w would then be 36.5503. 6.25, This came from a written answer question For (i), because of the lack of memory, we can evaluate it as oqo: = 60S [030634 is 1 = 9(30) = 1 - e" = [0.776870 Giiyis te reciprocal ofthe force of mortality, or (20) For (jv), varianee isthe square of the mean Tor an exponential, or 400}, 6.26, ‘The variance for de Moivre’s law isthe square of time to w divided by 12. (w=15P 2 (w= 15)? = 8100 w= 105 ‘The complete expectation of life is half the maximum remaining life (w ~ 25 = 80), or[40} (B) 6.27. is de Moivre’s law with w = 28 = 50. Then (0 ~ 10)" Var(7(10)) = PS © 6.28, This is variance of curtate lifetime, We'll use formula (6.9). expected value is eep3) = Bor * 2900+ 361 ta lt ho ta _ 118,432 + 6,823,367 + 6,616,155 je eee 20,6 = 2.840737 ‘The second moment is £f(4067) 9 3) |= por + 3200+ 5390 = lg + Bigg + Sho der FF 7,201,635 > = 8.410493 ‘The variance is 8.410493 ~ 2.840737 = [0.340706 6.29. This could be done from the definition, and such a solution is provided in the official solutions. But we will ‘use formulas (6.9) and (6.10). The survival probabilities are p= 09 2 = 0.9)(08) = 0.72 Pc = 0.9)(0.8)(0.7) = 0.504 SOA MLC Sty Mapas eon opyreh Gz009 ASM, 94 6, SURVIVAL DISTRIBUTIONS: MOMENTS ‘The moments are ELK 43) = 0940.72 + 0.504 = 2,124 B[K A3¥] = 0.911) +0.728) + 0.5045) = 5.58 Var(K A 3) = 5.58 - 2.1247 = [7 (A) 6.30. ‘The survival probabilities are 2Pc = 0.105) = 0.35 (0:35)(0,3) = 0.105 4P,=0 ‘The moments are E[K()} = 0.7 +035 + 0.105 = 1.155 ELK(a)*] = 0.7 +.0.35(3) + 0.105(5) Var(K()) = 2.295 ~ 1.155" = [OBA 6.31, For constant force of mortality, the curtate life expectancy is Sctting es = 49 and solving for 14, we got a 0) 0.020203, Using primes for revised functions, adding 0.01, the revised pis 0.030203. Then ecto 45° 7 = [32261213] SOA MUC Study Manut—8hetton Copyright ©1009 ASM Lesson 7 Survival Distributions: Percentiles, Recursions, and Life Table Concepts Reading: Actuarial Mathematics 3.5 or Models for Quantifying Risk 2nd or 3rd edition) 3.4, 4.1-4.4 7.1 Percentiles A .100n percentile of survival time isthe ti ty that survival time is less than ye# such that there 1 =x. If x = 0, itis the / such that the distribution function 100% probat 1 In other words, it is ¢ such thet gy = 1, OF Pi F(t) = n, oF the survival function s(2) = I~. 'A special ease is the median remaining lifetime at age x, which we denote by (x), which is such that px = (9x = 0.50, If survival follows de Moivre's law, median remaining lifetime at age xis the midrange, half way from x t0 2, or #34. Ifsurvival has a constant force of morality 4, mneaning that it follows an exponential distribution, we must find # such that eM = 0,5, Solving this, we have that ¢ = 82, 1¢ doesn't matter what x is in this ease since the exponential distribution is memoryless. Here's an example thats itl harder: zante 7A. A person age 70s subject to the following force of morality: for <5 inl pn {ia 1s Caleulate median future lifetime for this person. e®™, Plugging fe sg- 0.2 t= 5, we get ¢°®5 = 0.606531, which is greater . We solve: Answer: We want ;ps = 0.5. Port <5, iP than 0.5, so the median is greater than 5. For > 5, .P 05 _0H-9) 295 05 -02t+ 1 = 10.5 =1n05405 _ aie If you are using a life table, then the 1007 percentile of future lifetime at age x isthe point where fx) is equal to (= a)lg. For example if J, = 1,000,000, then the 20th percentile is the ¢ such that iy, = 800,000. Usually there will be no ¢ satisfying this exactly, so you will only know the median is between two integral ages, To get an exact answer, it will be necessary (o interpolate between the ages. Interpolation is discussed in the next lesson, 59057 2 7.2 Recursive Formulas for Life Expectancy Por both complete and curtate future lifetime, we have formulas expressing them as the sums or integrals of proba- bilities of survival. We can break the formula for (x) up into a component for temporary life expectancy for a period of n yeats, plus a component involving the life expectancy of (x +n) times the probability of survival 1 years. A. special ease is when n SOA MLC Sey Manual eon 95 pen 200 ASH 96 7. SURVIVAL DISTRIBUTIONS: PERCENTILES, RECURSIONS, AND LIFE TABLE CONCEPTS Jooks like this: For complete life expectancy, the decompos be = bem + Pte al) ‘whereas for curtate life expectancy, the decomposition looks like this: som + nPxtsin (7.2) and when n= 1 = Pct Preset 73) “These formulas allow construc end ofthe table w for which ey xl, ‘These formulas also allow fast computation of life expectancy when you are given a piecewise constant force of morality, as the next example shows. n of a table of life expectancies a all ages. To construct such a table, start with the and then calculate ¢,. from ¢, repeatedly starting with x = « and ending with ExaMue 7B_A person age 70 is subject to the following force of mortality: foor <5 raion oe Caloulate zo for this person. ‘Answer: ‘This isthe same as exercise 6.1, but we will now do it without integrals. By the recursive formula, 0 ro + sere) Now, é75 is the life expectancy of someone with constant force of mortality 0.02, or exponential mortality, and we know that the life expectancy for exponential mortality is the reciprocal of the force, or ys = 1/0.02 = 50. Also, Now, consider a person age 70 subject to the constant force of mortality 0.01.. We will use primes for this person's mortality functions, By the recursive formula, Bio = gay + shia bas However, é = 8 = 1/001 = 100 and sphy = 6%, 50 45 = 100(L~ 8). And now the punch Tine: #5 = eqs. Why? Because for te first five Years, the forees Of morality for the parson in our example and the peison with constant fore 0.01 are the same, and 2 a funetion only ofthe free of mortality i the fist five years So we have br 24385) D ‘Another use of recursive computation for exam questions is calculating the effect on life expectancy of changing the mortality assumption for a year or for a period. You would start with the originale, then advance to the age +n beyond the change in mortality, then work back to e, using the revised mortality assumptions, ‘The formulas also work for temporary life expectancies on the left side of the equation, For example, equa tion (7.3) would become, 100(1 =e) + 506-95 = 4771 + 47.5615, Cem = Pat Pat sai Exaveue 7C For (x), standard curtate life expectancy is 72 years and standard qe = 0.01. Because (x) has better underwriting characteristics, qx = 0.005 for (x), but mortality for ages x + I!and higher jg standard alculate eurtate life expectancy for (2) SOA MLC Smdy Manual ton Copyigh @2009 ASM 7.3. CENTRAL DEATH RATE, Ly Cl ‘Answer: We calculate ey) by rearranging equation (7.3): ex = SPE Pa 72-099 agg Tee ‘Then for (x), we have 995 + 0,995(71.72727) = [7.36364] a 7.3 Central Death Rate, Lx, Tx, a(x) viduals: , (this is not the same as T(2)!) isthe total future lifetime of a cohort of le T= [baat s Using formula (6.1), the complete life expectancy can be expressed in terms of Tx: "Thus, T+ is the sum of Exy, for t going from 0 to co. Ly is less than or equal to fx because each of the [, individuals: lives at most 1 year in the next year, We can also define jy, the future lifetime of Js individuals over the next.» ‘years, by integrating from 0 to n instead of from 0 to 1 Note that if uniform distribution of deaths between integral ages is assumed, Ly = 0.5(lr + dasi), and more generally, if Iggy = le — Kt for 0 <1 1, since yg 88 foal death rte for n yeas: gms Would have about the same magnitude a ma, while yx Wold be about n times 9 a(x) isthe tation of the yea lived by those dying during the your bx= bint era Since the concepts inthis section lead nowhere in the current syllabus, I got the feeling tha exams are deem phasing them, In particular a3) snot mentioned in Models for Onanifying Rsk, one of the two syllabus options {you may use as a textbook for the exam, so it would be unfair to ask a question about it. xastut 7D ‘The foes of motaliyis constant; = 0.1, f = 1,000,000 Caleuate: 1. Ts 2. Lo 3. mg 4, (50) Avsiae: 1 T= fs toe tar = 00 000) Gi,379a7 2 ay = (1) | es. ty= ear 000 000 (ek dsp = lg ~lsx = 1,000,000e"5 — 1,000,000e"8* = 6737,9470 ~ 6096.7466 = 641.2004 mp = 41:20 9 : 9 = “6412.00 ~ Ttis easy to prove tha in general for a constant force of mortality, my = Hae 4 (50) x $412.00 = 6096.74665 41.2004 ‘The formulas in this lesson are summarized in table 7.1 SOM SILC Sy Sou Cepyigi- 92 A384 7.3, CENTRAL DEATH RATE, Ly. Ts» 2) 99 ‘Table 7.1: Formula summary by = bem + Pxesin tem = Eom + mPsExveramy MST Recursive formulas om = Comm + mPsCemtamy mM <1 een = Pat Patani Life table concepts <2 for uniform distribution of deaths 1% = hx | 050% 0.54 Central death rate and related concepts fae et de ae ie « | ‘SOA MLCSuy Mual—8h eon apa 200 AS 7. SURVIVAL DISTRIBUTIONS: PERCENTULES, RECURSIONS, AND LIFE TABLB CONCEPTS Exercises (eda teed rac Cut 7.1, Mortality follows Gompert’s lav, with x = Be*. You ate given thatthe 10th percentile of survival time is 40 and the 7OKh percentile i 80, on EB Woy ton tee) Determine ¢. wey eet vee 7.2. Mortality follows te Wong ee a6 sto}. ay ay" 120 x)" ‘oa BG) 7 sy f a) Osxs 120 MS. 90 61.5 Cateulate the median future lifetime for (30). 7A, (4rB6:14 You sre sven so) = by YP tg at Determine the median future lifetime of 0). 2 od yet B) y Ol OM} ® yh ‘74, (150-88727; 150-$90:7 is virtually identical] Which ofthe following are true? 1 pits ater for 12 Oandu 20,7 9h = (te 9 Pas et, He ugen & pies for 2 Oandu 2007 Pew thar ae 4 i HI, If s(x) follows de Moivre's law, the median future lifetime of (x) equals the mean future lifetime of (x).—" (A) Land If onty (B) Land IM only (©) Hand 1M only (®) Tend wt Hl The correct answer i na given by (A), (B), (C), or (D) « 78, (CAS4A-F93:20] (2 points) You are given a survival function s(x) = | ~ 0.0Lx for 0-< x < 100. Determine the median future lifetime of a life age 10. seo) eee (A) Less than 42, (B) Atleast 42, but less than 44 (GQ Atleast 44, but fess than 46 (D) Atleast 46, but less than 48 (8) Atleast 48 Mortality for Audra, age'25, follows de Moivre's law with w = 100, If she takes up hot air ballooning for the coming year, her assumed mortality will be adjusted so that for the coming year only, she will have a constant force of mortality of OL. cxyonenegt Calculate the decrease in the 11-year temporary complete life expectancy for Auda if she takes up hot air ballooning. (A) 0.10 (B) 035 «C) 0.60 080 ©) 1.00 Pray Cae MS Bream Cm payee ie Vet ae coos co eit) say SOAMMEC Sey Maat en erie continu on the nes page. Copy 0 aS 10x EXERCISES FOR LESSON 7 - Caz (3-£00:25) Given: @) Superscripts M and A identity two forces of mortality and the eurtate expectations of life ealeulated from them. a Rte ay voreen as hg BOS vp fea =) OSS ties Po = baste a hen = fas MEE n yes za Gh Ps ta Ge) = a= 100 Ge AE Can Che) BPS ent tues Adee Calculate ef, = (a) 92 @) 93 plas ©) 96 . Y as 78. You ae given hates; = 49 and pos tes Petr tn + Bs fy an out IPs is doubled for 35 < x < 36, what is the revised value of e357 719, (CAS4-82:33) Which of the following statements is true concerning the inequality éxe1 > ¢x? (A) ‘The inequality eannot bo true, ny Ce PEED (B) The inequality is true if and only if : ey Pe cers Me CHS en > PE ee a ex > & 4x (© The inequality is tue if and only if (D) The inequality is teve if and only if NG The inoquatiy iste if and only i 140. {CAS4A-197:20 (1 poind Fora ie age 50, the curate expectation of lf eg = 20. For that same life, you are alo given that pa = 0.97 yes 04 (1) €27) Determine es: (A) Less than 1875 (B) At least 18.75, but less than 19.00 (C) Atleast 19.00, but less than 19.25 (D) Atleast 19,25, but less than 19.50 (8) Atleast 19:50 SOA MLC Sty Mansa 8h eion -xeresesconine onthe net page Copa 2009 AKT JRVIVAL DISTRIBUTIONS: PERCENTILES, RECURSIONS, AND LIFE TABLE CONCEPTS TAL, (CAS4-S86:26] (2 points) Consider a subgroup of lives who have,been exposed to & certain disease, It is estimated that this subgroup will have a higher than normal rate of mortality for two years following exposure to {his disease. The mortality rate is 10% higher than normal ducing the frst year-and 5% higher during the second Year, After that the mortality rate returns to normal ) ge= 007 es aor eee teetind eataierit {i gies #040 ays oto Gi) gas = OL 5 Caleulote the reduction in curtae life expectancy, in yeas, for a person age (2) who has just been exposed to this disease, (A) Less than 0.050 (B) At teas 0.050, bat less than 0.075 o At least 0,075, but less than 0.100 (D) At least 0.100, but less than 0.125, (©) Atleasto.125 W) ea iceeeee Bao vor t wotae Crow 9 Exes 99 59 = Paw Uae és © be 1404 Gi) Boer Stat r fe Ban) 38.67 «iy Bos the ) w Bay = 1 yous Cee 46 026 Improvements in mortality at age $0 eause ,p3p to change to 1 ~ 0.0094 for 0. t $1. Calculate the revised value oféo, 2 : ho é 40? 7143, You ae sven eee i @ s@0)=09 Gwar ein gy 9 ve 88 860) = y fae The survival distribution function is Hinear between ages 20 and 60, bay = 60 bq =25 Determine y TA4-1S. Use the following information for questions 7.14 and 7.15: You ar given: ae : : 7 (10 ~ »} " wet sa) = = ogy x fh & oy sQy= SE osxst0 | 74, (150-61-94:47] Calculate the average number of years of future lifetime ofthe f survivors ofthe group at age 1 we. y (237 @) 243 2% 292 ph3.00 SOA MLCSty Marit eaton ‘Bteotesconrnae onthe nex pase. Copa 2009 AS EXERCISES FOR LESSON 7 i“ aoe ss ey ad-iS, (Repeated for comenience) Ose the following information for questions 7.14 and 7-15: "You are given: (0-3? Oar osesi0 ac tbe Tis stn 94] Catt ho avrgemombe of years lived between Land By fas of he HORNE, (a) 0461 @) 0473 (©) 0.484 @fo90 ® 0500 ial members at age zero and 7.16. {CAS4A-893:15] (2 points) You are given a survivorship group with 1000 the survival function is s(x) = 1 ~ of for 0 < x 5 125, where xis age. Calculate Tso, the total number of years lived beyond age 50 by the survivors (A) Less than 20,000 Ro = 100 Ant ® ‘At least 20,000, but less than 30,000 eee (c) ‘At Teast 30,000, but less than 40,000 tus = (*8 jas tte © 8 ae ot te sersb) = 1y90 1 EE = 2250 (D) Atleast 40,000, bat less than 50,000 je (B) Atleast 50,000 7.47. (CAS4-888:17) (2 points) You are given: {The total number of years lived beyond age 38 is Pos = 95,000. i) The toal expected number of years ved between ages 38 und 39 8 Los = 2,475. ii) ‘The central-death-rate at age 38 is ms = 0.021. (iv) The number of survivors to age 38 is fs = 2,500. Calculate és. ' Cy = Lay ag owe 6 OA (A) Less than 37.0 fe 7809 (B) Atleast 37.0, but less than 37.2 day = Mo be (©) Atleast 37.2, but less than 37.4 ys (D) Atleast 37.4, but less than 37.6 ae GA Atleast 376 bens UT cond aye 7.48, (150-F96:13] You are given: aeeaantea a (pe O8Y, 120 Gi) Ine = 4 Calculate Try. = (° wows - (a) 45 (B) 72 © 287 pH 359 &) 448 carte EL «Oe sux) ty SULA a fo = 1081 haw fo stare 1) Wa) cand Lass oe oar 2° on oa bor bo shel = Oe SOA MLC Sty Mansa do rercisescontinne on the net page Cepyng 200 ASKE 101 ___7. SURVIVAL DISTRIBUTIONS: PERCENTILES, R CURSIONS, AND LIFE TABLE CONCEPTS 7.19. [CAS4A-S97:17) (2 points) You are, Siven 2 cohorts of 10,000 tives each, all age 0. One cohort. ‘experiences Daye 10g MMA Hs = OOS, ‘The ote cohort experiences a force of moray given by = er fe 0sy< 100, Determine the difference in the total number of years lived beyond age 80 between the cohorts (A) Less than 15,000 cc ®B) Atleast 15,000, but less than 16,000 4, 1.00 @ ba AtTeast 16,000, but less than 17,000 ou ee ) At east 17,000, but less than 18,000 teosko 10089 209-64 ©) Atieast 18,000 7.20. You are given that éspy, = 0.9 and ey © Rsv Ago= 02 = dsp Calculate mso.- bso. 04 0% 4, \. oa ie Q so 721. You are given th the fection of age 86 ved by those who dee age 86, (86), i 0.7, and mug = 0.05. Calculate Les/Ies. oy Oe e Lae fer Yee Roo sdee 4 tee due dee use 722 25] You are give ead ucneee waa MEG deo ov (26 20% (fy = 1055 te des (i470) = 0.430 the average numberof yeas of lie ater age 70 for those who die between ages 70 and 71 mp = 0.100, the central death rate forage 70, sao arte = Un0.s day, Lane dag de Cateulate fn. fre de so bere tee an e tno (A) 940 955 (©) 967 ©) 973 © 976 7.23, (150-187:21) Assume mortality follows de Moivre's lay for 0 < x < w. Which of the following expressions equals ,? de Eien ae : Gy ike ist "S03 Ul ue de i wosde * gsde 7 te ‘ pe Vay sf a Me We te OSnswns- i nie ss oo Te ya QY Land It only (B) Land Mt only (C) Wand MH only ©) 1, Wand mt P) The correct answer is not given by (A), (B), (C), or (D). 7.24, [180-S89:4] Assume mortality follows cle Moivre’s law for 0 < x < wv, ‘The median future lifetime of (x) s denoted by n(x). Which of the following re equal touefor I \ wel es ws St aco ? 4 Nas us iL 14+ 0.5m, (A) Land Il only (B) Land 1 only (©) Mond I omy (PIT Hand nt (B) The correct answer is not given by (A), (B), (C), or (D). ‘SOA BC Stay Mon —Shedon Exercises conte on she nex pee opyat S008 ABN BXERCISES FOR LESSON 7 Who aire ere 7.25, (150-F96:1] You are given p(x) OS x< 80. 96. (Pre dlrs = AEC oy. 600) Cleutate the median future lifetime of (20). R 5.28 B) 6.08 (©) 8.52 (D) 26.08 ) 30.00 ‘216, {80A3-R03:18} A population has 30% who are smokers witha constant fore of rality 0.2 and 10% who ave non-smokers witha constant force of mortality 0.1. oxerere Ore? 078 Calculate the 75 percentile of the distribution of the future Ii dual selected at random from this population. (a) 107 @) 110 our 6 @ us 1.27, (CAS4A-S95:14) (2 points) You are given the survival function s(x Determine the central death rate, mz, at age 30. (A) Less than 0.045 f, Atleast 0,045, but Jess than 0.047 (6) Atleast 0,047, but less than 0.049 (D) Atleast 0.049, but Jess than 0.054 (B) Atleast 0.051 728-31. "Use the following information for questions 7.28 throug 7.31: time of an i pines Ma — exponencral You are given: — (Tis the random variable for the future lifetime of (2). Gi) ‘The pal. of T is fr@ = 2%, 120 7.28, {150-F96:24)} (1 point) Calewlate Bs. = 3. 4 os 20 © 100 ©) 200 © 729. {150-796:25} (I point) Caleulate Var(T). fy @o2s @ 050 © 1.00 ©) 2.00 @ 400 7.30, (150-F96:26) (1 point) Calculate (2), the median future lifetime of (2). im ¢ ina nd A> a> « a © i's ®t sya. [150-F96:27) (1 point) Caleulate-m,, the central-death-rate at age % = Als 2 : a) B) © 27 1 P52 7.32, (15097233) You are gi wie Dee HQ) = Os x21 6 Heat joo-e Cateulate the average number of years lived between ages 40 and 60 by those curently aBe 30, (A) 26° @®) 48 (64, 56 ©) 65 93 0m MLC Sy MeualB0 0 Bereties continu on th net page Coppa 22009 AS 106__7. SURVIVAL DISTRIBUTIONS: PERCENTILES, RECURSIONS, AND LIFE TABLE CONCEPTS Additional released exam questio: + CAS3-FO6:1 1, CAS3L-FO8:13 Solutions 7.1. Under Gomperta's law, ovo( =) 03 dividing the first into the second 1.427173 ViraziTa—T 060365 866025. To get the median, we'd like the age x where s(x) is half of this, me =)" 0.866025 a5" 2 = 0.433013 120~x on T29 = 04330137 = 0.1875 x=975 97-518 the median age to which (30) survives. Therefore, the median Futute lifetime for 30 8 975 30 = [67 72 We wan x such that sa) = (() eee Tex” 4H xe142y 1s the median age at death for (), Median future lifetime for (y) is x —y This survival function is a Parcto, You can see why a Pareto distributfen human life: the older y is, the longer his median future lifetime! @ ‘not a plausible distribution for 30 MLC Sty Manust—t0 eon Copia 2003 Ash 107 EXERCISE SOLUTIONS FOR LESSON 7 __ 7A, 1. Wecan compare the complements, uP or equal to the latter, It follows the complements and ype The former is the latter times Py and is therefore less then have the reverse elationship and Tis tue. Th ged = sPrusvts 804 Px S 1, $0 THis true, JIL, Both mean and median are equal to the(midrange, so Ils true, ©) 7.5, We recognize this survival function as de Moivre with «= 100, so the median is the midrange, 12912 = [45] © temporary complete life expectancy can be computed by the trapeasidal method or the 10d, as we discussed on page 77. Using the trapezoidal method ne same) re enema cone esti 2s tain =12(14 ssn 7.6, ‘The original 11-year conditional expectation meth 0.193333 ‘We recursively develop the modified 2,,rp using equation (7.1), We'll usc a prime to denote the modified Fanetons- San hs days fetta 11-2) e10(i- eM) e2@3288%) 1.951626 + 8.436998 = 9.388623 ban + Pasta) Es ‘The difference in mortality is 10.193333 ~ 9.388623 = [0:8647} @) 177 Adding something to multiples the survival probability by the exponential ofthe integral ofthe something. m= atow(- [ osc - ni) ford<1s1 “The integral of the linear Funetion 0.1(1 ~ 1) is 0.05, 50 ih = pagce ey ‘We use the recursive formula to get ne and then ey, oe ** = [9.8523] ©) 78. Doubling j squares pss. Using the recursive formula, and primes forrevised values a8.755, “SOR MLE Soy Manusl—8i Cops nlp ASM 108 2. SURVIVAL DISTRIBUTIONS: PERCENTILES, RECURSIONS, AND LIFE TABLE. CONCEPTS 79. By equation (7.3) c= Pet Dates So we want F001 > Pet Pxeor, > Dem geese Ps en @& 4 ) rb seh ~ Giles) 7.40. Using formula (7.3), 650 P50 + Paves 20 =097 + 0.97es, 19.61 es = 20 r) 097 «) AL Note tar moray rte refers tog, nt pe Would be cele force of metal, Using pines ton ‘modified function forthe subgroup of les exposed to the disease, ).07(1.1) = 0.077, PB day = 0.104.058) 0.105 Pa ‘We use formula (6,9) for the 3-year temporary eurtat life expectancy, 1.923 895 ‘and the recursive formula for e,. cil = Pat 2Pe + Pe .93 + (0.93}(0.9) + (0.93)(0.90.89) tet Pr eeer 51193 + (0.93)(0.9)(0.89)(5) = 6.23658 ‘923 + (0.923)(0.895) + (0.923)(0.895)(0.89) = 2.48430 €{ = 2.48430 + (0,923)(0,895)(0.89)(5) = 6.16038 6.23658 ~ 6.16038 = [007630] (cy 7.12, First we use the recursive formula to ealeulate és. 2.51193 #0 = Farm + Pats 35-= 94 0.8525) 26 ag = 30-588235 Then we use the recursive formula to calculate 85, psp = 0.99, 50, We use the trapezoidal rule to get pn) = 0.995, ‘Then bn ‘and since deaths are uniformly distributed over age 850 = aso.n + Pts 30,588235 = 0,995 + 0.9925, &s1 = 29.892157 ‘We use primes for the revised functions. pig = 0.991 and using the trapezoidal rule, SOA MLC Sidy Manel —Siheddon Copi@aonn ant 109 EXERCISE SOLUTIONS FOR LESSON 7 _ 743. We can use either logical approach or an algebraic/geometic approach. “rhe lopleal approach iso spit the universe It to groups the one that survive @ age 6D andthe ones Whe ons the eros whe survive 1 age 60 have an expected lifetime of 40 plus Ee, oF 6S, The Ones who don’t Hane a8 se sad ites of 20, since srvval i nifonm beeen ages 20 and GO. Expected lifetime at 20 is the weighed average of the expected lifetime of these two groups: 8,9 = snpa0(65) + (I ~ 40 20)20) 60 =20+45 opr ee pn = B75 But agp = s60)/s(20), 50 it fllows that s(60) = 8/9)s20) Pat ebrnegeometic aproach tarts with the recursive forma equation (7.1), to relate the given 2's B50 = Brain + s0P a0 20 6 toa +(2ehe0 (0 to 40, and ,pao is linear on (0,40) since s(x) is linear on (20,60), So the '20) and «opao = s(60)/s(20) = y/0.9 (at 1 = 40 or apa is the integral of spa from intogral i the ares of a trapezoid with bases I (ats = 0 oF 0) and height 40. So we have con asxany(1 +24) +23 ‘Multiply through by 0: 54 = 20(0.9 +)) + 25y = 18 + 45y Sy = 36 y= [a8] ‘44, We sw that forte generalized de Move, complete life expectancy for (x) 1 G5. Here, the exponent iB) © ty then dy = s1) ~ (2) = 88! = 0.17 and la = 3 = 0.64, We calouate is2a AS. Ieweset lo wis 10, 80 = 0.723333, So a(x) = 2mm = [OGH. CD) “16, Since survival follows do Moivre’s Taw, average foto ieime fr (50) is B52 = 37.5. The survivorship group at age 50 (ln) is 1000( 15532) = 600. So Typ = 600037.5) TAT. We need Tay and by. Toy = Tox ~ Lae # 95,000 ~ 2,475 = 92,525. We caloulate dh oe Tx dye iitaad 2,475 dyg = 51.975 SOA MLC Sly Manual ton epi 2009 ASN 110 ___7. SURVIVAL DISTRIBUTIONS: PERCENTILES, RECURS! So by = Ine ~ dag = 2,448,025, Finally, Tag 92,525 _ tn T= sqiegps “SUSE Ths TAB. We need tay for > 1. te = 2 = 10, 80 Dey 10, = 100.8"). Then byt =10 f cosyar ar = 358514] my 7.19. Foca constant fore of morality, lif expectancy is = 20, ‘Then Too olen = 20Iwy = 20(10,0006"°°) «9668.13, ‘The second cohort follows de Moivre’s law with w = 100, s0lifé expectancy is 10 for (80). Then Tq = 10lsy = 10(10,000(0.2) 20,000. ‘The difference is 20,000 - 3663.13 120, ess po, 20 959 = $8 7.21, We are given ois ‘The reciprocal of 1.015, [6:985223], is the answer to the problem, SO MLCSr4y Nana eon Coppin oro ash EXERCISE SOLUTIONS FOR LESSON 7 mw ).100, s0 Zap = 10d, Substituting for Zin the Formula for a(70), 722. Gil 1450 =[255]@) my # TF 05m, ~ 740.5% Hens T+ 05d, since under uniform distribution, Ly = Ty ~ 0.5, | ea Since qx = qh itis equal to py. Since js = dy for de Moivre's law, This equal to js. &x is 938, 80 its reciprocal is twice te. (AS 7.24, juz = ghz. The first expression is get WMw-x4) Pet (== wm x making Tequal 11 The median isthe midrange, &!, 80 Is g;, making it equal to. We already sw in the lst problem tha IIs equal tot.) 7125. ‘he conditional survival fnetion is = exp (2 Va0=n)), = exp(2( VOO=1 ~ V6O)) ‘We set this equal to 0.5 to get the medi 2( V60= 7 - V60) = In0.5 VO=7 = YOO+ SOA MLC Sey Manassa edloo Ceyrantgnans ashe 2 2. SURVIVAL DISTRIBUTIONS: PERCENTILES, RECURSIONS, AND LIFE TABLE CONCEPTS 7.26. The population has a mixture distribution; the probability of lifetime greater than x is 0.3e-°®* +. 0.7e-®, ‘We want to set this equal to 0.25, so that there will be a 25% chance of living longer than x andl therefore a 75% chance of living less. Let y = e°®, Then we have 0.2y? +0.7y~025=0 0.74 VOATTOS 06 eM 2031470 x= 10103147 131470 [iiseis8] — @) 7.21. For exponential survival, m, = j,, and we see in the exponent that pi, 7.28, In the copy of the exam I have, answer choice (B) was blank! TT is exponential with constant force 2, so the mean is + =[08} (A) 7.29, T’is exponential with constant force 2, so the variance is & = [6:28} (A) 30,__‘The distribution fmetion is 1 ~ e°% ‘The median ist such that ¢"™ = 0.5, or ~21 ~ In0.5 = ~In2, oF @ingy/2| (C) 731, For constant force of mortality, mg = fiz @) (&) Ifyou didn’t know this, you'd have to calculate dy SG teabetteuslt “"o Blewedt 732, ‘This isa generalized deMoivre, and ,pyo is a f a wen exp | ToT ‘Then life expectancy between 40 and 60 is obtained by integrating ,pso from 40 to 60. This generalizes equation 6.5 abit 1) = en i701) ~ 1070) (eH B[ (70) - 10) 020] = £ aad “La 605-405 way ~ E24] CO SOA MC Say Nasst-—8th eon Copyrigit2009 ASM Lesson & Survival Distributions: Fractional Ages ending: Actuarial Mathematics 3.6 ox Models for Quantifying Risk (2nd or 3rd edition) 4.5 Lite tbles, suchas the Istative Lie Table, st morality rates (gy or ives (1) for integral ages ons OA eee aa denin ves at rational age (Uke sf an integer) oF mortality rates for ration of year, We need some way (o interpolate between ages. Ieee eal teed interpolation methods: linea, geometric and harmonic, We wil consis ee ne ceerplation sos eed tas mniform dsriuton of dears between ie) cee ths of hem et pertain is usually refered t as constant force of mortality between inal ae, At raya be ealled exponential interpolation, Harmonie inepolation is eee 98 Iyperbatic interpolation of Balducci's hypothesis. 8.1 Uniform Distribution of Deaths sty easiest interpolation method i Hear intexplation, or uniform distention of deat This means 1 We Tae ea elias wage xt a,O <<, lea welghed average ofthe number a ives at age x andthe pares of lives atage x L ~sds a) = s)he + Sheet = bese “the graph of lee isa staight line between s = O ands = 1 with slope hors sPe 550 fess ets 1 (sa = 5 82) “That is one of the most important formulas, 50 let's state it agal More generally, for 0s s-+45 1, 83) Teaid, T= tg. vere the last equation was obtained by dividing numerator and denominator by f. The important post © pick up setna white og the proportion of the years times qr sys 18 10! su but ib im fact hight than sqy. ‘The number vr tives dying in any amount of time is constant, and ince there are fever and fewer Hives the year progresses, the aaa seh is in fact increasing over the yet. The numerator of sais the proportion ofthe eat ‘being measured meio ihe dent rate, but then this must be divided by 1 msinus the proportion ofthe year hat elapsed before the start of measurement, or most problems involving death rates it will suffice if you remember that Ly linearly interpolated. Try ‘working out the following example before looking at the answer. 0 MLC Sty Mant een 413 enya 82000 ASH 14 __ 8. SURVIVAL DISTRIBUTIONS: FRACTIONAL AGES Examete 8A You are given: ge = 01 Uniform distribution of deaths between integral ‘ages is assumed, Calculate iy2geers ‘Assorens Let y= 1. Then let = Z(l = qx) = 09 and dy = 0.1 Lineal interpolating, site = b= bd, = L~ HCQ1) = 0.975 dest = be dy = 1 = (0.1) = 0.925 Jeri lua _ 0.975 ~ 0.925 TiO ‘You could also use equation (8.3) to work this example, Uadena = ExaMete 8B For two lives age (2) with ind Uniformly distributed between integral ages, Caleulate the probability that both lives will survive 2.25 yeas, lependent future lifetimes, yqy = O.1(k-+ 1) for k = 0, 1, 2. Deaths are ‘Answe: Since the two lives are independent, the probability of both survivin 18 2.25 years is the square of 22s), {he probability of one surviving 2.25 years If we let lz = 1 and use dys, leaves We get 46 = 0.101) = 0.1 feet = 1 ade = 1-0. =09 Ide = 0.12) = 02 faa = 0.9 dest =0.9 ~0.2 29x = 0.18) = 03 bags = 0.7 ~dyyp = 07-03 trombar 1075 64 ‘Then linearly interpotating between fx,2 and Iya, We get te4225 = 0.7-0,25(0,3) = 0.625, and aaspx = dep = 0, Squaring, the answer is 0.625? = [03390635 eecegee a ‘The probability density function of T(3), sPsstzen isthe constant gy, the M0d+e derivative of the conditional cumulative distribution function .9, = sq, with 1 Pee Fespect tos. That is another important formula, since the density Is nesled tn compute expected values, s0 It’s repeat it as 2 84) 0.45 Qe Sts 65% 1 ‘The foreo of morality increases over the yea, as illustrated in the graph tothe right for gigo = 04, {ide the uniform disibution of deaths hypothesis, the Faction of te year ved by thove dying during the year isaa(x) = }. Therefore, Ly = L(t, + beat) and 2, a Oe Tee” T= 05q, bal SOAMLC Staly am—tieon Copyiahe 2000 Reh 6.1, UNIFORM DISTRIBUTION OF DEATHS ae 15 Complete Expectation of Life Under UDD fine complet fre lifetime random vaible 7 is wren ®t ‘45, where Kis the cutate fore lifetime and 1 com te a yea ved, he Kand Sa menses ey vay no! eve if uniform dstsibation S athe on ame, ie isnt on (0,2, EIS) = band YentS) = ye flows from B{S) = 3 hat geared @n More emmeon on exams afe questions asking youro ovens temporary complete expectancy of life under Lupin ou cam las evalte he temporary complete pee 5% ether or not UDD is assumed, by integrating vp, a indicted by formula (5) om page 16, Te einen between integtl ages 80 evaluating We Peal i partcarly asy, a8 We shal sein the next example. xan 8C You are given @ qr= O01 GP Bente runny disoued Between nea! 955 Caleutaté é,a} rans We cass wo wyato save san algerie meng anda goes method Me des th st cat for @ wis sion the midpoint, deaths occut i een nega gs, that meas iat ese who le win se contained within a yea survive all the period on the average, Pe apexample, tose who die within 4 survive a5 STS of 02, Those who survive Os survive an averaRe of oa ot course The temporary life expectancy is (he ‘yeighted average ofthese to BTOUPS, °F o4de(0.2) + 04Px(04) ade = 0.A)(0.1) 0.04 ape = 1 0.04 96 dan = 00402) + 0.60.0 = [392] ‘An equivalent geometric: method is to draw the Ps function from 010 0.4. ‘The integral of yps is the area under ‘the fine, which is the area of a trapes ‘the average of the heights times the width. The followir is the graph (not drawn to scale): Ds (04,096) .0,09) o aa raped Ais bean were ined in Hs aenis 38 + 0908 4) = (0383) o Quiz 8-1 Once again, you ate give @ qe O41 Gj) Deaths are uniformly distributed between integral ages. Calculate 8,040 ‘Gatton iva the end of he sTaions tothe cnerles Son ME Sty Mansa —€i 8 Seri 6200) ASM 16 &. SURVIVAL DISTRIBUTIONS: FRACTIONAL. AGES {at's now work out an example where we cross an integral boiundary, ExaMeue8D You ate given: @ a “i Answene into those 0.25 on th the mi Ge= 04 uss = 02 005m. Let's start with the algebraic method, who die before x-+ 1, those who average since the period to x + 0f 0.5 and 1 is 0.75, so they live 0, Now let's calculate the probabilities, ‘These probabilities could a itt Ret lesa and Either way, we Peas ac unitrmlydsributed betveen integral ages Calculate ince the mortality rate changes at x + 1, afterwards, and those 1 is length 0.5. Those wh = 0500.1) Sotees = TO. 5 ospains= 1-5 = 0 Osm34n05 = (lesa) Iso be calculated desis. Then oss Ye now ready to calculate 8, Seosn = 510.25) + 910.75) +81 2028) + 900.95) + 81 0 Farr BM Le ago. buat ea dea = 0.5(90 + 100) = 95 feats = 0.5(72 +90) = 81 95 For the geometric method we dravy the following graph: SOA NLC dy Manat ion Copy 2009 ast 8.2. CONSTANT FORCE OF MORTALITY He 47 Pees 1 (05.8) (1.0.8) ' 1.0 0 05 x15 405 xtl ste eights tx and x 1.5 ar as we computed ove, Then we cOmPuE Sah i 1 (1+ B05) al The aro Bis 4 (8 + BJO) fly. Adting them up, we eet SGT = [EYP ‘area separately. The area of A ‘Gale D The probability tat a batery Fils bythe end of the th Jronth is given inthe following teble: T Probability of battery failure by the cendof month k k{ ew 1 0.05 2 020 3 0.60 Between integral months, ie of fallre forth battery is uniformly distributed. acre tcapeced amount of time the battery survives within 225 ‘months. cant am or xan nfogers in ers of én, note tha hose who ane Ta ‘oniibate the same to aed conti en average off more to xm ince they dion he NERS the middle ofthe year “Thus the difference is nds ben = enn 105095 68) sm distribution of deaths between integral ages “ixanrur SE Calculate 8p i qs = 001 for 30 05, 90. *nosPeat SOA MLC Sty Mami eon Consent 20 SK 8.3. HYPERBOLIC ASSUMPTION _ 9 ‘0 the second integral is otf asp = 098 [owas -(0") os o.an705e-+.0.448897 = [0.935896 o ‘ANthough constant force of morality 15 not sed as often as UD, itcan be usefl fo simplifying formulas under aaergpeircumstances. Calelating the expected present valve of a insurance where the death benefit within a year eats an exponential patie is ean PAPPEn WEN at Yjeat benefits the discounted present ate ‘of something) say be easier with constant Force of ‘mortality than with UDD. (0.948683)(0.473116) = 0.448837 ‘The answer is 2,05 8.3 Hyperbolic Assumption Under thie hyperbolic assumption, on bales bes = Ta se ere the lst equality is obtained by dividing merator and denoiinator byl Dividing bah sides by I, wehave 8.12) 1d) More generally, @.14) soot = T= (Ne Notice thatthe denominator Increases #61 Inercses £9 that quar is a decreasing faction of #86 plausible, sri best derived a8 $19 Ps xt Not very Ps wea a wa) fon see decreases over the year. 1000 and 400 = ‘and we see that the force of mort ge 8.2 shows gos 400 pies FOF fen is assumed. Catcatxing mis messy. First we ealelate Zr 45 if hyperbolic assumption for fractional aB°S by equation (8.12) S508 MLC Stay Mamst—tiheon Repyise 200 ASHE 120 foes Hos 1000 1 a 550 Jos %———_| 5 tors © 00, EXAMeue SHC You are. ® a =0.1 Gi) Mortality fotiows the C/2)q, (0.5\(0,1) 0.05 r ee the answer isthe same as with uniform Examee 81 You are given: Oqe G) Qui = 02 distribution of deaths, Caleulate 8.9 Ansoren: We calculate fp, 93 dt. We s “10 1 for ag *PHCAhis Up into 2 integrals, one fromm 0 to 0:5 for age x and one from 0.5 10+ 1. By equation (8,14), oa 1-0 ~ne, meee 00M mee G+ Ole $04 MLCStady Mansa 80 eon opyigh e008 ash 18.3, HYPERBOLIC ASSUMPTION ‘The first integral is s ova) past? [al eee i (Hacasnon os = (mle (1-5 +0} oj (0.95) = (e\- Ings) = 0487286 Under the hypeoli sumption asauves = Oda 8 asPna4 this example is | ~ 0.5041) = 0.95. Fort > 05+ spss = 05Pas05 -05Pee1 = 095 1-09Pe41 so the second integral is (changing the variable, reducing by 05) Pandt os os [es =o095 { i J, Peidi=09 J, Tm 0 Diet = (C2222) vo, 5 _(0.95x0-8)1n0.9 = In08) = 02 = 0.447576 o “he answer is 0.487286 + 0.447576 Cone convenient thing about the hyperbolic hyhesi is that yoyfee = (1 ~ Se This can be convenient If the “exaneating age at ih a popu i observed vanes within an age. 101. Their birthdays xm 83 A population of 60-year olds with ife insurance policies has mortality rate geo = xe uniformly distibuted throughout the year: irons of this population surrender their eonrats on 78n- LL They are assumed to have randomly dis- tsibuted birthdays a fr the 0 ives a8 ofan 1 the cxpected mabe 9 oats before their next birthday if «Deaths between integral age tow the hypeboie esomPto +2, Deaths between integral ages are uniformly distributed. ,o1C1 —), where 1 ~ ss the amount of answane 1 The pected nomber of deat or ach 1 =e fof birthdays (density 1) from 0 t0 1, re tote next bichday. We integrate this over he wniform istribution and multiply by 50: A so {oon ~as = [05 2, Tho expected numberof deaths for each life is ooia- 8) waders 0A MC Siedy Mani —th eon Copynen 209 ASH ae at —§. SURVIVAL DISTRIBUTION: : FRACTIONAL AGES Leeil P+ 809) ste] (0-094) P| (1-0 ~)9,) sae f(t=(0 ~@+0)n) 6/00) Pete [1 0- any “Us py) tet ‘multiplicative constant O01. We then integrate the expression as follows: les 99. T00is =! - TF 55 = "ds Jf Tor = 100-99 ("ae = 10+ 90010 ~ 0010 = 100-4 99001n0.99 = o.so167s Mantipiying this result by 0.01 and by 50 lives, we get 0.5(0.501675) = (asso), 8.4 Summary ‘The formulas ave summarized in table 81, Figure 8.3 compares fe and jays for the 3 assumptions, using x and gy = 0.45, ‘What do you need to memorize ‘0 answer all probability questions formula, for the exam? For UDD, you know the tineer formula fr f,you willbe able You will be able to redeti ve the formula for 4s, you may memorize the Hee Ihe eeeased exams, but Ics easy. If you know that “% you have the whole story si t/Pexbolic assumption isthe hardest one to te or ge bt is unlikely they will sk anything buyond a may aan’ Weston oni. 1 you memorize ihe fornele ne ‘dren YOU shouldbe able to get by. To play sale you ‘may also memorize the formula foryer SOA Sty No Cena 20009 ash Being EXERCISES FOR LESSON 8 23 hoes ‘Moos 1000 5. 1 LEG Uniform distrib tion of deaths Constant force of mortality Hyperbolic assumption her Oe Figure 8.3: Comparison of the three assumptions for distribution of deaths between integral ages ‘You should also understand the picture, figure 8,3. Namely, UDD kills off tives the slowest, and hyperbolic the fastest, UDD has an increasing force of mortality, and hyperbolic a decreasing force. ‘The constant force of mortality assumption has a constant force of course, and isin the middle of the other two, Exercises 8.1, {CAS4S85:16) (1 point) Deaths are uniformly distibuted between integral ages. ‘Which ofthe fellowing represents ps hipaa? ys (A) sas 8) ayaa © inp ©) 29 Phiars 8.2, {Based on 150-888 5} You are given: tea 1-99-43 4 © oasqeons = 3 Gi) Mortality is uniformly distributed within age x. Caoutate g, 83-4, Use the following information for questions 8.3 and 8.4: ‘You are given: Deaths are uniformly dstibuted between integral ages. Gi) a= 0.0. ad i) gays = OS. vont B3. Calculate yageesye 0 vata Baca #1 na Prvtty tm Paty 8A. Calculate ognsanos © OoGu4 6a ht04 dere 0 0G Pang 0.2 PO SOA MUC Sey Maus!—8 eon ‘Beooses continue on the next page= Copyin 2000 ASN 1a ___ 8 SURVIVAL DISTRIBUTIONS: FRACTIONAL AGES 8.5, You are given: (i) Deaths are uniformly distributed between integral ages, (i) Mortality follows the Tihusicative Life Table, Calculate the median future lifetime for (45.5). 8.6, Deaths are uniformly distributed between integral ages. frsnrs = 0.04, = Ax Caleutatem,, x 1048 mY vost (at Ao are give @ Deaths are uniformly distributed between integral ages, [= | as | 52 [0.10 53 | OL 54 | 0.13, Cateulate smo. 8.8. [160-F90:5} You are given: a fo 8050) (A survival distribution is defined by col ty sce) P = 1000(1-(2.)), osx we OR t= 100(1 iia) }.0 << 100. ‘Hz denotes the actual force of mortality for the survival distribution, })_ nf denotes the approximation of te foree of mortality based on the uniform distribution of deaths assump- tion for i,, 50 < x < 51. Vey Cateutte 3005 ~ gay tea ee 8, -0.00016 (B) -0.00007 @o ©) 0.00007 (B) 0.00016 “8.9. A survival distribution is defined by i) s(k) = 1/(1 +0.014)! for ka non-negative integer. foo 5 G9) Gi) Deaths are uniformly distributed between integral ages, seus Caleulate o4g2n2. ate SOA MLC Stay Manes ion xerctes continue onthe next page Copycht ©2009 ASM EXERCISES FOR LESSON 8 i as 8.10, [150-$89:15] You are given: ated over each year of age. (Deaths are uniformly distri eae ae 35100 36 99 796 38 92 3987 ‘Which ofthe fllowing ae true? ames. ob F yau=0081 ee she EAM OE Tinie my = 0.043 Yr 1-05 91) ey tn HW omaes=0.021 0% Ye JL 405 Me) B) Lana only (6) Vand Henly (©) Mand only (D) Lttana {Th contect answer isnot given by (A), (8), (C), 0D) 8.11, (150-82.94:5) You are given: (Deaths are uniformly distributed over each year of age. i) onsps=025. 1 8849 9 Mee A ‘Which ofthe following are rue? / ve ta 10 0548 HK pasqevos = 05 Oaeeeaa W939. = 05 Be mosn0s 1 06 (8 Land oly (@) Tend TH only (©) Mand Monly——— (D) Etna tt (B) The correct answer is not given by (A), (B),(C), of ). 8.12, (3-800:12) Fora certain mortality table, you are given: bo gts aster 0S fins oF Pe (805) = 0.0202 Gi) 1481.5) = 0.0408 (i) 4482.5) = 0.0619 tiv) Deaths are unifonmly distibuted between integral ages. seeps ec weaic by Calculate the probability hat a person age 80.5 will die within two years. Beet 6 00782 (8) 0.0785 ©) 0.0790 ©) 0.0796 (8) 0.0800 8.13, You are given Deaths are uniformly distributed between integral ages. eau 03 Vane Gi) q= 04 Yow 4 2 pea Ae oae Gi) guar = 03. { ; it Ged Caiculate 2,02 i i , coe | 1 ww) Brio or é v ee ; Eaten Wes eee ie oo a 1 yo? @% 6 09 RY 11 "ekies continue onthe next pase ‘SOA MUCSuxy Maral elon Copan e009 At 126 8, SURVIVAL DISTRIBUTIONS: FRACTIONAL AGES 8.4, Deaths are uniformly distributed between integral ages. (44s = 001, ff tas tag) Gil) g46 = 0.011, if Calculate the variance ofthe 2-year temporary complete life expectancy on (45). 2 wen uemiat)= 7("tePedt (VC Peau 8.15. You are given: Deaths are uniformly distributed between integral ages. Gi) ope = 02. Calculate 8579 - ecm = a awe © 1 a wPed. 0 4 1c too theo Pao 9.8 Por 816, [4486-21] You ae given eae Be aa) geo = 0.020 a Gi) gs = 0.022 a (ii) Deaths are uniformly distributed over cach year of age. Calculate fyrg. 5 WAY 4 OS toMearT 10.40) (A) 1.447 (B) 1.457 (C) L467 RR 1AI7 (@) 1.487 8.17, [150-F89:21) You are given: © q=0040 ' Gi) an 044 | 1 (ili) Deaths are uniformly distributed over each year of age { \ i Cateuate Bp. settee (a) 1435 @ 1s © 1455 ©) L465 © 1475 8.18, [3-801:33) For a 4-year college, you are given the following probabilities for dropout from all causes: ska tr osts sane p9 (1.050.058) a= 005 n eae Ne Dropouts are uniformly distributed over each year: Compute the temporary 1.5-year complete expected college lifetime ofa student enteriig the second year, 2.53 (ay tas @) 130 © 138 By 140 @ 14s 8.19. You ae given (Deaths are uniformly distributed between integral ages. ) exvosan Calculate q,. a as.( reac awed S 0A MLC Sty Manat eaton ‘Beercises continue on the nex poze ‘Coppa @0005 ASH wee EXERCISES FOR LESSON 8 wT 8.20, (150-$87:21] You are given: () dpaklorx=0,1,2...901 Nowse aeons Gi) Enna = 38 wd Teas are uniformly distibuted over each year of #82 ° 20 Calculate soo oat wy 01125 jos @) 0:67 0.200 B21, (150-889:24} You are given: 7 i) Deaths are uniformly distributed over each year of age. os] i Gi) pass = 05 Aas 9 Mas: 04 i 4 ‘ 95 4 ° 1 Calculate 24s (a) 04 @) 05 (©) 06 () 07 gos 22. (CAS%.S04:10] 4000 people age 2) each Fay an antl er fand. Immediately after the 1,000" an efi wl be dissolved ad each ofthe survivors wl Be ele ‘$50,000 «Mortality follows the Iustrative Lite Table, using linear interpolation at fractional ages. 2 ish Oy 4 Reo Gravee Cateulate P. deve se ede reed FES (A) Less than 515 out a0 = * (B) Atleast 515, but less than 525 arene (CQ) Atleast 525, but less than 535 2 mot (ja) Atleast 535, but less than 545 (p) Atleast 545 tea ate auimey tea tea 8.23, [160.8735] Based on given values of fc and Ite APset = 49/50.inder the assumption of constant force of mortality Calculate yaPeetsa under the uniform distribution of deaths hypothesis. » ae er (A) 09799 (B) 0.9800 (©) 0.9801 ff 0.9802 ©) 0.9803 8.24, [160-889:5) A mortality study is conducted forthe age interval (x, ++ Uh a ‘Tf.a constant force of mortality applies over the interval, ¢2s9x¥01 05, 7 + We a PA Catena pans assuming a uniform Gstibution of deaths apples o¥=" wheinersa, NA (a) 0.044 GY 0097 (©) 0.050 (p) 0.053 & 0.056 8.25, [150-89:29] You are given that qx = 0.25: pcre 19%) act on he constant fore of metal assumption, the face of mo = os ved om the uniform dsibution of eas assumption the free of mony ian0 0. ‘Uniform distribution of deaths Balducet hypothesis Constant force of mortality 829. ‘Death rates are calculated according to the following assumptions: Mortality Rate_ ri ie 4a ya Which of the following is corrbot? (ay yal < yah < yah yal H>U @®) C>USH © UrceH (@) UrH>e @ H=c=U SON MIC Say Marant—Binedlon Exercises continue on thé nex page epi 2000 ASH 132 SURVIVAL DISTRIBUTIONS: FRACTIONAL AGES 8449. 160-891:5] With rospectto te interval (x, + 1], you ae given: @ py=0.5 Lit) mg! denotes the contra death rate under the hyperbolic assumpto ii) mf denotes the central death Tale under the exponential assumption, Calculate mf! — me, (A) ~0.028 (B) ~0.026 (©) 0.000 (B) 0,026 ®) 0.028 Additional released exam questions: CAS3-S05:31, CAS3-FO5113, ‘CAS3-$06:13, CAS3-FO6:13, SOA M-F06:16, CAS3-S07:24, CAS3L-S08:16, CAS3L-S09:3. Solutions 8.1, pPattesi/ais the density function, 8.2. Using equation (8.3), OF des Ps = ~ 34 So the sum ie 1 © PS eetltlte the probability that x + 3 survives for ha year In onde odo this, we beak the period IP etGin Yeah The probability ot surviving forte fst quater ofa year by eqeion Get MaPssaye = ZB, = 0.25(0.15) = 0.9695, Tals = abs. The probaly of r+ I)survving to x4 1d5 sam The answer tothe question i then laden UaPsvste = 1— 1yaPessye yaPaes = 1 — \(o.9605) iB at age x, with ly = 1. Then lyst = (10.1) = 0.9 and nal ages is obtained by lincar interpolation, so Lesos = 0.75(0.9) + 0.25(1) Joe1.as = 0:25(0.765) + 0.75(0.9) Jesias _ 0.86625 75) Alternatively, you could build a life table star tesa = 0.9(1 — 0.15) = 0.765. Under UDD, I; at fractior PP yn = 7 = Tos = 0.93649 dye = 1 ~ ippya = 1 ~ 0.93649 = 84, Thisisoapesn4~onPraoa: Th former is $221 = 22. The probabil A582 = 38, and the probability (x41) survivesto 12 1 ty of survival to x+1 is, by equation: (8.3), Eigt = 1210-15) = 097, Sothe answers 2 — (2) .0.97 [o.0ssa7s ‘Miematively, you cou use te life abe fom the solton to the lst problem, and lines iterpoate besoa = 0.4(0.9) + 0,6(1) = 0.96 SON MLC Study Manus eon Cony 2009 AS EXERCISE SOLUTIONS FOR LESSON 8 133 Jesnn = 0.7109) + 0.3(1) = 0.93 Jea1.2 = 0.2(0.765) + 0.8(0.9) = 0.873 8.5. Under uniform distribution between integral ages, Ixeas = 3(Ix-+lae1), since the survival function is a straight line between two integral ages. Therefore, hss = (9,164,051 + 9,127,426) = 9,145,738,5. Median future lifetime coceurs when le = 4(9,145,738.5) = 4,572,869, This happens between ages 77 and 78, We interpolate between the fags to get the exect median: ny ~ Clay ~ fp) = 4,572,869 4,828,182 ~ s(4,828,182 ~ 4,530,360) = 4,572,869 4,828,182 ~ 297,8228 = 4,572,869 4,828,182 — 4,572,869 _ 255,313 297,822 * 397,822 = 08573 So the median age at death is 77,8573, and the median future lifetime is 77.8573 ~ 4 8.6. First we use equation (8.5) to get qu. 4s 0.04 = ps1015 = T9759, 0.04 ~ 0.039. = a ‘Then from equation (8.6), 104, 1% 8.7, We use the fact that for each age, Ly = H(t ler) so\slsa = 4 (sa Bsa + 2a + fe3), yn we have dso amiss = 7 8° Tse + iss + 25a +155) Dividing numerator and denominator by ts2 23952 J+ 2psa + 2opsa + spo {1 ~ (090¥0.89x0.87)) + 2(0.90) + 2(0.90)(0.89) + (0.90}(0.89)(0.87) 0.60626 Soom = Wai000) SOA MLC Sy Manual eon Cop 2009 ASN 1a __8. SURVIVAL DISTRIBUTIONS: FRACTIONAL AGES ’. ‘The force of mortality is calculated as the derivative of In ,po __dlnepe _ Ait) ax de Tagg ORF OS = 0.0134449, JERE = 0.986533 and gs .986533 = 0.013467, so under UDD; 430 0.013467 025(0.013467) tee 0.013512. Has ‘The difference is 0.013445 ~ 0.013512 = [0.600067]. (B) 8.9. 5(20) = 1/12" and s(21) = 1/1.214, 50 qn0 = 1 ~ (L.2/1.21)8 = 0.03265, Then O4ga __0.4(0.03265) TBoes * Toran ~ Wntais] veqnaa 8.10. pare = 42 = GRP = 0.09091, s01 is true, This statement docs not require uniform distribution of deaths. By equation (8.6), ms = 24h. = 5249; = 0.042553, s0 IIs true. coves 1s SRI, which is 32%) = 0.018436, so TK is false, T can't figure ovt what mistake you'd have to make to get 0,021, (A) BLL. First calculate g,. 10.759, Geel By equation (8.2), o3gx = 0. By equation (8.5), ss05 = 7-55 = 2, making II false. (A) 8.12, We use equation (8.5) to back out gy foreach age. Hos Haas Odea = T 03095 = 0,5(0.6) = 0.97. Therefore by equation (8.3), os pans = $35- pat = 0.96 and o5peq = 2080, (Geshe. 960.97) = [0.0782] (A) SO MLC Stedy Monsl—Stheten Cepyigh 2000 0560 EXERCISE SOLUTIONS FOR LESSON 8 i 138 8:13, To do this algebraically, we split the group into those who die within 0:3 years, those who die between 0.3 and 1 years, and those who survive one year. Under UDD, those who die will die a the midpoint of the interval {assuming the interval doesn’t cross an integral age), so we have Survival | Probability’ ‘Average Group | _ time ‘of group __| survival time 1 | ©,03) | 1-osPr07 OS HT | 3,1) | ospeor1Pov07 | 0.65 Mm__|_(,e) 1P x10 1 We calculate the required probabilities. 09 oaPaso7 = ggg = 0.967742 09 sper » 2 (1 0.703) = 0.764516 Tm aaP07 1.967742 = 0.032258 : oapsva — 1Pasoa = 0.967742, ~ 0:764516 = 0.203226 saan * 0.032258(0.15) + 0,203226(0.65) + 0.764516(1) = [CHOTES2| ‘Alternatively, we can use trapezoids, We already know from the above solution that the heights of the first trapezoid are 1 1 end 0.967742, and the helghts ofthe second trapezoid are 0.967742 and 0.764516. So the sum of the area of the two trapezoids is Besanm = (03M OSI + 0.967742) + (0.7(0.510.967742 + 0.764516) = 0.295161 + 0.606290 = [0.901451] the expected value, we'll use the recursive formula, (The trapezoidal rule could also be used.) aad, Pes 048 + pustean ; an yse ‘ com wt stom -001s) { ral 979555 7 i: 2a ‘We'll use the formula in terms of ,p, forthe second moment. + e ¢ 4 nary =2 f° unsat wa you 0 : ¥ ea) ane z 1€0.99)(1 - 0.011(¢— a \ |. of [12-0010 tae (i -o0) on O1ny? = 2(0,49667 + 1.475925) = 3.94518 So the variance is 3.94518 ~ 1.979555? = [0.02654] 8,15, As discussed on page 8.8, by equation (8.8), the difference is i 5104 {SOA MLC Sty Manus eon ‘Copyah ©2009 ASS 136 ____ 8 SURVIVAL DISTRIBUTIONS: FRACTIONAL AGES 8.16, Those who die in the frst year survive year on the average and those who die in the first haf of the second Year survive 1.25 years onthe average, so we have P= 098 1.520 = 0.98(1 ~0.5(0.022)) = 0.96922 %ears = 0.5(0.02) + 1.25(0.98 - 0.96922) + 1.5(0.96922) = [EATS] (0) Alternatively, we use the trapezoidal method. tes sasea(*( tera $0000 + (Jase ase aso) 7303] @) 8.17. pio = 1 ~ 0.040 = 0.96, pra = (0.96)(0.956) = 0.91776, and by linear interpolation, 1 5p70 0.91776) = 0.93888. Those who die in the first year survive 0.5 years on the average and those who half of the second year survive 1,25 years on the average, So .5(0.96 + in the first, .5(0.04) + 1,25(0.96 — 0.93888) + 1.5(0.93888) = [445473] —(C) Sora = Alternatively, we can use the trapezoidal method, ‘The first year’s trapezoid has heights 1 and 0.96 and the seodind yéat's trapezoid has‘heights 0,96 and 0.93888, s6 Fara = 0.5(1 + 0.96) +-0,5(0.5)(0.96 + 0.93888) 4gti2] (©) 8.18, First we calculate py fort = 1,2, gi = 0.90 (1 aint ~ a2) = (0.90)(0.95) pa 2PL 1855 By linear interpotation, 15p1 = (0.5)(0.9 + 0.855) = 0.8775, The algebraic method splits the students into 3 group ropouts, and survivors. In each dropout group survival on the average is to the group, 1.25 years for the second group) and survivors survive 1.5 years, Therefore 873 = 0.100.5) + (0.90 ~ 0.8775)(1.25) + 0.8775(1.5) = LASSIE] (D) ist year dropouts, second year (dnrough time 1.5) point (0.5 years fr the first Alternatively, ve could som the two trapezoids making up the shaded area «Pt at the right 1MO.S)(1 40.9) + (0.5)(0.5)(0.90 + 0.8775) 0.95 + 0.444375 = [F394375] — (D) 4a 8.19. Those who die survive 0.25 years on the average and survivors survive 0.5 years, so we have she 0.25 osqei0s +05 ospuos = $ 0.59, 14, oas( 2a) -05( a8) =i 0 SOA MUC Stay Mast eon Copyien 2600 ASS EXERCISE SOLUTIONS FOR LESSON 8 : 137 0.1259, +0.5 ~0.59, 1s 22 1 2 Alternatively, complete life expectancy is the area of the trapezoid «Pos shown on the right, 80 1 Ml osPans 7 = 05105KI + osPu0s) ‘Then aspss0s = 3, from which it follows i: 05 1 ae os Vos a 059% ‘sos ile ceach year of age, mortality follows 8.20. Since dy is constant forall x and deaths are uniformly distributed de Moivre’ fae. We back oi a, Complete life expectancy is the area ofthe trapezoid shown below. I, 1 w~40 o=20 20 740 * ‘Alternatively, we split the population into 2 groups. ‘Those who die within 20 years survive 10 years on the average and the others survive 20 years, 80 1020920 + 2020p20 = 18 20 fo 40 =18 ea) (=a) 200 +2009 ~ 800 = 180 ~ 360 2 = 240 gu) U8 ro tw tole w= 120 (oaiti} (a) Once we have w, we compute sonogs0 = 95 821, We use equation (8.5) to obtain os ‘Then éysqi = 05(1 + (104) = [0] SOA MLC Sty Manust eon Copa ©2000 ASN Pers. 8 Son cs her far. der > SL Roe. Sond 138 8. SURVIVAL DISTRIBUTIONS: FRACTIONAL AGES (B22; According to the Ilustrative Life Table, ly = 9,501,381, $0 we are looking for the age x such that fe = “0:75(9,501,381) = 7,126,036. This is between 67 and 68. Using linear interpolation, since ley = 7,201,635 and deg = 7,018,432, we have 7,201,635 ~ 7,126,036 7,201,635 ~ 7,018,432 ‘This is 37.4127 years into the future. } of the people collet 50,000, We need 50,000(2) person. (D) 8.23, Underconstant force of mortality, sPy41 = Pf, 80 Px = y4Phyyj4 = 0.98" = 0.922368 and gy 0.077632, Under uniform distribution of deaths, x=674 7.4127 ~ 30. fr _ MI Abra =~ Oe (4/4(0.077632) T~ (1/4}(0.077632) 0.019792 = [0580208] — @) 8.24. Under constant free, spar = Pb 80 Pe = 0.95% 185494, Then under uniform assumption, 0.814506, a. 0254, _ (025N0.18549) oasis = TO 1g, = 1 -0.1(0185494) 8.28. For constant force, 4 is a constant, equal to ~ In p, = ~1n0.75 = 0.2877. Then in T= sq = 0.2877 = 0.2877 Mees 0.25 T- 025s 0.2877 — 0.25(0.2877)s = 0.25 0287; © (0925x0287) ~ ©) 8.26. We integrate ,p, from 0 to 2, Between 0 and 1, pe = eM Between 1 and 2, ps = PeriPast So the answer is 0.99 + 0.98(0.98) = © 8.27. We calculate the temporary expectation of life for one-year, 8.7, subtract py for the amount lived by those who survive, then divide by gi. p; and g; don’t use constant force of mortality assumption, 2), W028 | 6 P= * GO= 1 * Bt SOA MLCSuiy Maousl-sieden Copp ©2000 ASM SOLUTIONS FOR LESSON 8 7 gato Pisa fort < te Since py = €*, =~ In Gf = 0235966. Men welt ane [met os constant force, sPL 17/81 5235566 wen ane y= 0500S CBD fat TH sah. Under uniform dsliion, che numbers of $6075 ¢ each half of the year are equal s0 if 120 deaths athe fst bal of = 96, then 120 oceyred it sa ed alan y= 480 ~ 120, 360 hen oso = Sg = 02, then gen Qnsqor = 0-4 $0 Por = OF ‘Under constant force, ipa = 3 OS. The wer 360 VO = [21BBa8] ) og 8.29, ‘The Balducci hypothesis speeds wp deaths the most and the uniform hypothesis speeds them up the east, 80 the answer is (B). 830, In general, Balducei speeds up the ‘deaths the most and UPD the least. When ooking at deaths in the Jast aero year ts mean Bale wl hye et ‘and UDP the most, or (A): Sr ca yw help make ma CONTE, NH ee force of mortality, this equals 4s while under Balducel yadavs is 8 decreasing unetion of sand under UDD yj4dses 18.8% ‘increasing function of - 8.31. For the hyperbolic assumption, Aalst = bee TFs = Loot) _ 236)(284) DAB = 536 + 48 (2361284) = BOO - a70; 364 ars SES = 2702581 770.2581 - 236 _ B “ For the uniform assumption, 248 = 284 “236 — 284) = 284 ~ 4B which yields w= 075. 6-0 = omig7- 0.75 = [0003628] ®) 8.32. For the uniform hypothesis, 737) ‘ Qade, apesdaes = For the hyperbolic hypothesis, “SOA MUCSmdy Masaal—ihebon Repyneng200) ASH 40 & SURVIVAL DISTRIBUTIONS: FRACTIONAL AGES 2 —/t--9g 1-(1-a3/16)a, rd ~ sg, ~ B/16)q, (R~s)a. CHO, ‘The numerators ae equal, Bquating the denominators, we get s 8.33, Weave a) Pte: Gray 2. d-aan 9 T-GaF $90! ~ 42) = 12(1~ ba, $94, ~ 4948 = 12 1294392 52g2-6ig,+ 12=0 61+ Yer IER ee 104 Since dx < Ps, we choose the lower solution, 33 = [038]. (ay conta te Bom morally dapsone ihe yon Pulte ‘under the hyperbolic assumption, ‘he other formulas are correct. (B) 8:35. Tis incorrect. Dividing numerator and ‘denominator by I, we have 5, ‘numerator, tte 8nd there should not be a ¢in the 50 y must be 10. Sette o> 911 and hx $912. Then pig = HE = 0.826446 an 40 We solve for s ~ 0.826446 = 0.173554, 0911 =— Pe 0.826446 "Pio * soi9 ~ ORB6§4G + O.1TASSHG stig BBE ce. nom 0.080740 Savoy = 0465214 O.t7a554 = 046524 tion of deaths, (= 0468214017355) : -oassaiagionssais T= @aesziayo.r7riss4y ~OLDE] ey SCA MLE Sty Maat eon Cemyrght e009 asst EXERCISE SOLUTIONS FOR LESSON 8 pet Une UD, eel tee ft we F = 0.95, Under Baldueci, eee aecceteaee 9 We Sorig, OD+OM OE aoe: de oe frat [Fo -semt0-noy=aaatess 1o00(F ~ 6) = 10000095 ~ 0.948245) = [1755] O) 8.37. The best approach is to use 21 1 and 1 — qx = 0.9 and width 15 the area 0.04 8.38. 0.042553. 7 043478, Uk sage 0,88"? = 0.061917. ¥ @ 6:39, We'llealeulate yap and aapeo, The former minus the laters the probability of death between ages 60.5 and 61.5. 5 gop, alte deat cur in tha half the yor natin ats the probability of soe aris (03) = 085 andthe protabiiy of ving Y= i ‘Oil -0.5(04)) = 056. The difference is 085 ~0.56 = 029. ‘Under Bake, we have 525, The difference is so unten = 94 an agus = $e Then rape = 8 = 0.822529 aul ae congs = 0.823529 ~ 0.525 = 0.298529. So the answer is 10,000(0,298529 - 0.29) 85.29} (B) 8.401) Since the fature lifetimes are independent, ‘we calculate the probability of one hair not surviving and cube Bs eal a dying in yer lis On your 202, which ae uP 2: dee “tay = 5. Apply Baldueci, OIG!) T-@5\ 05902 3 ao sia = 2405050 cali) snccorsaa +0) 0a foresis} ©) cof one probe filing and cube it. Probability of filre in the first 2 years is 3 = 3. By hyperbolic assumption, 0.490905" (8.41. We calculate the probabil ‘1 0203, In the third year, do = Fy = 3 0250/7) 7-076") 157895 ojos26° =[o.06187] &) ‘504 MLC Sty Manasl- incon enn T008 ASM 144 9. SURVIVAL DISTRIBUTIONS: SELECT MORTALITY Table 9.1; Select and ultimate mortality table used in Example 9A, | gua _| avaer-| auger | gees | x43, 1 40 | 0.002 | 0.005 | 0.008 | 0.012 | 43 41 | 0.003 | 0.006 | 0.009 | o.o1s | 44 i 42 | 0.004 | 0.007 | 0.010 | ors | 4s x [guy [aos [a1ae2 [go [x43 ‘ i 21 i 19 2 204} 23 fl 21 24 ‘ 2 25 7 23 26 ' 24 27 : Suppose you were given Thble. 9.1.25 a.select-and-ultimate mortality table, Let's compute 2quaya. This is ‘quayra* Puoy2 Jeo, bectuse quays = ges when the select period is 3 years. quoye2 is read off horizontally on the row { for age 40, and isthe entry. 0,008. gas is the next entry horizontally, or 0.012. The answer is 0,008 +-0.992(0.012) = 0.019904, To test yourself, compiite ayqury. (Answer below!) t Suppose we wanted to construct a select-and-ultimate life table. We would want.to stat with a radix like 10,000,000 in the upper leftmost cell. But we would also want the tables to merge inthe tighimosi column, To build the table, we would have to start with age O duration 0, calculate hoy through the end of the select period n, thén ly for x > n, Afet that, we would have to work backwards to fil in the select table for all starting ages other than 0. Let's illustrate this with the above (small) solect-and-ultimate table. Exameue 9A You are giver @) Mortality rates are select and ultimate witha sleet period of 3 years, and are given in Table 9.1, Gi) baa = 10,000,000. i Compute hay Avswen: We must compute l4s recursively, 10,000,000(1 ~ 0.002) = 9,980,000 ‘ 9,980,000(1 ~ 0.005) = 9,930,100 ,930, 100(1 ~ 0.008) = 9,850,659 ,850,659(1 ~ 0.012) = 9,732,451 ys = 9,732,451(1 ~ 0.015) = 9,586,465, So far, the life table Looks like ‘OTFOD = 5566460 - 8101660 81 BMH" oH, 865060" (#100 1XSt00~ Dea DErOL660 = tnlas t 8101664 = (Go0'0~ iMeo00 ~ 1) = (de lungs —tge tbe ~) 30 NLC Sty Manat eaton Copy c0000 ASN 9, SURVIVAL DISTRIBUTIONS: SELECT MORTALITY _ : us =] te tat | tye | ds | 43 40 | 10,000,000 | 9,980,000 | 9,930,100 | 9,850,659 | 43 a1 9.732451 | 44 a 9)586,465 | 45 [Now we work backwards from ls to Zz. 9,586,465 aya = Fg = 2.688267 9,683,267 hay = piggy = 20751588 9,951,558 ay ‘The life table now looks like this: x fo Ket eye deg [43 40 | 10,000,000 | 9,980,000 | 9,930,100 | 9,850,659 | 43 at 9.732451 | 44 42 |_ 9,190,121 | 9,951,558 | 9,683,297 | 9,586,465 | 45 Fr additional practice, filin the row for (41). The completed table isin the footnote:> a Exam questions may ask you to fil in blanks ina life table, or to use select-and-ultimate mortality as you would use aggregate mortality to compute life expectations or mortality at fractional ages, Here is a pair of exam-style questions, ExaMnie 9B You are given the following select-and-ultimate life table with a two year select period. T [Lt tan | tne 90 91 | 1250| eo} 920 92 | 1000 | 900 ‘You are also given that guyss = “lqeur 1, Calculate fos 2. Caleulate ly wy | Saws | Laces PaseTre | Teroare |e sy | isveas | scvozvs | octowes | ors'cose | ir & | es90se6 | cores | oovoses | ooxoonr | oF erp i | em py Tm fe sa 2 5400 48 99 LL 066 = wn e'coss = SOS = ny oes = SOPH «my cranes = SF ” exes ny SOA MC Sty Maoust— tt eon Copyighi@n008 ASM 146 9. SURVIVAL DISTRIBUTIONS: SELBCT MORTALITY. Answ 1, Since gion = go2/3 and goer = $902. Goyer = 2qyon, But from the life table, qoxy = 1-7 = 0.1, $0 qpnys = 0.2 and loner = sy = 1150, Then gon = 1~ Hg = 0.08, 4go2 = 3qyn = 0.3, and gooyer = 34a = (F3qon) = 0.16. So Bars Vay Par ta @ rate Prasren 920 z Gay = Craver woo) _ loom = Gea —o.16) ~ AS 2, From the fife table, we have qioajst = 194/920, butt quays = 3493, 80 a Exercises 9.1. You ate given the following information from a 2-year select and ultimate mortality table: @ f OL Gi) GD) gue = (iv) fon Calculate iy 9.2, You are given the following extract from a 2-year select and ultimate mortality table: % | ha | Mee | bee 80 | 1000] 950 | 900 ai} — | 920 | = x] — | — | 860 ‘You are given that gauyss = gt Calculate yqiay1 SON MLEStdy Maal —Bh ton rere conte on the net pe. Copyght 2009 Ashe EXERCISES FOR LESSON 9 447 9.3, Fora slest-and-ltnae mortal able you ae given (i) Utimat monly follows de Motes In with = 120, Cid. During the 2 yar selec eid, a= i) Calculate ea. 94, Fora 2-year select and ultimate mortality table, you are given (ene =A, OS 1S 2. Gi) eon = 58. Gi) egya = 19. Gv) apy = 17. @) by = 575 Determine A. 98. Ina 2-year select and ultimate mortatity table, deaths are uniformly distributed between integral ages and durations. You are given the following mortality rates: riety 0 Peas jo] 09s | oas boa Pon > Io Se | dee ny - 2Pard = 9.0% j 2| 0.08 Peas Sere G 0.08 a. 437) 007 oPravy ) 006 0.0? Calculate agoinas- 9.6. Fora I-years integral ages and dur Calculate Bp) lect and ultimate mortality table, px) = Pe + 0.001. Deaths are uniformly distributed between 8. The complete life expectancy for (x + 1) is 78. 9.7. (150-887:12) From a life table with a one-year select period, you are given: x] to | dea | eo 85 | 1000 | 100 | 5.556 86 | 350 | 100 | ‘Assume that deaths ae uniformly distributed over each year of age. Caleulate (A) 508 @) 513 (© 5.22 ©) 5.30 (&) 5.39 2) etan revises continue on the nex page 148 9, SURVIVAL DISTRIBUTIONS: SELECT MORTALITY. 9.8, (150-F87:2] You are given the following extract from a 3-year select and ultiate mortality table: = Tey [cae [ton | toes [3 7} — | — | — | 1600) 79 n| — | aes} — | — | 72 | sors | — [592] — | 75 Assume: (The ultimate abe follows de Moivre’ av GY don = days = eas 8 = 70,71, 72, where us = gr ~ lane Calculate 1000971). (ay 26.79 @ 2a (©4337 ©) 4788 © 4899 9.9, (150-S90:21} Fora 2-year sslect and ulintate mortality table, you are givens (© Ukimate mortality follows the Musiraive Life Table 5 forall x. Say forall x. (iv) Calentate fon. 9.10, (150-891:10} Por a two-year select-and-ultimate mortality table, you are given: @ 1 ~28)ge ai, (=Bqet ity 90 Ww) 100 ow wi) Cateuate tsar (A) 82 B) 83 ou @) 85 (B) 86 9.11, (180-83-96:3] For a ten-year select-and-ultimate mortality table, you are given: Im x X55, * © toons FP (1- 75). os1< 10 tow = “P=! 10<¢<70 Caleutate dom (A) 21.0 (B) 39.0 (C) 42.0 (D) 45.5 (B) 48.5 SOA MLC Sy Mattel verte conte on hes pepe Copynahe eaand ase EXERCISES FOR LESSON 9. 149 9.12. {3-FO0:10) You ae piven the following extract from a select-and-timate morality table with 22-year select period ee ee ee 60 80,625 79,954 78,839 62 61 79,137 78402 77,252 63 62 72,575 16,710 75,578 64 ‘Assume that deaths are uniformly distributed between intepral ages. Calculate osacovos. (A) 0.0102, (B) 0.0103 (©) 0.0104 (@) 0.0105 (&) 0.0106 9.13. {CAS4-082:17) The table below represents a section of a select and ultimate mortality table. It shows the rates of morality quyen at alained age x-+1 among a group of lives insured at age x. 21 000120 “0.00150 0.00170 0.00180, 22 0.00125 0.00155 0.00175. 0.00185 23 0.00130 0.00160 0.00180 0.00195 ‘You are given tha zn = 1,000,000. Calculate hea (A) Less than 997,500 (B) At least 997,500, but less than 998,000 (C) Atleast 998,000, but less than 998,500 (D) Atleast 998,500, but less than 999,000 (B) Atleast 999,000 9A4-18, Use the following information for questions 9.14 and 9.15: [Thee force of mortality for a ife selected at age x follows the model: | HA = HHO, 20 Youares: | © O(x) =f +0,006S + 0.003x ag | ae {i if (3) smokes | | 0, otherwise © 10Phyq) = 0.96 ‘+ A superscript of s indicates the ease where S = 1 and n indicates the ease where S 9.14, (C3 Sample:21] Determine x such that gis) = df 9.18. {C3Sample:22] Calculate the probability that a life, drawn at random from a population of 30 year olds of which 40% are smokers, will survive atleast 10 years SOA MLE-Stty Mars —th een -Beeretes contre onthe nex page Cope oro ASH 150 9, SURVIVAL DISTRIBUTIONS: SELECT MORTALITY 946, [CAS4-S87:17] (1 point) You are given the following select-and-ultimate mortality table: X | ava | geser | gtae2 | guarss | devs 33 [0.02 | 0.015 | 003 | 0.025 | 0.035 34] 0.01 | 0.025 | 0.02 | 0.03 | 004 35 | 0.02 | 0.015 | 0.03 | 0.035 | 0.05 36 | 0.01 | 0.025 | 0.03 | 0.045 | 0.04 37 | 0.02 | 0.025 | 0.04 | 0.035 | 0.03 38 | 0.02 | 0.035 | 0.03 | 0.025 | 003s 39 | 0.03 | 0.025 | 0.02 | 0.035 | 0.045 40 | 0.02 | 0.015 | 003 | 0.04 | 0.04 41 | 0.01 | 0.025 | 0.035 | 0.035 | 0.035 42 | 0.02 | 0.03 | 003 | 0.03 | 0.035 Caleulate the probability that a life age 36 who has been insured for two years will live to age 40, (A) Less than 0,86 (B) Atleast 0.86, but less than 0.87 (©) Atleast 0.87, but less than 0.88 (D) Atleast 0.88, but less than 0.89 () Atleast 0.89 9.17, (180-898:1) For a select-and-ultimate mortality able applicable to patients after heart surgery, you are given: (Ultimate monalty follows the Illustrative Life Table, Gi) During the four-year select period, Pree = (0.804 0.058), k= 12,3 Catoulate 4pynye2- (A) 0.531 (B) 0539 © 0-781 ©) 0.857 (&) 0.916 9.18. [CASHA-S select period: £7] (2 points) You are given the following excerpt from a select-and-ultimate table with 2-year La] 1000946) 100044}61 10009542 4004380574 0.699 410.453 0599 0.738 42 0477 0.634 «0,790 43° 0510 0.680 0.856 44° 0551 0.737 0.937 Calculate 10004941). (A) Less than 1.20 (B) Atleast 1.20, but less than 1.40 (©) Atleast 1.40, but less than 1,60 (D) Atleast 1.60, but less than 1.80 Atleast 1.80 SOA MLC Study Manat edon xeretss conte onthe next page Conse @nne ase EXERCISES FOR LESSON 9 ast 9.19, {CAS4A-$97:10) (2 points) You are given two groups of people. Group 1 consists of 100,000 people, each age 30, selected at age 30. Their mortality is described by the following illustrative select-and-ultimate mortality table: Xa eet ered 29 0,00130 - 0.00134 0,00138 0.00142 30 0,00132 0.00136 0.00140 0.00144 31 000134 0.00138 0.00142 0.00146 32 0.00136 0.00140 0.00144 0.00148 Group 2 also contains 100,000 people, each age 30, taken from the general population. ‘Thi described in the following table: ' 0, midrule 0 0.00138 1 0.00140 2 0.00144 3 000147 Calculate how many more people from Group 1 survive to age 32 than do people from Group 2. (A) Less than 7 (B) Atleast 7, but less than 9 (©) Atleast 9, but less than i (D) Atleast 11, but less than 13, ©) Atleast 13 9.20, [{CAS4A-F98:14] (2 points) The following is extracted from a select and ultimate mortality table with a 2-year select period: Le) fy tener teva__ 42 24 42,683 26 2 — — 35,000 27 26 — — 26600 28 Forall x, goget = 1Squer and guaen = 12atenes Determine fey (A) Less than 35,000 (B) At least 35,000, but less than 35,500 (©) Atleast 35,500, but les than 36,000 (D) Atleast 36,000, but less than 36,500 (B) Atleast 36,500 SOA MUC Sy Maal edie iercises continue onthe net page Cope @2009 ASH 182, 9, SURVIVAL DISTRIBUTIONS: SELBCT MORTALITY 9.21-22, Use the following information for questions 9.21 and 9.22: The force of mortality fora life selected at age x follows the following model: Heal) = 9D, 120 GY) apes = 1.25apre5) 9.21, {150-98:39] Calculate fy, (A) 004 (B) 0.05, © 0.06 @) 007 ©) 0.08 9.22, [150-898:40) Calculate fy (a) 0.0012 (B) 0.0017 ©) 0.0025 ©) 0.0034 ®) 0.0050 9.23, {3-F01:2} Por a select-and-ultimate mortality table with a 3-year sleet period: @ 2 4g tn ty tees 8 60 0.09 O11 O13 O15 63 GI 0402s OG oes Ot ce GIS ete QISe iT E e165 63 0.12 O14 ONG B66 64 013 OST? Gi) White was a newly selected life on 01/01/2000. Gif) White's age on 01/01/2001 is 61. Gv) Ps the probability on 01/01/2001 that White will be alive on 01/01/2006. Cateutate P. A) Os P<043 B) 043s P< 0.45 © 0455 P<047 @) 047 n, O otherwise, ‘The sum of a term life insurance for n years and an n-ycar deferred life insurance is a whole life insurance, Deferred term insurance 1t is possible to combine deferral and term. The actuarial present value of an insurance payable only if death occurs between times n and n +m would be denoted by yy, ‘The random variable Z is ifn n+m oem ; Purcendownen | % P54 ArhereBs | dowment woren i insurance ven el 10.2 Constant Force of Mortality THis rather bard to integeate the expression for moments, because ofthe ¥ term, Very few density Funetions combined ‘vith v' lead fo something you ean integrate in closed form. Only a cdnstant density end an exponential density (constant force of mortality) will integrate easily, As aresult, the ‘majority of exam problems will involve a constant force of mortality, or atthe very leas, a piecewise constant force of mortality. ‘Lats frst work out the simplest case: constant benefit of 1, constant interest, constant force of mortality. The exponential distribution has no memory, so the answers will not depend on the age of the insured Exanene 10A The force of mortality for (x) is . The force of interest is 6. 1, Let 2; be the random variable for 1 unit of whole life insurance. Calculate B{Z:) and Var(Z1). |, Let Z be the random variable for 1 unit of term insurance for 5 years. Cateulate E{Za} and Var(Za). =001, 2 3, Let Zs be the random variable for | unit of 10-year deferred whole life insurance, Calculate E{Zs] and Var(Zs). 4 Let 2; be the random variable for the present value of 20-year endowment insurance of 1, Suppose 6 = 0.04, Caleulate E{Z4) and Var(Zs). Answer: 1. Under constant force of mortality, ,p, = eM". We have, EA] cE hemp lo ddt = [eter = Hol” ee ee rez] SOA MLC Stuy Mavenl—Bosedion Coppi 209 05 164 10. INSURANCE: PAYABLE AT MOMENT OF DEATH—~MOMENTS—PART | By the rule of moments, we calculate the second moment by doubling 5. f iets (uy p26 (av 2. The term insurance is the same as the whole life insurance, except that we integrate from 0 to 5 instead of from 0 t0 0, wale [ "Herne ale] BA = ig (1) = Hp ersrn) [ft \ 0 saan vn fate) - (8) EZ] = i he Mudt (evan) + asl? muy = IL eon : Var(Za) be ww) 4, We are using numerical values for this part of the problem to make it more concrete, He FFor the endowment insurance the expected value isthe sum of the expected values of term insurance and a pure ‘endowment, Let the term insurance be Zs and the pure endowment Zs, For term insurance, by substituting 20 for 5, we can use what we calculated above: B25] = —A (1 — ¢ 260) wad = oH 0.2(1 ~ 0.367879) = For the pure endowment, Ele) = appv = e449 = 0.367879 Therefore B24] = 0.126424 + 0.367879 We now eateulate the second moment. bya Mead if py (at Hy gta) g gta weal! re ‘SOAMLC Say Masti eon ‘Coprge 2009 nSht 102. CONSTANT FORCE OF MORTALITY 168 In the second integral, we pulled out the constant ¢~*8 and were left with the integral of the density from 20 to.co, which is the survival function, For an exponential, the survival function at ¢ is e™#”, Continuing, nel)eets (1 = 0.165299) + 0.165299 = 0.258043 9.258043 - 0.494304? = [0.013707 0 {ter working outa lot of problems, - willbe engraved in your head. Once iti, a convenient way to handle constant force of mortality questions forarbirary insurances i fo express them in terms of pure endowments and whole life insurances. Thus: «A defered whole ife insurance is a discounted whole life insurance. Ay = alc Ayan. For constant Free it follows that ners) wre ae « A term life insurance is an immediate whole fife insurance minus a deferred whole life insurance. Arm = Ag ~ nEx Aran. For constant force, it follows that (1 eo) pal") ¢ An endowment insurance is a term life insurance plus a pure endowment, For constant force it follows that Aen a (1 erm) + ero ‘You may split the insurance up into aterm insurance and a deferred insurance. This will work forcaleuating the expected vale, snes E(2, +25} = F(Z] +H8(Zel Spiting the insurance will work even for higher moments, because Zana 2 are italy excsive, 90 B{Ch + Za] = BIZ] + BZ) (ll the cross texms ae er). However, the Variance must be caloulated as second moment minus first moment squared, since Var(Z + Za) # Var(Z1) + Var(Za). Spliing an insurance into a term and a deferred insurance willbe especially helpful if @ parameter changes at some duration, For example, i he force of mortality or the force of interest changes atime f, you would evaluate sole fife insurance as & 1-year term life insurance plus a ryesr deferred whole life insurance, The next series of examples ilustrates this. Exanece 10B An insurance company offers « retirement plan which pays a $1,000,000 lump sum to a person who survives to age 65. For a specific purchaser, you are given: (i) He is age 50. His force of mortality is 0.005 at all durations. i) 6 = 0.05, Calculate the actuarial present value of the retirement plan for this purchaser. | Answer: The retirement plan isa pure endowment. The actual present value of a pure endowments x = VaPr- If forces of interest and mortality are constant, v* = eS and ,py = e%". The actuarial prsent value of a pure endowment is then ,, = e-*-Therefore, the actuarial preset value of the pure endowment inthis example is 1,000,000 sso = 1,000,000e"0%5#99)'s = (438,235) bo ‘SOA MLC Sy Manual son opsish 009 ASH 166 10. INSURANCE: PAYABLE AT MOMENT OF DEATH—MOMENTS-—PART 1 Exanus 10C An insurance company offers a retirement insurance plan, This plan pays $1,000,000 atthe time of death, if death occurs after age 65. For a specific purchaser, you are given (@ He is age 50, Gi) His force of mortality is 0.005 15 roo= i Sis Gi) 6 = 005. Caleutate the netuarial present value of the retirement trance plan for this purchaser, Answani In the previous eximple, we caleulated the-value of a’ 15-year pure: endowment for this purchaser at 0.438235. This factor discounts o age 50 the value of any benefit the person receives at age 65, Once the purchaser reaches age 65, the value of the insurance is os as (_¢ 1,000,000 = 1,000,000; = 1,000,000(6 55° Notice that at age 65, the lower force of mortality before age 65 is irelevant to the calculation, ‘The actuarial present value of the retirement insurance plan at age 50 is 2,000, om 125,210] 1,000,000 so = ouaaas 7 0 Students sometimes mistakenly think that the ofthe second period (0.02) inthe above example should be used for computing the discount factor sp to apply tothe deferred insurance in the above example, If you understand the above two examples, you will avoid this mistake, The pure endowment discount factor uses the w and 6 of the discount period, regardless of what you are discounting Now we're ready for an example where you would split the whole life into term and deferred insurance. Exaneut 10D A whole life insurance on (x) provides a benefit of 1000 atthe moment of death, You are given foor_r 10 Let Z be the random variable for the actuarial present value of this insurance. Cateutate Var(Z). ‘Answer: Let Z be a 10-year term life insurance for 100 and Z a 10-year deferred whole life insurance for 50. ‘The appropriate rate to discount a whole life insurance deferred for 10 years back to f = is e-'00"® = ¢-100011005) = 6, and we will use this inthe calculation of the moments of Zp. Then 7 0.0L 7 1040.014005)) -«*s wa) fears pa) (t-e mene) «195 0.02 as esout{ 002) safe Bias stereo)" 45) | — 8) fe) _ 50 seo) ef) = 3 Because ofthe way the parameters were rigged the expected valve came ou rational, but the second moment won't on) «ipa r 2.00 z ) ( 0.02 ) (0.02 + 0.08) 10,000 sos ett - 27% 12 (p= 1209 rao EZ) 2) = soot( 22! BYZa} = 100 era BZ] = 50°" E27] 2 495.1386 | ° “The variance is therefore 772.9164 ~ (2) SOA MLC Sey Mass ten (Copygh ©2009 ASM 168 10. INSURANCE: PAYABLE AT MOMENT OF DEATH—MOMENTS—PART J ‘An occasional exem theme isa benefit increasing ata continuously compounded rate, Yoit can net tht rate of, increase against 6 and then proceed as above. Examrte 10K A term life insurance provides a benefit at the moment of death of 99 if death occurs at time 4, 15 10, You are given: (4 = 001 Gi) 6 = 0.05 Zs the actuarial present value of the term life insurance. Caleulate Var(Z). Avsivex ‘The net force of interest after subtracting the rato of benefit inerease is 0.05 ~ 0.02 = 0.03. Doubling this net force of interest will yield the second moment, 5 (1 ~eremvomnen) 1082420 0.071916 _ e tobany BA Sor a(R) ‘Var(Z) = 0.071916 ~ 0.082420" 5123] A problem can also have the benefit depend on the single benefit premium. You then solve for the premium, Exasirus 10G A special insurance coverage pays a benefit of 1 plus a refund ofthe single benefit premium wlthout interest atthe moment of death, You are given that = 0.01 and 6 = 003, Calculate the single benefit premium, Answan: Let be the single benefit premium, Then (14 Aye *M0.01 dt _ lta a Table 11.1 on page 184 summarizes results for standard insurances (of I) payable at the moment of death for constant force of mortality Exercises aut 10.1. A ton-year endowment insurance is payable atthe moment of death, p(t) = 0.01 for > O and 6 = 0.06. Calculate the single benefit premium. ye eM AUS) eo M/ASY vow its 10.2. An -year term insurance payable atthe moment of death has actuarial present value of 0.05720, You are given @ ws(0) = 0007, 10. (i) 6 = 005. Detern SOA MLC Stay Macnal-6 etn exercises come onthe next page Cepygh 109) Ashe EXERCISES FOR LESSON 10 eSiietstett 169 10.3, {150-F87:16] You are given: Aen = 0.4275 Gi) 8 = 0055 (i) py 0.045 forall Cateulate Arm (A) 0.4600 (B) 0.4775 (©) 0.4950 @) 0.5245 ® 0575 10.4, {SOA3-F03:2] For a whole life insurance of 1000 on (x) with benefits payable at the moment of death: 4 _ [00% Ors 10 o a={oor 1o2 10.6. [CAS4A-S93:16] (3 points) Dave wants to purchase a $ year pure endowment with a net single pret ‘The amount of the endowment is $1,000. His insurance agent convinces him instead to use the same money to purchase a 5 year endowment insurance policy which pays at the moment of death or at the end of five years, ‘whichever comes first. You are given that je = 0.04 and 6 = 0.06. Caleulate the benefit amount for this 5 year endowment, (A) Less than $800 (B) Atleast $800, but less than $850 (©) Atleast $850, but less than $900 (D) Atleast $900, but less than $950 (©) Atleast $950 10.7. (CAS4A-F93:11] (2 points) You are given the following values for the net single premium of a whole life ingurance policy 1. Ag with force of morality j— and constant force of intrest 6 2. At with ul = ste +6, 6’ = 6, cis aconstant and ¢ > 0. 3. At where éf = ps, 6" = 6+, cis the same constant as in item 2 above Determine which statement is true. (A) L253 B)2>Ls32 Orbe DM) Badr SOA MLC Sry Manst—Bh eden erties comin on He net page. Coppin n00 ast 170 10. INSURANCE: PAYABLE AT MOMENT OF DEATH—MOMENTS—PART 10.8. {CAS#A-F96:10] (2 points) Bryon, « nonactuary estimates the net single premium for a continuous whole life potiey with a benefit of $100,000 on (30) by calculating the present value of $100,000 paid at the expected time of death, 30) is subject to a constant force of mortality j, = 0.05, and the force of interest is 6 = 0.08, Determine the absolute value of the error of Bryon’s estimate, (A) Less than $15,000 (B) Atleast $15,000, but less than $20,000 (C) Atleast $20,000, but less than $25,000 (D) Atleast $25,000, but less than $30,000 (B) Atleast $30,000 10.9. _(CAS#A-$97:21] (2 points) For some length of term, n, the expected value of an n-year term insurance with benefii payable at the moment of death equals the expected value of an n-year pure endowment. ‘The force of interest and the force of mortality are constant, Determine n, Ings) (ores; In(2q +4) ~ Inga) Pecan rere In(6) ~ Inu) © ae ae ©) aes (@)_ None of A, B, C, or D is correct 10.10, {CAS4A-F97:10} (2 points) The benefit under an m-year deferred whole life policy, with benefit payable at the moment of death, is twice that of a similar nondeferred whole life insurance. The net single premiums for these insurances are equal. You are given that = 0,08 and é = 0,06, Determine m. (A) Less than 2 (B) Atleast 2, but less than 4 (©) Atleast 4, but less than 6 (D) Acieast 6, but less than 8 ) Atleast 8 7 SOA MUC Sindy Manu 8i elon Exercises continue onthe net pore. (epyiat 2009 ASI EXERCISPS FOR LESSON 10 10.11. value ofa fully cor of interest is unknown, but is known to be c wt {[CAS4A-F98:17] (2 points) Which of the following is an expression for the expected actuarial present ‘nous whole life insurance, assuming a constant force of mortality #, and that the effective rate stant and uniformly distributed over the interval (0, 0.10}? (A) fl nemetinenar B) m5 © 10 f La +y-ne Maia ©) far nemarat ® 000 2” pallens 10.12, (CAS3-03:7) Given: @ i=5% (i) The force of mortality is constant Gi) 2 = 16.0 Calculate ays. (A) Less than 0.050 (8) Atleast 0.050, but Less than 0.075 (C) At least 0.075, but fess than 0.100 (D) At Teast 0.100, but less than 0.125, B) Atleast 0.125, 10,13. [CAS4A-597:18] (2 points) A continuous whole fife insurance is issued to a life age 30. This insurance pays an increasing benefit, starting at 1 at time of issue, increasing cont tinuously ata rate of 10% per annum for the first 10 yeats, and increasing 5% per annum thereafter. You are given: o a Detern “) @) © ) ® SOA MLC Suty Most Bean p= 0.05 6 = 0.06 ine the net single premium for this insurance. Less than 1.25 ‘At east 1.25, but less than 1.75 ‘At least 1.75, but less than 2.25 ‘At Teast 2.25, but less than 2.75 At least 2.75, ‘Bxerciescontiue on the nent poe eps en003 AS m 10. INSURANCE: PAYABLE AT MOMENT OF DEATH—-MOMENTS—PART 1 10.14, (C3 SamplesI1] A special insurance programm is designed to pay a benefit in the event a product fails, You are given: ‘* Benefits are payable at the moment of feilure. 300, Os1<25 100, 1225 004,120 (002, Os1<2s © (003, 1225 Calculate the actuarial present vaiue of this special insurance. (A) 155 @B) 168 © im @) 210 &) 213 410.15, {150-898:18} For a special whole life insurance on (0), you ate given: (Benefits are payable at the moment of death, 200 O 0 Gi) 5=0.10 ‘Which of the following are true? (A) Land Tt only (B) Tand IH only (©) Hand Monty (D) J, and tit (B) The correct answer is not given by (A), (B), (C), or (D). 50 MLC Sty Aicial-8ih ion Brercies conte onthe ex pig. Copyight ©2009 ASHE EXERCISES FOR LESSON 0° 173 coe 10.18, (150-891:3} Zis the present-value random variable fora whole life insurance of b payable st the moment of death of (3) be bw 7 ‘You are given: won feb Be ) © Hew 001 120 Hee : Hop ava) = + Gi) 6 = 005 ag Coreen) aria Gi) ‘The net single premium for this insurance is equal to Var). Calculate b. (A) 1.36 @) 1.68 (©) 2.00 @) 2.32 @) 2.64 40.19, [CAS4-$88:18} (2 points) five-year defered whole life insurance policy pays 10,000 atthe moment of death fr «person age 30. You ae given: (The foree of mortality is constant, 4 = 0.05. Ba bm Watt di) 6 =0.10, IN AIS ua syt Calculate the standard deviation of the present vale of the benefit payn bar preheat (A) Less than 1820 An Kee aan aw B) _Attesst 1820, but Less han 1840 aaa (C) Atleast 1840, but less than 1860 aad qe (D) Atleast 1860, but less than 1880 iieeeey (©) Atoast 1880 Ha 10,20, [CAS4A-S97:11] (J point) A whole lit and py = 0.04 for al x, ae Determine the variance of the net single premium for this insurance, (A) Less than 2.0 (B) Atleast 2.0, but less than 4.0 (C) Atleast 4.0, but less than 6.0 {D) Atleast 6.0, but less than 8.0 (B) Atleast 8.0 1021, [CAS3-I04:2) Fora 5-year deferred whole life insurance of J, payable at the moment of death of (2), you ae given: + Zs the present value random variable of this insurance. + 6=010 + n= 004 Caloulate Var(Z. (A). Less than 0.035 (B) Atleast 0.035, but less than 0.045 (©) Atleast 0.045, but less than 0.055 (D) Atleast 0.055, bot less than 0.065 (B) Atleast 0.065, ‘SOA MLC Sry Hasst—steton ‘Brercices comme onthe next page. epi 2009 ASN 174 10. INSURANCE: PAYABLE AT MOMENT OF DEATH—MOMENTS—PART 1 10.22.” {150-F87:26) Z is the present value random variable for a special continuous whole life insurance issued w@). ‘You are given for all mew = 001 Gi) 6, = 0.06 Gil), =e Calculate Var(2). (A) 0.033 B) 0.037 (©) 0057 () 0.065 (B) 0.083, 10.23, [150-F96:L1] For a special whole life insurance on (x), you are given: @ mO=", 120 Gi) 6 =, 120 (il) "The death benefit, payable atthe moment of death, i forthe ist 10 years and 0.5 thereafter, Civ) The single bene premium is 0.3324. (®)Zis tne presen-valus andom variable at issue of the death benef. ‘Calculate Var(Z). (A) Less than 0.07 @) At east 0.07, bt ess than 0.08 (C) At least 0.08, but less than 0.09 (D) At least 0.09, but less than 0.10 (©) Atleast 0.10 10.24, (SOA3-F04:1} Fora special whole Halt) = 0.05, 1 >0 (i) 5 = 008 (ii) ‘The death benefit at time is by = 9%, ¢ > 0, Gv) Zis the present value random variable for this insurance at issue, Calculate Var(Z). insurance on (x), payable atthe moment of death: (A) 0.038 (B) 0.041 (©) 0.043 (D) 0.045 ) 0.098, 10.25," [CAS4A-894:17] (2 points) A life insurance policy has a death benefit of 20 payable at the moment of death. Z is the present value at policy issue of the benefit payments, You ae given: Mz = 0.06 for x 20 6=0.12 Determine Var(Z) for this whole lif insurance policy. (A) Less than 22.5 (B) Atleast 22.5, but less than 27.5 (©) Atleast 27.5, but less than 32.5 (D) At least 32.5, but less than 37.5 @) Atleast 37.5 SOA MLCStady Manus eon Exercises conte onthe next page ‘Copygiee03 ASM EXERCISES FOR LESSON 10 _ 15 10.26, [CAS4A-$92:12] (3 points) is the present value random variable for & whole life insurance with variable death benefit b payable at the moment of death, For # > 0, you are giver: (Haas = 0.03 i) 6, = 0.05 Gil) b= 20" Determine Var(Z). (A) Less then 0.03, (B) Atleast 0.03, but less than 0.04 (©) Atleast 0.04, but less than 0.05 (D) Atleast 0.05, but less than 0.06 (B) Atleast 0.06 10.27. A special insurance pays « death benefit of 1 plus the benefit single premium with no interest atthe moment of death, You ate given that p(t) = 0.04 for allt and 6 = 0.03. Calculate the variance of the present value of this insurance. 10,28, (150-S88:30, 150-81-94:17] A continuous whole life insurance provides a death benefit of 1 plus a return (of the net single premium with interest at 5 = 0.04. ‘The net single premium for this insurance is calculated using j= 0.04 and force of interest 26. Caloulate the net single premium. wa @) 12 © 2B ©) 3/4 @ 1 10,29, [150-891:30] A whole life insurance provides a death benefit at the moment of death equal to 1 plus a return of the net single premium with interest at 6 = 0.08. ‘The net single premium for this insurance is calculated using j= 0.04 and force of interest 26. Caleulate the net single premium, (a) 025 B) 030 (©) 0.36 @) 0.42 © 048 10.30. A deferred 12-year endowment insurance provides the following benefits (A payment of I at the moment of death if death occurs between time ¢ = 5 and time ¢ = 12. Gi) A payment of2 if the insured survives 12 years. oon this insured, s(t), #5 005 for f < 8 and 0.10 for ¢ > 8. The force of ‘You are given that the force of mort interest is 0.10, Calculate the net single premium of this endowment, SOA MLC Sty Maasl—Bh een Exercises continue onthe net page Copy en00 ash 176 10. INSURANCE: PAYABLE AT MOMENT OF DEATH—MOMENTS—PART 1 1031. (CAS4A-S99:16) (points) Your chief actuary is concerned about the skewness of your book of business, You are given: (i) The forces of mortality and interest are constant Gi) A = 0.25 {Compute the third moment around the origin ofthe present Yalue random variable for a whole life policy issued 10650) (A) Less than 0,04 B) Atleast 0,04, but less than 0.08 (C) Atleast 0.08, but Jess than 0.12 (D) Atleast 0.12, but fess than 0.16 ®) Atleast 0.16 Additional released exam questions: CAS3-S05: (CAS3-S06:19, CAS3-FO6:33, SOA M-FOG:12. 17,38, SOA M-S05:7, CAS3- 15:35, SOA M-FOS:1 Solutions 10.1, We adil the values of a 10-year term insurance and a pure endowment. 10 am = [°eeereeoot arse 40% 1~e"\) oy =o. = ee = 0071916 +-0.496585 = [3585] 10.2, 0.465771 = 1 ~ erst 0.534229 = ¢° 0057 0.534229 0.057 10.3. First we have to back out e-? Hem = Be (l ~9Es) - 0.045 ,-(0.045+0,055)n\ 0425 = Sos cas (! ) 04275 om 045 095 ‘SOA ILC Sey Manaat—st eon {Cope ©2009 ASK EXERCISE SOLUTIONS FOR LESSON 10 _ om 0.05 = 6°" = Ach Bam = Am + Ach =0.4275 +005 = [0.8775] (B) le benefit premium for 1 is, ier + 108 Axst0 ix(l = 9B e) + 10 Asso 0.06 1 Taro I) at 96r0 + ase) +005 379272 + 0.214596 = 0.593868 “Therefore, for 100, the single Benefit premium is 10000593868) = [593868} (E) 105. ade=38rden = ¢ 2001s005-9o000 0.02 02+ 008 1 9!) 5 - ( ) OARS 1056. Agh = e582 = 01606531, whereas Ags = 4fi(t ~ €-°5) + eS = 0.763918, so the benefit amount sorance is 1000(344) = [733399] (A) interest rates lowers the value of insurance and increasing mortality increascs the value (B) If you wish to write out formulas, for endowment 10.7. Inwuitively, increas of insurance, so [2554 a= jeave 4, has a higher denominator and the same numerator as 1..and therefore must be less, whereas 2, increases 1s umerstor and denominator equally, which makes a number less than J closer to 1, or higher. 10.8. The expected value of an exponential distribution is the reciprocal ofthe force, oF apg = 20. The value of {$100,000 paid in 20 years is 100,0002-2909® = e°16 = 20,190, The true value of 100,004 is 100,000(59355. 38,462. ‘The error is [$8272]. B) 410.9. ‘The n-yeor term insurance has expected vale Big = Agente) = 2 (1 ermine) whereas the pre endowment has expected value e-, Setting them equal SOA MLC Sty Maul aon Cony 209 ashe 178 10, INSURANCE: PAYABLE AT MOMENT OF DBATH—MOMENTS—PART 1 emai “) nee neve)” se enon Ht mrs + 8) = Inp~ In(Qy + 8) Inu 4+ 8) - Inge rezy ® 10.10. In other words, the present value of a single unit ofthe m-year deferred policy is half the present value of single unit of a nondeferred policy, Since sis constant, Ac = Ayam, Which we use in the fist line ofthe following, ay = OSA, eum 0.5 olen 1011, The comet expressions 10 [2" [°¢1 +e, After negating ont, on aves a (8). 10,12, The mean of an exponential cistrbution isthe reciprocal ofthe foree, as explained on page 76, Here mean survival time is 16, so the force of mortality is 4 The force of interest 8 = In(1 +i) = In 1.05. Thus we have: awl, 208 Ay oserptoxan(_ (1/16) ) (1716) + In 1.05) 2oqaosasenessisy( __ 0.0625 (0.0625 + 0.04879) fuio6oed} wy 10.13, Let A be the net single premium, or the single benefit premium (they mean the same thing). We calculate the integral in two pieces: through duration 10, and later, = (0.1080)(0.5616) Bar ate tty [2.1\" erty asf Lite oases (1 fos99 (0.05)d fh : = [eemosie (i Camm racsa = (~3.40372)(—-0.13662) + (1.59233)(-0.81686)(-0.54221) = tos) If you wish to avoid integration, think of a continuously increasing benefit at a compound rate as a negative force of interest, Since the rate of increase in the insurance is per annum, the force of increase is In( I + 0. In effect, the net force of interest is 0.06 ~ In 1.1 in the first 10 years and 0,06 ~ In 1.05 thereafter. Therefore, = we +98 oes _0.05 (G17 — in 1.05; Oii-mit OA MLC Say Marl eon Cenygn one ast EXERCISE SOLUTIONS FOR LESSON 10 10) 4 —-O¥-I¥8040 816862) .40372(1- «0 40372(1 — 0.863382) + 0.863382(0.8 16862) = 4.17027) cy) 10.14, Let A be the actuarial present value. A= 300A) + 10025: Aas (00H _\ (yg asnataon) op 206000] s00( ofa) é ) +1006 soft canon = 155.374 + 12.750 = [1624] 10.15, Let A be the actuarial present value, A= 200, (001/00316) 4 190¢r 440069 0.0: aos +001 = 150(1 ~ 0.074274) + 60(0.074274) = 198.859+4.456 = [135] ) 10.16, Since B{Z] 5 = 3. Then ine B= a5 7 vag) =4-(2) = isa] @ 10,17. Lis true because j and do not vary with x, 80 Ay does not vary with x For constant force of mortality we know that RQ) and so Tis true and IITs false. wo 10.48, For one unit, BZ] = 325 = } and VortZ) = 5g ~ BIZ = 0 we need b such that b_ 2s 6 > 396 396 | o- Bon 2 © 50h MLC Sudy Maslin edtion Enya 2009 ASH 182, 10, INSURANCE: PAYABLE AT MOMENT OF DEATH—MOMENTS 10.28. Let A be the net single piemium, Then 00.4) 0.04 | 0.04 020 * 0:12 Va A=ge5a 4=[8] 10.30, We will calculate the actuarial present value of the benefit in 3 pieces: (1) the benefit for times 5-8, (2) the benefit for times 8-12, and (3) the pure endowment benefit:The actuarial present value of the fist benefit is, ['a0amosa «O88 $85 (e805) = as705 ‘The actuarial present value of the second benefit is 4820. 1044 0.102" 7 ig a Aromat) = (149182)(0.11118 08293, ‘The actuarial present value of the third benefit is 2-805-810-0300 « 96-2 = 0.27067 ‘The total net single premium is 0.05706 +-0,08293 + 0.2706: 1031, Since jig = 0.25, 6 = 34. The tied moment around the origin (or the thitd raw moment is obtained by tripling the fore of interest in order to cube the discount factor), so the third moment is oe ag 7g © 508 MLC Sty Manssl eon Copy @a009 ASM Lesson 11 Insurance: Payable at Moment of Death—Moments—Part 2 Reading: Actuarial Mathematics 4.14.2 0t Models for Quantifying Risk 2nd or 3rd edition) 5.3-54, 5:6 11.1 de Moivre’s Law ‘You ean calculate the present value integral for survival time having a uniform density, or de Moivre’s law, Rather than using the expression pj) inthe integral, it is casior to use the conditional density function fr(, which is constant, and then use equation (10.1) instead of equation (10.2). For de Moivre's law with e, the conditional density for survival time of (3) is b>. ‘Exanrux 11A A 20-year deferred insurance on (35) pays a benefit of 10 at the moment of death if death occurs no earlier than 20 years from now. You are given: (@ Mortality for (35) follows de Moivre's law with « = 100. (i) 6 = 0.06 isthe random variable forthe preset value ofthe insurance. Caeulate (2) and Var). BIZ\= £ 102-2044 ay by 6 [Note that the density is 25 for ¢< 65 and 0 for ¢> 65, s0 we set the upper bound ofthe integral equal t0 65. p= 10-2) Jeael” Z)=10\~F5)\o106)° lan 10. : = 10 (e009 _ 0) ooo ono = SD i fad = £ ote ae -[eitsl(erm-o*") ‘Answer: ‘The expected value is (0.12065) 100 = Fy (0.090718 — 0.000810) = 1.157798 0.638835) o ‘Table 11.1 summarizes results for stendard insurances (of 1) payable at the moment of death for constant force of mortality and deMoivre mortality assumptions. Notice that the APY of an insurance under deMoivre mortality {s equal to an annuity-certain for the period of insurance at force of interest 6 divided by time until w. 1720391 ‘Var(Z) = 1.157798 ~ SON MLC Sey Maras! edion 183, Cepynen ences ast 184 11, INSURANCE: PAYABLE AT MOMENT OF DEATH—MOMBNTS—PART 2 ‘Table 11.4: Actuarial present value under constant force and deMolvre mortality year tenn Hy pas) ine aval! | ao) deferred lif Hewson presen year deferred life ae ais a (oo +m) Penne com | Clleera) 11.2. Other Mortality Functions Another mortality function which could be integrated, with technique, is a gamma distribution. A gamma integrand also arises when evaluating increasing insurances, so we'll discuss technique in the next section, Other mortality functions could be integrated only if the benefit pattern were arranged: to cancel out the ob- Jectionable parts of the mortality funetion, or if zero interest is assumed, or if the interest function is altered; for example, if simple interest js used. Using zero interest isn't interesting (no pun intended), since then all you are doing is caleulating the probability that a payment will be made. We'll give an example ofan altered benefit pattern in the next section, 11.3 Integrating cre" Integrands of the form cf"e™, ¢ a constant, n usually an integer and a a positive real number, don’t come up often fon exams but they can come up. You ean integrate it by parts. If n = 1, one integration by parts will be neceseary, but ifm = 2, «wo integrations by parts will be necessary, tc. You want easier techniques, Clearly we can factor out c, so let's ignore it Let’ first consider the easier case of integrating from 0 t0 60, 113.1 fP eae ‘An exponential distribution has + seit = [Px Leta = }. Then the integral for E{X") is a times the integral we're interested in. So the integral we're interested in [leetwa tt, by Cant ExAM@rue 11B Future lifetime for (40) follows a gamina distribution with a Caleulate Ag, 8 = 10. 5 = 0.06. SOA MLC Study Mans eon Copyright @2000 AS ass 114. INCREASING AND DECRI Answer: For this gamma, the density is ou JO) = Fae Lf 01, ele m7 wom S i 1)I forma positive integer) so the APV of the insurance is By the above, this is (Remember that Tn) (0059608 100.165 ‘fyou are familiar with moment generating functions, there is an even simpler way 10 do bing asked for Ev!) = Efe™®% for a gamma distribution, But this is the moment generat ‘gamma distribution evaluated at 0.06, The moment generating function for a gamma i MM =~" ‘M(-0.06) 1 — (10)(-0.06)]-* = 1.6 a For the special exses of n= 1 and 2 = 2, ay (12) 11.3.2 fitear “The most common integral you will encounter is fi! teat, The formula for iis fetun 13) Instead of memorizing this, you may want to lear the technique discussed on page 875. 11.4. Increasing and Decreasing Insurances “The benefit under an insurance may increase in @ linear pattern: 1 if death occurs in the first year, 2 if death occurs Inthe second year, and so-on, The actuarial present valve ofan insurance which pays at the moment of dest if death occurs in the nth year is denoted by (ZA), with the usual additional decorations for term insurance, deferred insurance, and endowments, The pure endowment in the endowment insurance (/A)m would pay n. The deferred insurance would be for in the frst year after the deferal period, and would increase thereafter. "Areal tife application of increasing insurance i an insorance which refunds annua) premiums without interest. Similarly, «decreasing insurance (which must be term, since it eventually is zero) pays Inthe fis year n= 1 in the second year and so on, and its actuarial present vale is denoted by (DA) an if payable at the moment of death. ‘anendownent i also posible bt would be weird, snes it would pay 1 ifthe life survived m years, ihe benefit increases or decroazes continaousl, so that it pays ¢(fincreasing) orm ~ ¢ (if decreasing) if death ceats at inet, the symbol for its actuarial present valve has a bar over the Tor D. Note thet UA), < (A).y tho benefit under the later is the benefit under the former rounded vp tothe next integer. A real-ife appli bf continuously decreasing insurance is a warranty which pays a pro-rata proportion of the value ofthe item you ‘bought, rather than the fll value, SOA MLC Sy Manual eon Copy 000 ASN 186 411. INSURANCE: PAYARIB AT MOMENT € Exawers 11C An insurance coverage pays a benefit of ¢ at the moment of death if death occurs at time f, You are siven = 002 Gil) 6 = 005 (ii) 2s the present value random variable forthe insurance, Caleulate B(2) and Var(Z). ‘Answer: The moments are waa) [te 0004 lo wia\= ["Pe®™oo2u I Using the shortcuts we just learned above, we know from equation (11.1) thatthe first integral is OP, or EIZ) Sinilriy rom equation (11.2), the second integral is 242, of 2(0.02) _ 40000 178 os * 23} a 11.5. Variance of Endowment Insurance EZ") ‘The variance is #900 — (20)? ‘An endowment insurance is a sum of a term insurance and a pure endowment. If we let Z be the random variable for the term insurance, 2 for the pure endowment, and Zs for the endowment insurance, then B=D+h and therefore, by equation (2.1) on page 19, Var(Zs) = Vae(Zi) + Var(Za) + 2Cov(Zh, Za) Since the term insurance and! the pure endowment are mutually exclusive, B[Z:Z) = 0, and therefore Cov(Z,Z) = ~{Z;) B{2Z). Thus if we know the means and variances for the term insurance and the pure endowment, we can calculate the variance of the endowment insurance, Examete 11D You are given: ( Aly = 0.1 Gi) “Alyg = 0.09 GD) 1370 = 09 Gv) 1= 0.04 (¥) 2 is the present value random variable for the payment under a 15-year endowment Catculate Var(Z). SOW Mec Sy Mas Capyiah D209 ASM. 187 11.6, NORMAL APPROXIMATION [Axsiwexs ‘The mean of the term insurance is givens 0.1, and he variance ofthe term insurance is 0.09 0.1? Jet Z; be the present value random variable forthe pure endowment, The mean ofthe pure endowment is 09 Riga) = Agth = EPG = 1499738 teal = Auth = Ga. * 04s * 7" “The variance ofthe pure endowment can be computed using the Bernoulli shorteut which we learned in section 2.4 on page 23 since the payment is Bernoulli—either you get rr or you don't—so itis (0.90.1) TDR = 0027149 ‘f you don’t like the Bernoulli shortcut (but why not?), you can also compute the second moment and subtract the mean squared, aeOo) BUA) = 7 gg = 0277487 Var(Za) = 0.277487 ~ 0.499738" = 0.027749 ‘So the variance ofthe endowment insurance is Var(Z) = 0.08 + 0.02749 ~ 2(0.1)(0.499738) = [0:007805 o “The samo principle could be used to calculate the variance of a whole life insurance from the means and variances ‘of an n-year term insurance and an n-year deferred insurance. 11.6 Normal Approximation For a large group of insureds, the distribution ofthe present value of benefit payments can be approximated as « normal distribution with mean and variance as calculated by the methods of this lesson, When working out such problems, remember thatthe Variance ofa coverage of x on one insured is 2? times the variance of a coverage of 1, but the variance of a coverage of 1 on 1 insureds is only» times the variance of a coverage of 1 on one insured, Exaxeue 118 A company sells an appliance with a 5-year warranty, The warranty pays 100 atthe time of failure if the appliance fils within 5 years. You are given () The force of failure is 0.1 Gi) 6 = 005, i) 250 appliances are sold at onetime. Using the normal approximation, calculate the size of the fund needed atthe time of sale in order to have a 95% probability of having sufficient funds to pay for failures under the waranty ¢ Let the payment present value random variable be Z, Then wcll ‘Avsw 1014005) = 20 (1-005) = Ba ogress = 35.1756 BIZ’) = (000755) ~ 801008) = 5000(1 - e“*) = 5000(1 — 0.367879) = 3160.6028 Var(Z) = 3160.6028 — 35.1756? = 1923.2826 “The 95th percentile of the standard normal distribution is 1.645, Therefore, the fund needed is 250(35.1756) + 1.645 -Y(250)(1923.2826) = [9934.55 ° SOA MLC Stay Marat etten Copyngbe e009 ASM 188 11, INSURANCE: PAYABLE AT MOMENT OF DEATH—MOMENTS—PART 2 Exercises 114. [CAS4A-199:15} (2 points) If, = 103 - x for 0 < x < 103, and the force of interest is 6 = 0.06, calculate Asm (A) Less than 0.24 (B) Atleast 0.24, bat less than 0.30 (©) Atleast 0.30, but less than 0.36 D) Atleast 0.36, but less than 0.42 Atleast 0.42 11.2, (180-887:5] A continuous whole life insurance i issued to (50). Zs the present value random variable for this insurance, You are given: (i) Mortality follows de Motvre's law with w = 100, Gi) Simple interest with ¢ = 0.01 Git)», ="1000- 0.47 Calculate B{Z). (A) 250 (B) 275 ©) 500 ©) 625 ©) 750 1L3. | (CAS4A-$94:19] (2 points) You are given that a life is subject to de Moivre's survival function with terminal ‘age 100 and force of interest é = 0,06. Catoulate Adray. (A) Less than 0,125 (B) Atleast 0.125, but less than 0.126 (©) Atleast 0.126, but less than 0.127 (D) Atleast 0.127, but less than 0.128 (B) Atleast 0.128 114, [Based on CAS4A-$95:3} (2 points) ‘The survival distribution follows a de Moivre function with w = 80, ‘The force of interest is 6 = 0.05. Determine the net single premium for a 5-year term insurance with benefit $1000 for a life age 40. (A) Less than $75 (B) Atleast $75, but less than $85 (©) AUeast $85, but less than $95 (D) Atleast $95, but less than $105 B) Atleast $105, 118. Mortality for (40) follows de Moivre's law with « = 100, ‘The random variable Z represents the present value of a whole life insurance payable at the moment of death, 5 = 0.06. Calculate Var(Z). SOA MLC Sty Monte aton Exercises conte onthe next page Copyright ©2009 ASM 189 EXERCISES FOR LESSON 11 _ 11.6, {CAS4A-S95:18) (2 points) You are given that the probability density funeion of Faure Tifeime 1 for (2) is 4 foro3 Crem « 6=0005. Calculate the size ofthe initial fund needed to satisfy claims 95% of the time, using the normal approximation, 11.16, (150-F97:10] An auto repair shop plans to sell a new brand of car battery with a 4-year warranty. You are given: (@) The warranty promises the return of cash equal tothe pro-rata share ofthe price ifthe battery fails within 4 years. i) The pro-rata share is proportional tothe remaining time of the warranty period. For example {ails 2.50 years following purchase, 37.5% of the price will be returned. Gil) ‘The price ofthe battery is equal tothe sum of the manufacturing cos, loading for profit, and the actuarial present value of the warranty, (iv) The manufacturing cost of the battery is 50. () Profit is equal to 10% of the manufacturing cost. (vi) The failure rate ofthe battery is j(?) (vil) 6 = 005, Calevlate the price of the battery. (A) 66.69 B) 67.68 (©) 68.75 (D) 84.49 ©) 86.28 ifthe battery 504 MILC Sty Manual don Exercises comtaue onthe nex pase coy e009 A 11, INSURANCE: PAYABLE AT MOMENT OF DEATH—MOMENTS—PART2 11.17. {CAS4A-$94:10} (2 points) 100 independent lives, each age 35, wish to establish a fund that will pay a benefit of 10 at the moment of death of each life, ‘They will each contribute an equal amount at the inception of the fund and will then make no further payments to the fund. You ate given: Hz = 005 forx = 0 5= 003 Using the normal approximation, determine the minimum amount that must be contributed to the fund by the {roup to have 95% confidence that all claims witl be (A) Less than 600 (B) AUeast 600, but fess than 625 (©) Atleast 625, but less than 650 (D) At least 650, but less than 675 @® Acteast 675 A118, [C3 Saimple:t, CAS4A-197:9} A manufacturer offers a warranty paying 1000 athe time of fllure for each ‘machine that fails within 5 years of purchase. One customer purchases 500 machines. The manufacturer establishes 4 fund for warranty claims from this customer. The manufacturer wants to have at least a 95% probability that the fond is suficient to pay the warranty claims, ‘You are given: ‘The constant force of failure for each’machine is pt = 0,02, ‘© The foree of interest is 6 = 0.02. ‘© The times until failure of the machines are independent. Using the normal approximation, determine the minimum size of the fund, (A) 27,900 (B) 55,600 (©) 78,200 @) 86,400 (8) 90,600 11.19. A 5-year endowment insurance of 1000 is payable at the moment of death, 6 = 0.05, 1,(1) = 0.03 forall « ‘An initial fund of 800 per insured is sot up. Using the normal approximation, calculate the number of insureds needed so that there is 2 954% chance that the fund is sufficient, 11.20, {3-$00:13} An investment fund is established to provide benefits on 400 independent lives age 2. On January 1, 2001, each life is issued a 10-year deferred whole life insurance of 1000, payable at the moment of death, (i) Bach life is subject to a constant force of mortality of 0.05. ii) The force of imerest is 0.07, Calculate the amount needed in the investment fund on January 1, 2001, so that the probability, as determined by the normal approximation, is 0.95 that the fund will be sufficient to provide these benefits. (A) 55,300 (B) 56,400 ©) 58,500 @®) 59,300 (®) 60,100 S00 MLC mg Marts Exercises come on the nex page Copyreh 2009, | | | | EXERCISES FOR LESSON 11 193 11.21. [CAS4A-192:4] (1 point) Let Z be the present value random variable for a 10-year term insurance with & ‘benefit of 10 payable at the moment of death of (30), You are given that B{Z] = 5 and Var(Z) = 375. Determine “Ajy.79: (A) Less than 3.6 (B) Atleast 3.6, but less than 3.7 (©) Atleast 3.7, but less than 3.8 (D) Atleast 3.8, but less than 3.9 (B) Atleast 3.9 11.22, [CAS3«04:4) A fund will pay death benefits of $10,000 on exch of 900 independent lives age 30. You are given: + p= 001 ‘©The death benefits are payable at the moment of death. ‘The initial amount of the fund is established so that the probability is 0.95 that sufficient funds will be on hand to withdraw the benefit payment atthe death of each individual, Caleutate th (A)_Less than $1.90 million {B) Atleast $1.90 million, but ess than $1.95 million (©) Atleast $1.95 miltion, but less than $2.00 mil (D) Atleast $2.00 million, but less than $2.05 million (Atleast $2.05 million 1 fand amount 11.23, {34F01:8] Ench of 100 independent lives purchase a single premium 5-year deferred whole life insurance of 10 payable at the moment of death. You are given: “@ (i Git) F is the aggregate amount the insurer receives from the 100 lives. ‘Using the normal approximation, caleulate F such that the probability the insurer has sufficient funds to pay all claims is 0.95. (A) 280 @®) 390 © 500 @) 610 & 70 {SOA MLC Sty Maus! eon Exercises conto onthe net page Cope G20 ASHE 11, INSURANCE: PAYABLE AT MOMENT OF DEATH—MOMENTS—PART2 11,2425, Use the following information for questions 11.24 and 11.25: {A bullder offers Hfotime-oFthe-house fire proléaton on houses that he bulls The Farce of falare de wo We isa constant equal 0.0005. (@) 5 = 0.10. ‘Any fice toss will be a total loss and coverage rem (ii) There re no other causes of loss. Gv) Payments are made atthe time ofthe Gre. ins in effect regardless of « change in ownership. 11.24. [CAS4-S88:19] (2 points) Caleutate the actuarial present value of this benefit fora $100,000 house. (A) Less than $350 (B) At least $350, but less than $400 (©) Atleast $400, but tess than $450 (D) At least $450, but less than $500 ®) Attest $500 1128. [CAS4-888:20} (2 points) The builder has 150 $100,000 houses on which he plans to offer this protection. {In addition to putting the actuarial present value of the benefit in a reserve, the builder will set aside additional funds 0 that the probability is 0.95 that the funds will cover all losses, Caleulate the amount of additional funds needed for each house, (A) Less than $600 (B) Atleast $600, but less than $800 (©) At least $800, but less than $1000 (D) AL least $1000, but less than $1200 (B) Atleast $1200 11.26, % is the presont value of a 10-year term insurance payable at the moment of death. Zz is the present value of a {0-year deferred insurance payable at the moment of death. 7s isthe present value of a whole life insurance payable at the moment of death, You are given: @ (= 0.01 for 0 <1 < 10. Gi) 6 = 0.04, Gi) BIZ div) -Var(Za} Calculate Var(Zs). SOA MLC Sty Sanat etion ‘Brecisescomthue on the nes page opp 20 ASM EXERCISES FOR LESSON 11 39s 11.27. (CAS3-FO3:6] Let Z; be the present value random variable for an n-year term insurance of 1 on (x), and let Z; be the present value random variable for an n-year endowment insurance of 1 on (x). Claims are payable at the moment of death, Given: @ v=0.250 1) Pe = 0400 (i) fz] = 0.400 iv) Var(Z,) = 0.055 Cateulate Ver(Zi). (A) 0.025 (B) 0.100 © ous @) 0.190 @ 021s 14.28, (150-$89:1} Z isthe present-value random variable for an 1-year term insurance payable at the moment of death of (x) with by = (1+ i) Determine Var(Z), ao (B) age ©) Ag~ nde ©) nde aPs (8) "Ae ~ Gas)? Acdditional released exam questions: SOA MLC-S07:27, CAS3L-S08:21, CAS3L-S09:11 Solutions 11. Recognize this as deMoivre with w = 103, As we mentioned, a (eri or whole life) insurance then equals 4 continuous annuity-certan divided by length of time to «, here 103 ~ 45 = $8. For an endowment insurance, we ‘most add a pure endowment. Aacan = Spt aes Lael? get? 80.06) * 38 0.200806 + 0.197334 = [0.398140] «D) 11.2, The density funtion is a constant * 1000 0.10 (1 sic air Gol _ 1000 f° 1 -0.0001"2 Baa 14+001t dt 750] &) SOA MC uty Manat —Bén eon Cepyist e2008 ASME . 196 11, INSURANCE: PAYABLE AT MOMENT OF DEATH—MOMENTS—PART 2 11.3, Time to w is 100 ~ 40 = 60. 425330] (B) 114, Time tow is 80-40 = 40. 10004 as, = 100038 Laos eae oa (110.60. ® ILS. Time to w is 100 ~ 40 = 60. The mean of Z is w fz) = aaae [er emsay 60 Hie dite 0.270188 = tee ~ 02701 (en 008) and the second moment is computed the same way but doubling the interest rate 22) = Seas EZ’) 60 Toe? * (eoyo.ra) ~ 918785 So Var(Z) = 0.138785 - 0.270188" = [0687 11.6, Time to w is 75. We calculate the first 2 moments, The second moment is calculated using 26. Biz) = ages = 0.133260 75(0.10) ‘Var(Z) = 0.133260 ~ 0.260395? = [Gea 1.7. The density function is constant, 0.02, The mean is 1 EZ) = 002 {” wont! 0.02 * OL = 0.216 7 wm sou ( SOAMUC Study Mapal—a0h eon Copyighe 200s ASM © EXERCISE SOLUTIONS FOR LESSON 11 7 if 197 ‘The second moment is 1 2) - |-009 | eon @) vac =} 260? = 118, Because interest is zero, you should use a special, simple, method to calculate the variance. There are only two posible present values 100 if death occurs in the first year and 200 otherwise. So the preset value random see aie is-¢ Bernoulli variable-—a variable which can only assume one of two values. The variance of such @ vanable is the product ofthe probabilities ofeach of the (wo values, imes the square of the difference between the ahs, "The probability of dying in the fist years jh, and the probability of not dying inthe fist yea is So the vatiance is 100) 5% = [900] ®) 119, This i deMoivre morality function with = 100, and time tow at age 25 38 75. ‘Therefore = In += Int. Jams tz) = “ett eee (: ss ais) 75(0.055310) \' ~ 1.18, = (0.139894)(0,907704) = 0.126982 an EZ) 75 i 1 = W0mTd al'-ais) = (0,069947)(0.991481) = 0.069351 1000 var(Z) = 1000(0.069351 — 0.126982") = [$3367] (©) 11.10, This is a generalized de Moivre, with p, = ('9 iy [ps [2% = {i050 30-9) 1 woos" a SON MLE Sey Manes! don Cenmid 2009 aS 1}, INSURANCE: PAYABLE AT MOMENT OF DEATH-—MOMENTS—PART 2 0.06, We have wey [re"@onu From formula (11.1), this integral is -f o 11,12, First we back out , Remember to double 6 # eons 75 = 02514003 = 0.08 ‘We must evaluate ([A),: - A, f recseesenrooaa = 004 [tear lo ‘The integrand js from a gamma distibution with @ = 10, a = 2, so the constant needed to integrate to 1 1g Therefore this integrates to 100, and we have (, = 0040100) =[7] ©) 1.13, The warranty isa continuously decreasing insurance. Is expected value is 60(B Ayr. 60 f bv pode 10 = 60 | (10~1e9e-0™ 9.02 41 hb ‘0 = 12 (loc 9M) ae I Let's evaluate the two exponential integrals. The second one will be eveluated using our shorteut formula, equa tion (11.3), 0-106" [Peer 00] k fea = al! = (1+ @.nuo)e0000) = 100(1 ~(1 + De") = 100-200"! So the present value of the warranty 1.2(10(10~ 106") ~ (100 ~ 2006") = 1206" = © SOA MLC Sty Mant hon Cepyram e200 AS EXERCISE SOLUTIONS FOR LESSON !1 ie au 199 LLA4, Presumably the benefit is payable at the time of fallure. This results in an expression requiring integration by parts, The setuarel present value is {f00—amyeaeeeootar cette) [woe Fee ee eanae mee frets th an fetta = 10e"®* 5065 + 100(¢ = 1102" — 150¢°% 3) If you are the memorizing type, you may instead evaluate the second integral sing formula 11-3: firettarm [rere f "ean =aa((-G+o use) (1-(1 +osc)e*)) alee or =1502"° + 1102" (sets te") ating the two ings togetor, we have oe"! 4095-08 (1106 ~ 1506 IS, Let be te value of 1 for one retigertor tht breaks own, Noe hat the density i constant between 3 and 5 3 5 sage naa= [ eromaesrootars [et fo hs w7 4 ois = er = 0.105165 + 0.085465 = 0.19063 ean ig 030 _ ¢r850) = Sa ety, 2 =i ay = 0.097987 + 0.070060 = 0.168047 ‘Var(Z) = 0.168047 — 0.190637 = 0.131707 For 1000 refrigerators and 500 per refrigerator, the mean is 1000(500)(0.19063) = 95,315, and the variance is 1000(5007)(0.131707) = 32,926,804 so the fund needed for 95% confidence is 95,315 + 1.645 32,926 804 = [104,754, ‘SOA MLC Stay Maca eon pyran ast 200 __I]. INSURANCE: PAYABLE AT MOMENT OF DEATH—MOMENTS—PART 2 116, "ICs reasonable to assume that the warranty pays at the moment of failure. Since it refunds pro-rata, it constitutes a continuously decreasing insurance. If we let P be the price of the battery, then P= SO(1.1) + P(0.25(DA)hn) 5 25a So the key is calculating 0.25(DA)ha). Since the failure rate is uniform, the density is fs, and the integral we need is: 0.25(DA)La f bv sar =f (Ae So-o05if 1) “Lat Let's first calculate 2 useful integral, the second using formula (11.3) en ee [ f ‘We can now caleulate 0.25(DA)ha, 02KBia = 7 f = (2-26 1 ae on gage (1 A+ 0.2) = 400 ~ 480e"% 8 + 10(0.818731) = 0.18731 So the battery price is ;=ySty5; = [BT6TERE], (8) LLI7. ‘The expected value ofthe payments is 000 H. 100010) 3 e and the variance is | H ) | iat wg) wa 3 So the fund needed for 95% confidence is 625 + 1.645 V639.2046 = [66659] — (D) SOA MLC Sty Ham Cony @n009 ASME EXERCISE SOLUTIONS FOR LESSON 11 201 LLB, Let Z be the expected value of 1 atthe time of failure of one machine. 2 BIZ (1 = $9) = 0.090635 0.02 ‘Tors 200.03) 090635? = 0.078179 EZ) (1 = e785) = 0.086304 ‘Var(Z) = 0.086394 ~ ‘The expected value of 1000 for 500 machines is 1000(500)(0,090635) = 5,317 and the varlance is 10007(S00)(0.078179) so the fund needed for 95% confidence is 45,317 + 1,645 [39,089,046 = [55,602 11,19. Let Z be the present value of a 5-year endowment insurance of 1, We will ealculate the first moment, the second moment, and then the variance. 39,089,646 (B) oo as +o05 t~ 231-08) se% = 07050 EZ) = f0031085)) 4 «8003085 (1 ersemez00n) , ¢-soesr200 3 ae = (0%) +0 nears ‘Var(Z) = 0.632343 — 0.793950" ‘An slematve method fr caeulatng the variance, which is harder in my opinion, is fo ase the method of Section 115 Teamry out the eaculton jst show you tht it works, Lot 2 Be he present vale rendom variable a the Sea erm insurance and Zs the present value random variable ofthe 5-year pare endowment, Then 1001986 -H000005) * meas I~ 4) = 0.123630 1510.03 + 20005} 05} = 31-28) = 02110297 a “Var(Z,) = 0.110297 — 0.123630" = 0.095013 fq) = e7800209 «6°94 = 0.670320 H{ZB] = ¢ SOOO = 65 ~ 0.572046 Var(Za) = 0.522046 ~ 0.670320? = 0.072717 ‘Thon the variance of the 5-year endowment insurance is Var(Z) = Var(Zy) + Var(Za) ~ 28(Z,) 84a) = 0,095013 + 0.072717 ~ 2(0.123630}(0.670320) = 0.001986 SOA MLC Sud Manoal—Atnedon Copy 0009 ASM 202 11, INSURANCE: PAYABLE AT MOMENT OF DEATH—MOMENTS-—PART 2 ‘That completes the alternative calculation of the variance, ‘The fund needed for 95% confidence for n insureds is (1000n)(0,793950) + 1.645(1000) VO.001986n 0 for 1 insured, it is 793.950 + eae 300, 73.3087? 3 (egg «ar = ass 147] insureds are needed for 95% confidence. 11.20, Let Z be the present value of a 10-year deferred insurance of 1 = 0.05 __ -rop012) BIZ = Soss007" = 0.125498 0.05 pz?) = 295 _,-r0(019) BZ) = 5o5 5007) 0.039360 ‘Var(Z) = 0.039360 ~0,125498? = 0.023611 ‘The fund for 95% confidence is (400)(1000}(0,125498) + 1.645(1000) y(400)(0.023611) = 50,199 + 5,055 = [55284] (A) 24. ay 375 (5\* ‘aa = tn * (a) 11.22, Let Z be the actuarial present value of insurance of 1 for 1 life. © ool Oor+ 0.04 ~ 0.2 If is the present value of aggregate death benefits, {5} = 900(10,000)(0.2) = 1,800,000 Var(S) = 900(108)(0,071111) = 64 x 10° Marl) = 80,000 So the answers 1,800,000 + 1.645(80,000) = [EUSIEG00}. a) ‘SOA MLC Siaty Mapa eon Copy 2009 ASK EXERCISE SOLUTIONS FOR LESSON 11 203 11.23. Let Z be the actuarial present value of the insurance for each life, The expected value of the insurance for each life is Ez] = 10 i =100.04 | ls 9089, 046°2 May 426123 = 4e°% = “The second moment of the insurance is calculated using twice the force of interest, or 0.12: 1 [7 7 1000.04) Fe ls = 25e%8 = 11.233224 1219 04° Mar ‘The variance is therefore 11.233224 ~ 2.426123? = 5.347153. Using the normal approximation, the insorer would need 100(2.426123) + 1.645 1006 547153) = [2806510] (A) is 11,24, The actuarial present v Hw). ease sooo00(4) = $100,001 0) 11.25, ‘The variance for a single house is fe (ey wtb] For 150 houses, the variance is 150 times this. The additional funds needed forall 150 houses equal the 1.645 times the square root of the variance for 150 houses, oF 1,645 -f150 % 24,690,137 = 100,109 Perouse his comes otto I = [6675]. 8) 690,137 1126. ° bin = xO tte) = 0078 vi) < pxPOl eM «0057 ‘Var(Z;) = 0.065937 ~ 0.078694? = 0.059744 Cov(Z,Z4) = {Zi Za] ~ Blea) EZa] = 0 ~ (0.078694)(0.35) = ~0.027540 ‘Van(Zs) = Var(Z1) + Var(Za) + 2Cov(és Za) = 0,059744 + 0.23 + 2(-0.027540) [0.234658 SOA MLE Sudy Manali een ‘Capri ©2009 ASH 204 __11. INSURANCE: PAYABLE AT MOMENT OF DEATH—MOMENTS—PARY 2 41.27, Let Zs be an n-year pure endowinent, Then we have, as discussed in section 11.5, Vat(Za) = Vas(¢s) + Var(Zs) ~ 2(24) 825) 2 is a Bernoull-type variable, with mean yp" = (0.400X0.250) = 0.1 and varkance Pal ~ nP)9™ = (0,400)(0,600)(0.250") Since Z = 2 + Zs, BlZ1) = B[Za] ~ B{Zs] = 0.4 ~0. 1s, 13. So we have 0.055 = Van(2y) + 0.015 ~ 2(0.3)0.1) Vari21) = 0.055 ~ 0.015 +2(0.2y0.1) =[058] (8) 11,28. Because the discount factor is (1 + 1), the present valuc of the benefit is | if death occurs within n years, O otherwise. This js a Bernoulli variable Git only has 2 possible values). ‘The probability that it equals 1 isthe Probability of death within n years, or ngy. So the vatiance ofthe insurance is nl ~ nfs) = ate vaPs or (D) $08 MLC Stady ana) ion Coppi @200 ASM Lesson 12 Insurance Payable at Moment of Death: Percentiles Reading: Actuarial Mathematics 4.1-4.2 or Models for Quantifying Risk (2nd or 3rd edition) 5.3. [Bxam questions on percentiles were rare before 2007, bt one question sppeared on Spring 2007 MLC and allegedly at least two questions appeared on the unreleased Fal! 2007 MLC, ‘The 100p* percentile for the present value of payments on an asthe number € such thatthe probability thatthe random variable is less than £ is p- For a level benefit whole. surance, the present value of the payment, Z, is v", where 7 is sut Since v < 1, Z is a monotonically decreasing function of 7. Thus to determine the p* quantile of Z, we need «such that PZ ~nuy/d) = p 0 the p quantile (the 100p" percentile) of Z is the w such that the probability of surviving an amount of greater than (~Inu)/6 is p. By determining the 100(1 ~ p)th percentile of 7, we determine the pth percentile of Z. This can be seen in figure 12.1a, Similar remarks apply (0 term and endowment insurances, which are also ‘monotonically decreasing functions of 7; see figures 12.1b end 12.1¢, ‘To repeat, for a level benefit whole life insurance, the 100p! percentile of the actuarial present value is Joased on the 100(1 ~ p)' percentile of the time fo death random variable. So if is the present value random variable for a whole life insurance on (x) and I wanted to calculate the 30th percentile of Z, 1 would determine Aya, for a certain 1, What percentile of the su Exaneue 12h For life age 30: (H Ayolt = 0.0001" Gi) 6 = 0.05 Gil) Z.is the present value random variable for a 35-year endowment of 1 on (30). Calculate the 40th percentile of 2. Answan: We celeulate the 60th percentile of 7, (You may recognize this as « Weibull distribution.) We calculate pao [oro = e(-222) Psat 0.00010 ‘ines. 90, ‘SOA MLC StayBszal—ti eon 205 Copy eno ash 206 __12. INSURANCE PAYABLE AT MOMENT OF DBATH: PERCENTILES Zz 1 San Za vw 7 <20 og : Eto T's 20 06 : 04 02! 10 6-200 30 40 50 7 10 200 30 40 50 7 (© Whore (0) 20,yertem zal Ps2 zu, Ts2 oe POTS 20° 08) Str rs20 0.6 | 06 04 oa 02 02, 0 2 a0 40 37 13030 a (6) 20-year endowtent (0) 20-year defend whee Figure 12.1: Relation of prosent value random variable to survival time random variable, ‘The F such thal ;pro = 0.40 is the 6Oth percentile of 7, since there is a 405% chance of living this long or longer and therefore a 60% chance of living less. So 2 nl 0001 Jos 3 ; 20001? «naa = 09a P= 2748.72 t= 30.1799 ‘The corresponding value of Z is e~°0500.1795 If thad been 35 or greater, the answer would have ‘Been €°8.95135), o A pitfall to avoid, especially with constant force of morality, is treating s1 as a discount factor. Suppose X is 4 random variable, and f(2) is a monotonically increasing function, and you want the pth percentile of /(X). You would caleulate the pth percentile of X. IF itis mp, the pth percentile of f(X) is f(a). In applying f, the random variable X no longer plays a role. Tn our case, X ts the time at death and (2) isthe function expressing the APV of & whole life (or some other) insurance in terms of X, like v*. The pth percentile is w*, oF €°*, not e-W¥O%. is part of the random variable, not a discount factor. Jn the next example, we illustrate this SOA MLC Sid Marut—a0 eden Copan @n008 A 207 Gi) 5 = 0.05 iii) Z is the present value random variable for a 20-year term insurance of 1 on 30 payable at the moment of death 1. Determine the median of Z. 2. Determine the 80th percentile of Z. ‘Answer 1. Nolice that Z is monotonically decreasing as survival time increases, If survival time 7 is greater than 20, Z = 0, which is less than its value if Z < 20. ‘Median survival time is # such that ,ps = 05, and ,pso = 0 205 ni 34,66 > 20, the median of Z, the prosent value of the insurance if the insured survives 34.66 years, is 2, Now we need z such that = 1116 <20 This not o -006r <006Ien8/002 — gg then the present value of the insurance is € ‘A trickier case is when Z.is not a monotonic function of 7. This is true for a deferred insurance, where the benefit is worth 0 if 7'is during the deferral period, but is positive thereafter. Ifthe deferral period is n; to calculate the 100pth percentile, you must consider the point x > 1 such thatthe probability that T is between x and wis 1~ p. ‘This i ilusirated in igure 12,16. Examen 12C Fora life age x 2) po( = 0.00017 Gi) 6 = 0.05 iii) Z.is the present value random variable for a 20-year deferred insurance of 1 on (30). (so) and 6 are the same'as in example 12.) Calculate the 80th percentile of Z “Answer: We have to caleulate x > 20 such that Pr(20 <7" < x) = 0.20, This is illustrated in figure 12.2, where is ‘chosen so that the interval between 20 and xis the interval on which Z is maximized and such that there is a 20% probability of 7 being in that interval. We have: 9.000102 ~ exp (2 Pr < 20) = 0.765928 = 0.234072 SON MLC Sty Masses Sonigh 1009 ash 208 ‘HE 12, INSURANCE PAYABLE AT MOMENT OF DEATH: PERCENTILES —_—_—___+T 10 20 X30 0 50 ‘ Figure 12.2: Deferred Insurance example, example 120 ‘So we want x such that Pr(7 < x) = 0.434072, Pr(T > x) = 0.565928. | re | (2002) seme 2 i OHO 05658 = 0560290 x= VI7078.63 = 25.752400 ‘ ‘Then Z = ¢-005025.5240) Non-monotonie present values cen also occur ifthe benefit is not level. An extreme case is a continuously increasing insurance, whose present value is 77, so it reaches a maximum and then declines thereafter. ‘o calculate \ the 100pth percentile, one would have to find a less than the maxiraura and B greater, such that the probability that ‘ isin (a,6] is 1 ~ p. Such a question would be too dificult fr the exam, \ Exercises I 12,1, Z is the present value random variable for a 30-year term life insurance on (x) payable at the moment of death, You are give ' _ {001 +<10 I O w= 0.02 4210 (i) 6 = 0.04 | Calculate Pr(Z > 0.25). I SOA MLC Study Haml—8i eon Beercises continue othe next page... Copy eza09 ASM 209 EXERCISES FOR LESSON 412.2, (4-$86:21) You are given the following for (40): (Mortality follows de Moivre’s law with w = 100. Gi) Z= the present value random voriable for a 5-year deferred life insurance of 1. Gil) M= Mode of Z, ‘Which of the following are true? L Fx(0)>0 I M=v Mm Pz < 9) =05 (A) Tand ILonly @) Land II only (©) Wand Ml only (D) Mand (B) The correct answer is not given by (A), (B), (C), oF (D). 12,3, A special whole life insurance on (50) has a benefit of 1 plus a refund ofthe single benefit premi interest, payable at the moment of death. You are given: (Mortality follows deMoivre's law with «= 80. Gi) 6=0.06 Caleulate Pr > 0.5). 124, Fora 10-year deferred whole life insurance on (2) payable at the moment of death (@). Zis the present value random variable. : for +10 @ HO™ Joon 13 10 Gil) 6 = 008. Caleulate Pr(Z. > 0.6). 125. Fora special whole life insurance payable at the moment of death on (35): (@_Zisthe present value random variable. 1000 +10 2000 +> 10 Gi) Mortality follows the Illustrative Life Table, with uniform iv) §= 0.06 Caleutate Pr(Z > 700). a 4, -| ion of deaths between integral ages. 12.6, Zs the present value random variable for a whole life insurance on (35) paying a benefit of L000 at the moment of death, You are given: Mortality follows the Ilusteative Life Table, with hyperbolic interpolation between integral ages. Gi) =005 Calculate Pr(Z > 500), 01 for ¢ > 0. 6 = 0.06. 12,7, A 10-year deferced whole life is payable at the moment of death, 4x(0) = Calculate the 90th percentile of the present value random variable for the insurance, SOA MLC Sealy Manas eon Buercltes contin onthe next page Copy 0009 ASME 210 7 12, INSURANCE PAYABLE AT MOMENT OF DEATH: PERCBNTILES 128, [C3 Sample:44) For a 10-year deferred whole life insurance of 2 payable atthe moment of death on a fife age 35, you are given: The force of interest is 5 = 0.10, © The force of mortality is: = 0.06. ‘© Zis the present value random variable for this insurance. Determine the 90th percentile of Z, (A) 0.1335 B) 0.1847 © 02631 @) 0.4512 ) 0.4488 129, [3-800:241) A risky investment with a constant rate of default will pay: @) principal and accunmilated interest at 169% compounded annually at the end of 20 years ifit docs not default; and Gi) zero if it defauls, A risk-free investment will pay principal and accumulated interest at 10% compounded annually at the end of 20 ears ‘The principal amounts of the two investments are equal. ‘The actuarial present values of the (wo investments ate equal at time zero. Caloulate the median time until default or maturity of the risky investment, as @) 10 ou @) 12 (13 vF. The force 12.40, {CAS4A-196:6] (2 points) A whole fife insurance has a present value benefit function Z. of interest is twice the force of mortality, and both are constant, Determine the ratio of the mean of Z to the median of Z. “ (8) AUeast 0.80, but less than 0.95 (C) Atleast 0.95, but less than 1,10 (D) Atleast 1,10, but less than 1.25 (B) Atleast 1.25 12.1, You are given: (® (2) is subject to a constant force of mortality, pe() = 0.05. (ii) Zs the present vatue random variable of a 5-year deferred whole Ii payable at the moment of death, Gili) 5 = 0.05, Calculate the 90th percentile of Z, surance of 1000 on (x) with benefit 12.12, A 20-year endowment insurance on (40) is payable at the moment of death, You are given: (i) Mortality follows de Moivre's law with w= 100. (i) = 0.04 Caloutate the median of the present value random vatiable for the insurance, SOA LI Sey Manual eon ‘eretesconinue on the net page. Capit ©2009 a3Ht EXERCISES FOR 2a 12.13, Z is the present value random variable for a 40-year continuously decreasing insurance on (40) payable at the moment of death. You are giver (Mortality follows deMoivre's law with wo = 100. (i) 6= 005 Calculate the 60th percentile of Z. 12.14, Zs the present value random variable for & whole life insurance on (x) payable at the moment of death. ‘You are given: @ 10 Gi), 6 = 0.04 Calculate the 80th percentile of Z. 1018 12.15. Zis the present value random variable for a whole life insurance on (x) payable at the moment of death, You are given: @ o> B.1<40 (ii) 5 = 0.04 Calculate the 30th percentile of Z. 12.16, Z is the present value random variable of a 10-year deferred whole life insurance on (x) payable at the moment of death. You are gi @ px) = 0.0lr Gi) 6=004 Caleulate the 70th percentile of Z. 12.17. Z.is the present value random variable of a 60-year continuously increasing insurance payable at the mo- ment of death. You are given’ @ m= 0.02 Gi) 6=005 Calculate the maximum value of Z. 12,18, You are given: (x =0.001.0.059. (i) 6 = 0004 Caleulate the 75th percentile of the present value of a whole life insurance of I on (25). 12.19. Zs the present value random variable for a whole life insurance on (x) payable at the moment of death You ate given @ pat) = 0.01(.07) (i) 1=005 Calculate the 40th percentile of Z. ‘SOA MLC Sty MagaalB edton ‘Ecercies continue onthe mex page. Copycat 0008 ASM 212 12, INSURANCE PAYABLE AT MOMENT OF DEATH: PERCENTILES 1220-21, Use the following information for questions 12.20 and 12.21: ‘A special whole life insurance has @ benefit of 100 for yenrs 0-10 and 200 thereafter, payable at the moment of death. You are given: p= 0.02 for allt, i) 5 = 0.04, Gil)” Zis the present value variable for the insurance. 12.20. Calculate the median of Z, 12.21, Calculate Pr(Z < 80), 12.22, Fora special 20-year term insurance payable a the moment of death: (Zs the present value rendom variable, Gi) The benefit is by = "+ < 20 Gil) = 002 Gi) 6 = 0006 Calculate the 80th percentile of Z, Additional released exam questions: SOA MLC-S07:22, CAS3L-S08:22 Solutions 121, Z= 9%, 50 Z> 0.25 = eA 5 0.95 In0.25 TQ) < = 34.66 0.04 However, since it is a 30-year term insurance, Z = 0 unless 7(x) < 30, so we need the probability of death 30 years, wh 1 = er 00110-00200) 1 — ¢-O5 = [195955] 12.2. I Let be time to death for (40). Since Z can never be less than zero, Fz(0) = Pr(Z = 0). A S-year deferred life insurance pays nothing if death occurs within 5 years, so Pr(Z_ Pir s5)> 0.0 IL For z > 0, the density function f(z) for x > 0 can be calculated from the density function for 7, which i fe(®) = 1/60, ‘The transformation from T to Z is z = g(t) = v, of ¢ = g°(@) = Inz/Iny, Note that is 8 monotonically decreasing function of 7, so that the formula for transforming densities requives a negative sign. We get L 1 ‘60zinv — 60zIn(i +f) ‘The larger zis, the smaller fz(z) is. The value v* is the largest possible value for Z, so it is not largest value of S80). X MLZ < if and only if death occurs within S years (so that Z = 0) or T > 30 (so that Z < ¥® dus to discounting 30 years). The probability of death within 5 year is 1/12 andthe probability of death after 30 years is }2 = 0.5, Therefore, PriZ < v9) = 05+ 1/12# 0.5. X. (B) a Ha = fg SOA MLCStiy Manual ton oprah nme AS EXERCISE SOLUTIONS FOR LESSON 12 a 73 30, so we have 12,3, Let Pe the single benefit premium. w ~ int [om 1600830) *~"30(0.06) P=(1+P\As 0.463723, 463723 0.463723, 0.864707 ve (r 10, ‘The probability of that is : oonom6s) - [003909] 1oPs ~ wa26sPx 128. There are two ways Z could be greater than 700: 1. 7Q5)-$ 10 and 1.06-T° > 0.7. 2, T(BS)> 10.and 1.06-79 > 0.35. We'll caleulate the probability of both. 6.1212 1.064 = 0.7 => “The probability of 7(35) < 6.1212 is derived by calculating Zs.rz by linear interpolation in the Ilustrative Life ‘Table: ).1212(9,259,571) + 0:8788(9,287,264) = 9,283,908. loam ‘The probability of 7(35) < 18.0169 is derived similarly. 1s369 = 0.0169(8,712,621) + 0.9831(8,779,128) = 8,778,004 1 = caiapas + 1oPss — wsoveopss: Using bys = 9,420,657 and las = 9,164,051, ‘The probability of Z > 7ot 9,420,657 ~ 9,283,908 + 9,164,051 - 8,778,004 21> 100) = en aa ‘SOA MLC Stty Mast eon pyri 209 ASM 214 _12, INSURANCE PAYABLE AT MOMENT OF DEATH: PERCENTILES 12.6. Z > 500 if and only if 1.0579 > 0.5, or 7(35) < = In0.5/In 1.05 = 14,2067, We need gaverpes. Under hyperbolic interpolation, by equation (8,13), this is pay/(L —0.793344s). 1~0,00546 = 0.7933(0.00546) = 0.998867 oxterPap Then (la) aoeras = uprsoaea be = (72 80 (7) < 142061) = wana = 1-058 127, The highest value occurs at ¢ = 10, We want the value x > 10 such that Pr(10 < ¢ < x) = 0.1, 50 that the probability of the payment being higher is 0.1, since the payment would be higher only for times in the range 10<1< x, So wehave F(x) ~ F(10) = 0.1 BONO) B01) «9. 0.904897 eM = 2411s ‘AL2L.7115 years the present value random variable is #7118096 ~ [6577800] 12.8. ‘The present value of this insurance is Of 7 < 10, then is maxi 10 and monotonically decreases thereafer, Therefore, the highest 10% of values will be atained for 10 <7 <1, and we have to determine 1 Pas = €°, we must solve 10P35 ~ Pas = 0.1 06 g-006 9) 006-069.) 0.064 = ~ Infe“®S ~0.1) x Infer®6 0.1) 1335253 ‘Then the present value if death occurs at time 13,35253 is e 02681} (C). ‘This problem is unusual in that the solutions ate not in increasing order. Answer (E) would result if by mistake you discounted at 6 = 0.06 instead of at 6 = 0.1 12.9, First we calculate the default rate, Since the two investments have equal present value, 20H 1629 Li = sa Hh on 76 and we want ¢ sueh that e! SOK MLC Stay Mana 8th eon Copyrigl 2009 ASK EXERCISE SOLUTIONS FOR LESSON 12 as nos imi “) © 12.10. ‘The median occurs atthe point m such that e°" = 0.5, 50 m = 12 and the median of Z is, ~ 1 tates gta (1) i) 025 ‘Then the ratio of mean to median is 24? =[§$] 00) 12.11, A S-year deferred whole life insurance is worth the least if death occurs di worth the most at time 7" = 5 (that is the 100th percentile of the value), and then monotonically decreases ‘Therefore we want r such that Pr(S < T < 1) = 0.10, since this will be the time for which the probability the insurance is worth more is 0.10 (and the probability itis worth less is 0.90), making it he 90th percentile PAT s)-PAT <5) 12008) - (1 - 208) ig the deferral period. Ttis value, 0387428 _ a 1456 5 For t = 7.7486, the present value of the insurance is worth 1000e"°47489 = [62819] 12.12, The present value isa monotonically decreasing function of time, so the median is assumed atthe median time of death r = 30, in which case the endowment is paid after 20 years. ‘The answer is therefore «200 = Ce 1213, Z decreases in value as time increases, so we nced ¢ for which the probability of surviving past tis 0.6. Pa = Gq = 06 1-7 =06 1 = 60(0.4) = 24 ‘The value of Z is (40 24)e-09 = [4-819 12d, Z = is highest when 7(, time to death, fs lowes, We want such Pr(“47 < <9) = 0:8, of 8, s0-we want to solve,p, = 0.8 fort. Survival follows a Weill with In,py = ~0.0057 = 100.8 0s e=-Me 7 aos = 44628 1 = 6.68047 SOA MLC Say Mavacl-8ieiton Copyabe 2009 At 216, 12. INSURANCE PAYABLE AT MOMENT. 1218, Zishighest when 7(3) is lowest, We want such thatthe probability of living beyond tis 30%, orp: = 03. ‘wae For this generalized deMoivre, :px = ($3!) 40-1)? (x =03 oh 29.303 p03" = oorso7s 1= 40(1 ~ 0.018075) = 39.2770 ‘ThenZ 1216, Z = 0 for 72) 10, and ie then a decreasing poste faction, We want such that the probability of Z being greater than is 03,80 we need ¢ such that P[10-< 7x) <7} = 03. 10Ps ~ 1Px = 0.3 pvoo(- fetta) =e ops = 6°25 = 0.606531 1Px = 0.606531 ~ 0.3 = 0.306531 60005? = 0306531 0.306531 = 1.18244 8244 153782 So Z = e-00KIs.32 ie 12.47, Z= Te, We'll maximize for T by differentiating.IU's easier to differentiate if we log first, and logging doesn’t affect the maximum, Inz= Ine 005+ dinz 1 595 SE = 7 -005=0 1 t= 57 ‘The maximom is assumed at 20, and is 20e"#0510) 12.18, The survival function is calculated as follows: ee ( 3876}, 1 played no role in the solution. yas o.on( Pe jin 1.05, 0,011.05), as 95" 1) 1 069407(1.05' ~ 1) bs Now, Ps =exo(- [ tose) exp (-0.06940741.05'~ 1) SOA MLC Stely Maes etn Copy 20 AS EXERCISE SOLUTIONS FOR LESSON 12 217 We want to solve fort, Note thatthe present value is a monotonically decreasing function of survival time, so the 25th percemile of survis (the # for which the probability of living less than that mount of time is 25%, or for ‘which ,p2s = 0.75) corresponds to the 75th percentile ofthe present value of the insurance, e-Posssmnos- =0.069407(1.05' ~ 1 1.05" — 075 In0.75 = ~0.287682 = 4.144886 In5.144886 _ 1.638003 In1.05 ~~ 0.048791 3357241 ‘Then 0910357200 .. [26708 12.19, Z decreases in value as T(x) inereases, so we want t such that ,p, = Pe[ (2) > t] = 0.4, fec..onen=o.01 (# aL = ooi( ior Ps evo(-o01 -001 (L071) 1,07! = 1 + 100(0.06200) = 7.2 n72 29.176 ‘Then Z = 1,05 = [024087] 12.20. This problem is difficult because the present value jumps at time 10. Figure 12.3 shows the present value Figure 12,3: Present value of varying benefit insurance, for problem 12.20 of the insurance benefit, Z, as a function of 7, time at death. In the shaded ereas, however, there is a unique time at ‘which the present value is equal to the Z coordinate, This provides 2 easy possibilities, which we study first: 1. The median occurs atr 2 10 but rs at # point where the present value is greater than 100. ‘SOA MLC Stdy Mansa etn Cepye 209 ASKE 218 12, INSURANCE PAYABLE AT MOMENT OF DEATH: PERCBNTILES 2, ‘The median occurs at & 10 and fs ata point where the present value is less than its value right before ¢ = 10, 0, the present value is 100e"°4 = 67,032 which jumps to 134.064, ‘The value decreases to 100 when 100, or = 25In2 = 1732868 and Pr(10 < £5 25102) = ¢-&%00...g-00%25I62) — 0.818731 ~0,707107 = 0.111624, and 1-0.111624 = 0.888376, so this range represents percentiles 88,8 and higher, not the median, eliminating the first possibility. ‘The value decreases (0 67.032 when 200e"® = 67.032, or 004 0.33516 0.33516 7.3287 and Pe(T > 27.3287) 1.57893, so the median is greater than 27.3287, making the second possibil- ity correct, We just have to find the point ¢ > 27.3287 where Pr(T > 1) = 0.5, or 00" 9.5 f= 50In2 and at this point, the present valuc i 200e"®0460W2) = 2096-21? = 290 = 12.24, From figure 12.3, we se that Z is less than 80 in 2 intervals: 1. When 7 is less than 10 but greater than the ¢ such that 100e"° = 80. 2. When 7 is greater than the f such that 2002-8 = 80. For the first interval, the sinllest 7 is, 100¢°% = 80 oO 208, nos ‘O04 and the probability of ~(1n0.8)/0.04 < T'< 10 is is20008H008 _ 10002) . g05IN08 _ 402 = WOE -e*? = 0.894427 ~ 0.818731 = 0.07570 For the second interval, the smnallest 7 is and the probability of F > =I is POON - VF = 0.63246 So the total probability of being below 80 is 0.07570 + 0.63246 = [070818] SOA ML. Sty Manual eon Copyright i009 As EXERCISE SOLUTIONS FOR LESSON 12, 9 12.22, This exercise is more difficult that what you'd get on an exam. Z = ¢°€*?-094 for ¢ < 20, The quadratic in the exponent attains @ minimum at = ~b/2a = 0.06/2(0.04) = 0.75, gets back up toO at 1.5, and Keeps increasing afterwards, Thus Z grows in value starting at ¢ = 1.5 until f= 20, hen becomes 0 in value, We want the f such that Pi efi = 0.2 for that IF > 1.5, this means ang ¢ suc that Pr(t < 7) < 20) = 0:2 In other words, (rea) >) —Pi( 70) > 20) 70021 _ gr 9220) 9.2 90% = 0,670320+0:2 = wthe f such that & ‘The probability of survival 20 years is °%%2” = 0,670320. ‘Thus we w: 0.870320, if 1> 1.5. This implies Jn0,870320 = 0.138894 0.138894 oa 7 ome Thus Z = eboussueny-neees4srn) Sa) {1 hed been less than 1.5, te situation would be more complicated you would then have to find «such that a(t <1) + Pi < 70) < 20) = I~ 7 forthe 100x percentile SOA MEC Suly Mann efiton Capp 1008 ASHE 220 SOA MLC Sly Mares—20 aon Copyign S009 ASHE 12. INSURANCE PAYABLE AT MOMENT OF DEATH: PERCENTILES Lesson 13 Insurance Payable at End of Year: Moments Reading: Actuarial Mathematics 4.3 or Models for Quantifying Risk (2nd or 3rd edition) 5.1-5.2, 9.6 "This lesson discusses the moments of an insurance with a benefit payable at the end of the year of death, rather than atthe moment of death, If K is the curtate time of death, then the random variable for the present value of the benefit ofa whole life insurance is Z = v**, ‘The moments are calculated by sums instead of the integrals used for benefits at the moment of death EZ] Sua" Sppetoutt ro fa Siar? = Sayan? ro) 3 Ez") "The rule of moments applies, so that the second moment can be calculated by calculating the first moment at double the force of interest. ‘THeIAN symbols for insurance payable atthe end of the year of death are the same asthe symbols for insurance payable at dhe moment of death, except the bacs are removed. Thus, the actual present value of: ‘Whole life insurance is denoted by A,, which represents v<*". year term insurance is denoted by A, which represents **" if K = Plugging in x = 50, the point at which the error is found, this becomes 1_ft-as1.05"®) sal G08 1.72073) Originally «is 100 so that the APV of the 10 year insurance of 100,000 is 1 “Top gg (-724739100,000) ‘When wis changed to 120, he APV is Sap gg7721791100,000) ‘The difference in premiums is: ee It is negative, so af refund ofay4i2] is due. (B) 13.20, We have jaf = 18 and yg’ = (0.95)(0.10) = 0,095, Thus the probability of death in the first year from a randomly chosen individual is (0.25)(0.10) + (0.75)(0.05) ),0625 and the probability of death in the second year from a randomly chosen individual i (0.25)(0.18) + (0,75)(0.095) = 0.11625. Discounting at 2%: 0.0625 | 0.11625 _ Set roe = 0.173010 Multiplying by the 10,000 death benefit, the answer is [173010]. (C) 13.21. I's unusual that the enswer ranges don’t all have the same size, Figure 13.1 shows the benefit levels of the two policies. ‘The difference is 5 through age 70, 10 thereafter. So ‘we need 10 calculate Sago +5 ya Aso Say = 5(0.10248) = 0.5124 5 gqAan = 5 soko Avo ‘To ealeulate upByo, we break it down into 70850 any $0 that we can look it up inthe Ilustative Life Table. 4oBag = 29Es9 208s = (0:29374)(0.23087) = 0.06770 5 quo = 5(0.06770\0.51495) = 0.17431 Answer = 0.5124+0.17431 =[06867] (C) ‘SOA MLC Sty Mara—8h eon pyran 2005 a 240 _13, INSURANCE PAYABLE AT END OF YEAR: MOMENTS 30 0 % @ 0 #6 Figure 13.1: The two polices in problem 18.24 13.22, A pure endowment either pays or doesn’t, s0 Z is a Bernoulli-type random variable, with payments of v2 (probability 29p,) and 0 (probability 1 — 2op;). We use the Bernoulli shortcut (section 2.4) for variance. We have. BIZ] = napa Var(Z) = pat = 20p,)v" actZ) _ 04 20) FD = C1 = mare) = v0.35 Bay = pe = 0.35) 1 “0 ae 102186] (A) 13.23, From (i), isp = pl. Using Var(Z) = 15ps(1 ~ sp2)s™ = 0.065 BZ) = 0.065,sp,v"* 0.065 15 Pee, DIS == 0.684299 Pe = 0.975027 0315701 [03a974) —«B) & 13.24, sigs = 0.060.941) 1060.94") “ yy Tom SOA MLC Sty Nant eon Copyagh ques ast ERCISE SOLUTIONS FOR LESSON 13 24h aay) *\io7)\t = 94/107) 461538 st “\1or 1 =0.94/1. 07} 0.06 1.07 -0.94 oe ‘Var(Z) = 0.292826 — 0.461538? = [0.079808] nd of year 1, the present values yy = 0.943396, otherwise 1 the Bernoulli shortcut 1, IMs slo 0889996, 13.25, If death occurs at “The variance is calculated us ‘Var(Z) = (0.98\(0.02)(0.943396 ~ 0.889996) 1326. We calculate the first two moments, using geometric series, ElZl= >) Anat Foes “ats my = 025860 2 an 1.126708 ~ 0.253940" “w Var(2) Exercise look familiar? It isthe discrete version of exercise 11.6, page 189. The original exam question did not specify whether ihe insurance paid at the moment of death or atthe end ofthe year of death, so the CAS had to give credit for both A and C. 13.27, ‘The prescript of 2 means everything is done at twice the force of interest: “Assam = "Ass ~ E55 “Aes + Ess We need 2s. 8s = ) 1 i (i 08 = Fase = 0271862 Now, “Assi = "Ass ~ (BBs “Aes + Bess ‘SOA MUC Su Manual een Sepyien 2009 ASM 242 13, INSURANCE PAYABLE AT END OF YEAR: MOMENTS = 0.13067 ~ (0.271862)(0.23603) + 0.271862 = [033836] 45, =°0.019. From (ji) using the Bernoulli shortcut (there ate only two possible values for the present value of benefits, O and zl) S48 = 0.017689, Dividing he second expression by the fist, ‘Adding this to the expression from (i), 1019 +0.931 1 ta 950 = udp — | = (0052657) cw) 13.29, The probability of death in the frst year is 0.1; the uncondi Pasuss = 0.9)0.15) = 0.135. ‘The three possible benefits, and their probabilities, are: nal probability of death in the second year is Year | Probability | Benefit rt ot by 2 | 0135 | 10-m 3 | 0765 | 20-b ‘We calculate B[Z] and E{Z*). E{Z} = 0.1b, + 0.135(10 — by) + 0.765(20 — by) = 16.65 — 0.86, Biz?) = 0.164 + 0.135(10 ~ oy +0:765(20 — by 19,5 — 33.30, + 2 Var(2Z) = 319.5 ~ 33.3b1 + BF ~ (16.65 - 0.81)" = 42.2775 ~ 6.66b) +0.366? ‘This is minimized at 36%; = [9 413.30, This problem is best done with the compound variance formula which we leam about later, and which you may know from your probability course, See equation (54.1) on page 903, The formula states that if is the number of items, X is the amount of each item, and S isthe aggregate amount, or. = if, Xj, assuming all the items are independent Vor(S}) = UN] Var(X) + Var(N) ELX}? Here, Wis the number of friends buying a policy, and that is binomial with parameters n E{WY] = (100.1) = 1 and Var() = (10)(0.1)(0.9) = 0.9. X is the actuarial present value of insurance at (22), which we'll call Z, {10002} = 1000A22 = 71.35 Vax(10002) = 1,000,000( 4x2 ~ AZ) 000,000 (0.01587 ~ 0.07135) = 10,779 Var(S) = (1)(10,779) + 09.71.35) =[1586E] SOA MLC Sudy Monatsh eon Copa 2009 AK EXERCISE SOLUTIONS FOR LESSON 13 saeeeteLeees OM 1331, The payment is either 1 or O, and therefore Z is a Bernoulli variable with variance p(t ~ p), where p isthe probability of payment, Since p( - p) = 0.16, p = 08. p cannot equal 0.2 since there is atleast a 0.5 chance of paying (qx = 0.50). p = 24,80 5 * Peers = 08 08 get = Es A highly unusual CAS problem in that they provided answer choices rather than ranges. 13.32. A repeat of the last exercise, but now p(J ~ p) = 0.09 so p = 0.9. pis the probability of dying either in the first or the second year, and gy = 0.5, 30 -[ps] @) Pagar = 0.9 ~0.5 = 04 je a= 55 ©) 13.33, Let Z be he present valu, BIZ) = 180.086, B[Z"] = 0.1005 + (0:90)(0.20)(100 — by 800 ~ 3604 + 0.2863 1800 ~ 360) + 0.280} ~ (18 ~0.088,)? = 0273663 ~ 33.120, + 1476 Var(Z) “This is minimized at 5342, = [BOS5E3} () 13.34, There are 3 possibilities: death in the first year, death in the second year, and death thereafter. The payment for death thereafter is 0, 30 we ean ignore it when calculating fist and second moments, The probability of death in the first year is 200 78,900 64 0.021546, and the probability of death in the second year is 7,200 75,100 78500 ‘The present value ofthe payment for death inthe fist year is 12, = 9.52381 and the present valuc ofthe payment for deat in the second year is 22, = 9.07030. Therefore the moments ac: E(@) = 0.021546(9.52381) + 0,026616(9.07030) = 0.44662 E(Z?) = 0,21546(9.523817) + 0.026616(9.07030") = 4.14401 Var(Z) = 4.14401 ~ 0.44662? = [BBA] D) 13.35, “Polly discrete” will be discussed in lesson 22, but it just means that benefits are payable at the end of the year of death and annual premiums are payable at the beginning of exch year. ‘They give you the interest rate in a weird manner, by telling you the lowest premium such that there is no chance of a loss. Since the present value of the death benefit for death in the first year is v, it follows that v = 0.95. ‘The moments of Z are: ate 0.026616. 1s(0.95) + Press 0.95") + 2Px(0 25(0.95) + 0,15(0.9025) = 0.372875 .25(0.95%) + 0.15(0.9025%) = 0.347801 347801 ~ 0.372875" ©) SOK MLC Sy Manus 80 eon Copyabt 2009 AS 244 ___13. INSURANCE PAYABLE AT END OF YEAR: MOMENTS 13.36, Let Z be the present value random variable for insurance of 1 for the combined group, 2° the present value random variable for insurance of 1 for smokers, ZS the present value random variable for insurance of 1 for non-smokers. Moments for a mixture (which here is 20% smokers, 80% non-smokers) are calculated using the proportions of the mixture, even though the variance of a mixture is not the weighted sum of the individual variances. O01 |, (0.99%0.02) 28 = SF CENOSD o270559 EI) = OE 4 CSO) - oorasssa pz8H) a 202 , 9800 - oo4sorse Bash = PO 1 C800) - oconr0875 BZ] = 0.30.027055) + 0.2(0.0450338) = 00306515 EZ") = 0.8(0.0245834) + 0,2(0.0410875) = 0.0278842 Var(2) = 0.027884 ~0,030651? = 0.02694472 For 1000 of insurance on 100 lives, the mean is (100)(1000)(0,0306515) = 3065.15 and the variance is (100y(10007)(0.02694472) = 2,694,472, ‘The fund needed is 3055,148 + 1,645 2,694,470 = [576839] 13.37, According (othe Illustrative Life'Table, 1000A¢s = 439.80 and 1000*Ags = 236.03. Therefore, the variance of an insurance of Lis 0.23603 — 0.43980" = 0.042606 For 1000 lives and insurance of 10, the mean is (1000)(10)(0.43980) = 4398 and the variance is (1000)(107)(0.042606) = 4260.6, ‘The standard normal dssbution has its 8th percentile at 0.842. "Therefore, the required fund is 4398 + 0,842 V4260 = [4483] (B) 13.38, This question seems to belong on what was then Course 2 (now Course CAS 2/SOA FM) rather than Course 3, but is included here for completeness, ‘The present value of the medical payments, since they're made atthe end of the year, is somo ig = 8B SS ~ fost (ti TOs 10,000) -4 Ts au 19588) = 201,865.56 1s ‘The annuity certain forthe indemnity payments is arg = 452% = 10,379658 where v = zz. Their present value is 274,000 — 201,865.56 = 72,134.44, Therefore the payments P must be 72/44 = 35.60 o SOA MLC Sty Mant —Bh eion Copyright 62009 ASM Lesson 14 Insurance Payable at End of Year: Recursive Formulas, Varying Insurances Reading: Actuarial Mathematics 4.3, or Models for Quantifying Risk (2nd or 3rd edition) 5.1, 9.6 14.1 Recursive Formulas Expected values of insurance on (x) can be related to expected values of insurance on (x+ 1) in a steaightforward manner, This enables building insurance function tables recursively by starting atthe end of the table and working backwards. You should not memorize these formulas by rote. You should understand them and be able to reconstruct them. ‘All of the following relates (o insurance payable atthe end of the year of death, Lot’s start with whole life. Whole life on (2) consists of term insurance for | year followed by whole life: on (x+ 1) discounted back one year. In other words Ar = Vet PxAes a4.) "The recursion atthe end of the table, x-+ 1 = «2, will have gu-t = ly Pact * 0, 80 Ayer = v1) + ¥O)AL, Now for endowments. An n-year endowment on (x) consists of term insurance for | year followed by ann ~ 1 ‘year endowment on (x + 1) discounted back one year. In other words Aum = V2 + YPeAgiian, 4.2) At age x = 1, the recursion will be 4, Lert) = Geol + WP sen since « payment wll definitely be made athe end 6 the yea Porn: Jear tam insurance on (1), We hae 1 year of term insranceon (x which has APY vg plus an (n—1)-year term insurance discounted back I year, or mn 0s + PAs eT ‘The recursion at x-+1~ 1 will have Axyy_t-n = Yaseen» BECAUSE Axyqm = 0. ‘An wear deferred insurance on (3) isan (x ~ I)-doferred insurance on (x + 1) discounted back one year, or 4 Ar = WPapiAest "The formula changes to regular insurance when the deferral period ends, 245 SOA MLCSray Merci eos Cepyngh ©2009 ast 246 ___4 INSURANCE PAYABLE AT END OF YRAR: RECURSIONS, VARYING Exameit J4A. You are given: (DZ isthe present value randoun variable for 10-ycer term insurance of 1 on (80) payable at the end of the year of death, Gi) Z is the present value random variable for a 9-year term insurance of 1 on (81) payable at the end of the year of death. Gil) geo wt ) FIZ) =05 (vi) Var(@) = 0.2 Calculate Var(Z). “Axswen: All ho recursion formulas apply equally well whea the force of interest is doubled, so wecan use them to ot second moments as wel as fist moments, and then to gel variances, So let's Gist get "ldhaq. "Ado = Var(Za) + BIAI = 02+ 08? = 045 ‘Now let’s use recursion to get first and second moments of Zp. Agosto, = 400 + MPeoAdh 9) 0.1 O9Ady Os aa Adin = 22 = 046667 9. io gan + ¥ Pro “An OL 9) 7Ad 9 “Asoo oe 1,042 1.04? ayt (0.45)(1.047) 0,1 0.38672 iy = LEHI =04 , O36 gary Var(Za) = 0.42969 - 0.46667" o 14.2 Increasing and Decreasing Insurance ‘An increasing insurance payable atthe end of the year of death pays 1 if death occurs in year 1,2 if death occurs in year 2, ec. An n-year decreasing term insurance pays 1 if death occurs in year 1, n~ 1 if death occurs in year 2, ste, The symbols forthe sctoaril present values are the same as forthe functions paying atthe endl ofthe year of death excep thee is no bar onthe A, So for an 1-year increasing term insurance, the symbol would be (zm and for an 1-year decreasing term insurance, the symbol would be (DA)! n. ‘The sum of an n-year increasing term insurance and an n-year decreasing insurance is an n-year term insurance for n 4 L, asi clear algebraically and from figure 14.1! However, hero ate other relationships as well. We can split the inoreasng term insurance vertically int a series of deferred one-year te ig amounts, ‘or horizontally imo a series of deferred term insurances of {, as shown in figure 14.2. From figure 14.2, we see the formula (in = Yk a though the figures use (A) and (DA) eymbol, the equation i true forthe present value random variables for increasing, decreasing, ad ‘en inserances ot jut for thelr expected vals. Copyiita 2079 ASAE 14.2, INCREASING AND DECREASING INSURANCE. 247 4 CAS, 2 (DA)k3 ' | tT 2 3 4 5 Figure 14.1: (14).5, + (DA) = GAL Posie ge un 1 2 3 4 = § (a) Increasing insurance assum of I-year term insur- _&) Increasing insurance es sum of deferred term n= aces surances of 1 Figure 14.2: Decomposition of increasing insurance into term insurances From figure 14.2b, we see the formula (Adin = 9) enAlacrn a Figure 14.2a doesn’t directly provide a recursive formula in terms of insurances and inereasing insurances, but figure 14,2b does. If you ignore the dotted lines, the upper area is an increasing insurance starting one year later, and the lower area is a level insurance. This would work even if the increasing insurance was whole life rather than term, The recursive formula is (Aim = Azm + PsA) san ‘lg ean be further broken down in terms of Alas sing the recursive formula fr term insurance ‘Similar remarks apply o decreasing insurances. Figure 143 shows the spl vetaly into a seis of varying one-year erm insurances Here we get. ecursive equation by removing the dotd lines. The area to the right is & decreasing insurance for one yea less andthe area the lef ea one-year term insurance, 0 (DAlrn = nan + yPDA chen SOA MLC Seay Namal eon epyah eaton Aste 248 14, INSURANCE PAYABLE AT END OF YEAR: RECURSIONS, VARYING See 5 4 L 4 3 - 3 faba : Ada 2 2 DA) taal 1p 1 L : aa 1 2 3 4 5 1 2 3 4 5 (a) Decreasing nsrnce assum of defered I-yer_@) Dereatnginuance a sm of tr isuranees tem insurances on Figure 14.3: Decomposition of decreasing insurance into term insurances In figure 14.3b, the decreasing insurance is expressed as 2 sum of term insurances of varying durations, Removing the dotted lines gets a useful formula, although not quite a recursion moving ahead by ages. The area on the bottom is an n-year term life insurance, and the area on the top is an immediate decreasing term insurance for n ~ 1 years: (A)em = Abn + DA), Exaneur 148 You are given: @ (Aw = 6 Gi) UA) = Gili) Aga Gv) UA) = 2! Caleulate Aso ‘Answer: This isa slightly different twist than the above f6rmulas. As we see in figure 14.4, we can decompose the increasing insurance at (40) into an increasing 10 year term insurance, an increasing insurance at (50), and 10 units cof whole life insurance at (50). With discounting: (CA)qo = AY soz0 + Asoaty((UA)s0 + 1040) 64 = 2,5 +0.6(4.5 + 10As0) 64-25 ~2.] = 6Aso Aso = [2 a SOA MLC Study Manust—B0 con ‘Copy 2009 ASM EXERCISES FOR LESSON 14 249 14] 12 10] 8 6 ee 7 ae aes Ty | WA)doro| che tants Ls 44 46 48 Figure 14.4: Decomposition of (7)4a in example 148 Exercises 14.1, [4-S86:15] You are given: @ i= 002 (i) psp = 0.98 Gil) Ag — As = 0.004 iv) 7As ~7Aso = 0.005 ‘Let Z be the random variable representing the present value of a whole life insurance of 1 with death benefit payable al the end of the year of death. Caleulate Var(Z) for x = 51. (A) 0.055 (B) 0.060 (©) 0.065 (©) 0.260 (B) 0.265, 14.2, [3-801:34] Leo, age 63, considers the purchase of a single premium whole life insurance of 10,000 with death benefit payable at the end of the year of death. ‘The company calculates benefit premiums using: ) mortality based on the Ilustrative Life Table, Gi) i= 005 ‘The company ealoulates contract premiums at 112% of benefit premiums, ‘The single contract premium at age 63 is 5233, Lee decides to delay the purchase for two years and invests the 5233. Calculate the minimum annual rate of return that the investment must earn to accumulate to an amount equal to the single contract premium at age 65. (A) 0.030 (B) 0.035 (©) 0.040 (D) 0.045 (B) 0.050 ‘SOA MLC Sty Mansa eion Exercises continue on the next page. ny 200 ASM 280 14, INSURANCE PAYABLE AT END OF YEAR: RECURSIONS, VARYING 143. You are given: @ Aw =03 i) Aggan = 045 Gil) 29p40 = 0.9 Gv) §=0.04 Caleulate Ago. 144, You are given: (Fora standard lite, Ads = 0.15. Gi) A standard life has mortality rate q4s Gi) v= 0.95. ‘Due to an extra hazard at age 45, a certain life has gas = 0.02, but has standard mortality at all other ages. Caleutate A,r for this ie, 14.58. [41786232] You are given: @ Aen =u Gi) Aum =y Gil) Arn = Determine the value of Ay. (A) (2p tue @) (-2utye © (tay-uz () (+2u~ye ® +2u-y 146, [150-$87:27} Fora select and ultimate mortality table with a one-year select period, guy = Determine A, ~ Avg, (A) 2A (Ata) ®B) Aggn( - Axe) (© Agyndl = Aton) @) 054m Ae) ® 054yyn(l - Ay) 5a. 14,7. (150-888: (eo = 0.58896 Gi) Age = 0.9506 (ii) Ag = 0.60122 5] You are given: Cateulate 60. (A) 0.017 ®) 0018 © 0021 D) 0.032 (©) 0.033 SOA MC Sudy Manoal—th eon Brerczes conte onthe nex page Copyright 209 AS \ EXERCISES FOR LESSON 14 251 14,8, [150-F88:1] A whole life insurance issued to (25) provides the following benefits (1) the death benefit, payable at the end of the year of death, is equal to 20,000 up to age 65 and 10,000 thereafter; and (2) the net single premium is refunded at age 65 if the insured is still alive. You are given: (il) Ass Gi) ops (vy) 202 Calculate the net single premium for this insurance. (a) 2,000 B) 2,400 © 3,000 ©) 4,000 ®) 4,800 14.9, [150-589:27] You are given: (i) At - Ax = 0.015 Gi) i= 006 Gi) r= 0.05 Caloulate Ar + Asst (A) 0.60, (B) 0.86. (C) 1.18 (D) 1.30 (B) 1.56 14,10, [CAS4-F92:14] (2 points) You are given the following: @ 4p=025 Gi) Assis = 040 Gi) Aggy = 0.50 Determine Ali. (A) Less than 0.050 (B) Atleast 0.050, but less than 0.060 (©) Atleast 0.060, bat less than 0.070 ©) Atleast 0,070, but less than 0.080 (©) Atleast 0.080 14.11. (CASA @ Ap= 0.632 Gi) Ag = 0.644 Gil) 1=3% 5:21] (2 points) You are given: Calculate ga, (A) Less than 0.013 (B) Atleast 0.013, but less than 0.015 (©) Atleast 0.015, but less than 0.017 (D) Atleast 0.017, but less than 0.019 ©) Atleast 0.019 xercses continue onthe net page ‘Copyeh @200) ASM 252 ____14, INSURANCE PAYABLE AT END OF YEAR: RECURSIONS, VARYING 14.12, [CAS4A-#96:12] (2 points) You are given that: @ Aig =0.30 (i) qx = 0.005 Gil) 7= 0.07 Determine Az (A) Less than 0.30 (B) Atleast 0.30, but less than 0.31 (©) Atleast 0.31, but less than 0.32 (D) Atteast 0.32, but less than 0.33 @) Atleast0.33 14.13. [150-789:9] An increasing whole life insurance pays k-+ I at the end of year k+ 1 if (80) dies in year k-+ 1, @ v=0925 (Gi) ‘The net single premium for this insurance is 4 if geo = 0.1. is the net single premium for this insurance if qgy = 0.2 and q, is unchanged for all other ages, Cateutate P. (A) 340 @B) 3.66 © 395 ) 3.87 (&) 3.94 14.14, [SOA3-I03:10] For a sequence, u(l) is defined by the following recursion formula GE) = ak) + BCR) x uke ~ 1) for k= 1,2,3, i) a) = (24) 0 =f) (i) Bk) ra (iii) u(70) = 1.0 Which of the following is equal to u(40)? (A) Aw B) Avo © Awa ©) Adan ®) Aesth 14.15, [SOA3-03:22] For a whole life insurance of 1 on (41) with death benefit payable atthe end of the year of death, you are given: @ i=005 Gi) py = 0.9972 Gil) An Aw (iv) Ag — Caleutate Var(2). (A) 0.023 (B) 0.024 © 0.025 (©) 0.026 ® 0.027 SOA MLC Study Monua-—Bih don recites conto onthe nex page Srp 2009 ASM EXERCISES FOR LESSON 14 253 14.16, (3-F00:28) A decreasing term insurance on (80) pays (20~k) atthe end of the year of death if (80) dies in year + 1, for k= 0,1,2,...519. You are given: i) 1= 0.06 (ii). Fora certain mortality table with geo = 02, the single benefit premium for this insurance is 13 (Gi) For this same mortality table, except that gao = 01, the single benefit premium for this insurance is P. Calculate P. (A td @) 14 ous ©) 120 ) 123 14,17, (150-F96:18] You are given: (iv) GA)ss = 3.7100 Calculate (14)x6 (A) 381 B) 3.88 © 394 ©) 4.01 ® 4.08 14,18, You are given: @ DA)zy = 0.08. i) DAdgtozra = 8 Calculate (DA) am. 14.419. You are given: (IA), = 13.00 (WA)as = 1525 Aus = 04 Ags) = 0.76 196 105 Calculate (DA). 14.20, [4-886:32} Which of the following are equivalent to (DA)? L Sib Abaera TL Sfp = Oadby TM, nvga + vp(DA) ta z=n (A) Tand I only (B) Land I only (©) Mand IM only (©) Land Or (E) The correct answer is not given by (A), (B), (C), or D). ‘SOA MLC Sty Manta eon ‘Bceelses continue'on the next page Copyright 200 ASM 254 14, INSURANCE PAYABLE AT END OF YEAR: RECURSIONS, VARYING ity Ae Abn 14.21, [4-F86:13] Simplity Wena A) Ar ®B) Ant O Am ©) Ain ® Ach 14.22, (Based on 180-F87:4] Two insurance policies are purchased by (40). ‘The net single premium for each policy is 100. One policy is a discrete, 20-year term insurance increasing annually. ‘The other is a discrete 20-year term insurance decreasing annually. ‘You are given: @ UA = 4.1736 Gi) Agg = 0.1614 Gi) aa(AYwo = 1.4865 Gv) yA = 0.05699 Calculate the number of years during which the death benefit ofthe increasing policy is less than the death benefit of the decreasing policy. an @) 12 OB @) 14 @ 15 14.23, [150-F88:11) You are given: @ 1000(7A)s9 = 4,996.75 Gi 10004dn = 5:58 (ii) 10004 = 249.05 (iv) (= 0.06 Calculate 1000(4)s1. (A) 5,042 (B) 5073 © 5270 (D) 5,540 ® 557 14.24, [1504°89:14) You are given: [Aden 0.250 0.385, os7t Calculate (DA)¢s.5). (A) 0.227 (B) 0.369 (C) 0.394 @) 0.413 (E) 0.580 14.25, | [150-82-94:8] Which of the following represent (DA)3)? 1. EB(30- ta psaeae TS Po(30~ WA, — Aggy ~ tor BcAratet) ML. SPoAnso=9 (A) Land Tonly (B) Tand UW only (©) Hand If only (©) 1, Wand mr (B) The correct answer is not given by (A), (B), (C), or D). SON MLC Sty Mani 8th ion Exercises conte onthe nest page ‘Copyshe ©2000 ASN EXERCISES FOR LESSON 14 - 285 14.26, [CAS4A-899:15] (2 points) Which ofthe following is NOT a correct expression for the actuarial present value of an annually decreasing 3-year term insurance with benefit payable at the end ofthe year of death? A) Wy B)3qiAr+ 2mAet mAs © Aim + Ata +4in ©) 4Ala~ CA) B) 392+ MDA)chi a 14.27, [150-$91:24] A special n-year endowment insurance on (2) pays pure endowment of 1000 at the end of n years and pays only the net single premium at the end ofthe year of death if death occurs during the n-year period. ‘The net single premium fortis insurance i 600. “The net single premium for an n-year endowment insurance of 1000 with death benefits payable atthe end of the year of death of (2) is 800, Calculate the nt single premium for ann-year pure endowment of 1000 on (2). (A) 100 B) 200 © 300 ) 400 ®) 500 14.28, [150-83-96:6] For « modified ten-year pure endowment of 1000 on (35), you are given: © Assim = 0.57 (ii) This modified pure endowment includes a death benefit payable at the end ofthe year of death. (ii) 2 isthe present value random vaiable ifthe death benefit is 1000;0Bs. (iv) Za is the presen value random variable ifthe death benefit is 750)0Bss- (¥) Za is the present value random variable if the death benefit is S00,0Ess- (vi) BEI = 1.005 Caleulate BIZ} (A) 550 (B) 555 © 560 @) 565 &) 570 Additional released exam questions: SOA M-S05:15,35, CAS3-F06:34, SOA M-FO6:35, SOA MLC-S07:10 Solutions 14.1. We know that Aso = ¥gs0 + ¥psoAst an , 098 To2 * 102"! 0.02 207 + 0,004 00 * 0% As = 0.004 602 ‘and we repeat this with twice the foree of interest to derive “As. When the force of interest is doubled, v is squared. 002 , 0.98 44s, -0.005 = 2% , 998 x, *As) ~ 0.005 = 5 + opr "Ast S08 MLC Sy Moats eon Copytigh 2009 a 256 14, INSURANCE PAYABLE AT END OF YEAR: RECURSIONS, VARYING. om) ~ 1.02? 7As, = 0.417252 ‘Var(Z) = 0.417252 ~ 0,602? = [0084848] (A) a 1-28) = 88 as 14,2, The benefit premium is $233 Fy =o, 10,0004¢ 80 Ags = 0.467232, We use the equation Aas = ¥96s + ¥" Pe G04 + VPs Aes .01952, ley = 7,823,879, los = 7,533,984 to obtain: os _ 7,533,984 and the values of go = 0.01788, goa = 962947 PO Ty ~ 7,823,879 _ 0.01788 (1 ~ 0.01788)(0.01952) 0.467032 = SF oy (0.467232 = 0.017029 + 0.017389 + 0.873422Aqs __ 0.467232 ~ 0.017029 - 0.017389 * 0.873422 ‘The contract premium at 65 is 1.12(10,000)(0.49554) = 5550. ‘The earnings rate needed is (5550/5233)!/?— 1 = (C030), (ay 14 0.49554 Aes Ao = Advan + 20Bs0dco on ~ 2080 + a9Barden Ew 14.4, We'll use primes for the life with extra hazard, Asam = 445 + Vas Adan (0.01(0.95) + (0.99)(0.95)Adg ‘Ales = 0.149389 (Ags.rm) = (0.02)(0.95) + (0.98)(0.95)(0.149389) OaS808T] 145. Ar= Azim +nExAsin ng = Azm~ Abn = Ar=yt(u~yye. = yl -2) tur (A) ~y SOA MLC Sty Manel fon Copia ©2009 ashe EXERCISE SOLUTIONS FOR LESSON 14 14.6. By the recursive relationship between A, and As. ' ‘Aus = vou + YPLadas Aqmn +901 — guys = Aggn + Yan ~ Acker gs + VPxAxet 2Atya +L ~g)Axer Ady + VAs ~ 2Agy TIA So te difference is | Aa Ava = Adan ~AdenAna Ande AG] 14.7. We use the recursive formula and the fact that Agah = »Poo- Aca = ¥a60+ Asad Aot : 0.58896 = vgn + (0.9506\0.60122) geo = 0.58896 ~ (0.9506)(0.60122) = 0.01744 ps = Aga = 0.9506 | y= vpeo + Veo = 0.9506 + 0.01744 = 0.96804 i — ¥960 _ 0.01744 ! geo = “He =F oerpa = NSE] ®) ! 148. Ir we let A be the not single premium, then we express it in terms ofthe three benefits: 20,000 for life, pure ‘endowment for net single premium at 65, and ~10,000 deferred insurance, ‘A = 20,000Aa5 + sop230"A ~ 10,000P25¥" Aes = 2000 + 0.16 ~ 1600(0.2) | 1680 _ 5 1680 fanaa] (4) 149, Wehave to caeulate Aas. An Jon = 0015 = 995 + FE bes (1.06 - 0.95)Ag1 = 1,06(0.015) + 0.05 0.0659 Asa oir 7 099909 18318] (C) Apt Aust = 2Asu1 ~ 0.015 SOA MLC Sealy Magali on Coppa C2008 ASK 14, INSURANCE PAYABLE AT END OF YEAR: RECURSIONS, VARYING 14.10, 0.25 = Alyy + A,.rh (040) = 0.5 —Agth + Agih(0.40) 025 Aah = oe ' 025 1 pees Aum = 0.50~ 52 = 75 = [OHSS] 14.41. 0s * 1 (1.03)(0.632) = 0,644 + gx(1 - 0.644) 0.00696 oT 06d eo Mt. 130 oe 107" 1.87 0,30(1.07) = 0,005 + 0.99542; ).30(1.07) ~ 0.005 fy = S200 = 0005 = BFE] © 14.13, If we let A be the value of insurance of k + 1 at the end of the year k + 1 starting at k 4 = (0.1(0.925) + (0.9)(0.925)4 P = (0.2X(0.925) + (0.8)(0.925)A (4-0. ons) = 0.185 + com (98 - FSS] 14.14. Its easier to see what’s going on if u(k — 1) is expressed in terms of w(k): ull) ~ a(t) Be) = + act Pes C4 D/p-1 2 pis + T+ = MVP + Vit Wk 1 So we start at 70 with 1, and the first summand discounts this back a year. The second summand adds a death benefit payable at the end of the year. This process is repeated, so at 40, we have the discounted value of I at 70 plus the discounted value of | payable at the end of the year of death for death between 40 and 70. In other words, (C). SOA MLC Step Nans-—th ton Copyright 000 Ast EXERCISE SOLUTIONS FOR LESSON 14 : 289 | 14.15, We will show how to calculate Aq), and the exact same formulas can be used with double the force of interest for *A4y, We know from recursion that Aap = vqao + »PaoA41, 80 ‘Aa ~ 0.00822 = vge0 + vpuoAat | Aai(1 = vpqo) = v4e0 + 0.00822 vga0+0.00822 _ 0.010887 { so + 0.0082 _ OOUSS? _ 0.21648 Aa 1 = vpao 0.050286 oe i a4 ¥2qu0 +0.00433 _ 0.006870 a 0.095510 eee Var(Z) = 0.071926 — 0.216496 = [61025086] (C) 14.16, If we let A be the value of insurance of 20 ~ k at the end of the year k + 1 starting at k = 1, 2(20) | 08 1.08 * 1.06 13(1,06) ~ 0,2(20) _ 13(1.06) ~ 4 08 08 0.120) | 0.94 105 * 1.06 2 09 4 = {06 os (s - ie) 12.2665] (E) 14.17. With the one-age difference (35 to 36), this problem screams out for a recursive calculation, (J4):s consists of a whole life insurance plus an increasing insurance starting at age 36, discounted back one year. ‘The interest discount factor is v = Assn = 0.9434. So (IA)ss = Ass + vpss(IA)se 3.7100 = 0.1300 + (0.9434)(0.9964)(IA)s6 3.7100 ~ 0.1300 _ pg Ads = G 5340996 ~ io | 14.18, DAL zy = DAVea + BAL + Apd(DA damm ten ~ Aud) = 095 ~0.9 = 0.05 08 + 18(0.05) + 0.9(8) = [8:08 14.9, We need 5 units in year 1,4 in year 2, ..., in year 5. We can get this from (DA), = Aga) ~ WA)e31 ~ 6 (SEs) since subtracting 1, 2, 3, 4, 5 units from 6 gets us 5, 4, 3, 2, 1 units, and we eliminate the pare endowment. In this . Fy = 228 - 0.75218: SEs = Tipps = 0752185 and (A)Lg = (A)e ~ sEx(UA)evs + 5Asss) ‘SOA MLC Sty Mauston {Cop ©2009 ASM 260 14, INSURANCE PAYABLE AT END OF YEAR: RECURSIONS, VARYING = 13.00 -0.752185(15.25 + 504) = 0024807 So (DA). 3, = 6(0.76) ~ 0.024807 ~ 6(0.752185) = [Oi 83] 14.20, Ts incorrect, The correct formula adds up term insurances for periods going from I ton, not from 2 to nel Tis correct, and sums up deferred one-year term insurances for amounts starting at n and decreasing to 1. TIL is the comect recursive formula; the value of this year's benefit isn, and survivors get a decreasing term insorance starting one year later fora period one year shorter. («© 14.21. The numerator represents the expected value ofan increasing insurance withthe frst year removed; this is Agh(TA)get + Ayyt} The denominator i the term in parentheses. (E) 14.22, We calculate the net single premiums for the inereasing and decreasing poicis. (A)foza = (A)io ~ 29(JA)ao 20201440 = 4.1736 — 1.4865 — 20(0.05699) = 1.5473 (DAYiom = 21(A4a ~ aso) ~ (LA) 20 = 21(0.1614 ~ 0.05699) ~ 1.5473 = 0.64531 So the number of units is JM; = 64.6287 for the increasing term and 9%; = 154.9643 forthe dear We need 64.6287x = 154.9643021 ~ x= 48195 Notice that we hd to use 21 ~ x, not 20 ~ x. For example, inthe fist year the decreasing insurance has a death benefit of 20 times 154.9643. ‘The answer is thatthe decreasing insurance is higher up to and including the 14th year [el] (D) 14.23, Since 10004Jon) = vaso = 5.58 and v= rhg, it follows that qsp = 0.059148, vpsp = 0.937816. So (ZA)30 = v9s0 + vpso((IA)si + Asi) 4.99675 = 0.00558 + 0.937816((IA)si + 0.24905) 4.99675 ~ 0,00558 iy = 122675 — 0.00558 _ 0.24905 As 0.937816 oa 1000(74)s, = [50707] — (B) 5.07307 14.24. (DAYis31 = 64d — CMe, = 6(0.133) -0.385 = [0413] «Dy 14,25. I provides deferred term insurances of decreasing amounts from 30 to 1, and is therefore correct. Tshould subtract Ai.q instead of Ay. It would then provide deferred term insurances of decreasing amounts from 30 t0 1 IM provides term insurances for periods of 30, 29, ... 1, and is therefore correct. @) SOA MLC Sty Mapu eon Cepia 2009 Asha EXERCISE SOLUTIONS FOR LESSON 14 261 (DA), Teeter see Figure 14.5: (IA) La + (DAL = 44x 14.26. Even though (D) has subscripts of 4's instead of 3's (unlike figure 14.1), itis still correct, af is seen in figure 14.5. However, (B) is missing ps and should be 3v9, + vps(DA),t1.01 14.27. We are given two equations in two unknowns: 600ANm + 1000A,an = 600 100A! m + 10004;m = 800 Subtracting one from the other yields 400A yim = 200 © 10004, = 800 ~ 1000Azm 14.28. From (vi) we have 1000 19835 Ads rai +1000 sos 750 oss Adem + 1000 10Es 005 (1.005)(750)Ag,.rq 03s + 1000(1.005)i0B ss = 1000445. ri0kss + 10000 ss (1000 ~ 1.0051750))A3sn9 = 5 Lie 3 = Asso = 1000 = 1.005(750) ~ Cd and feom 1 Fas = 0.57 ~ 0.020305 = 0.549695 35528] (B) So the answer is $00{0.020305)(0.549695) + 1000(0.549695) SOA MLC sud Manuel—8ih etoe Copyright 008 AS 262 SOA MLC Stidy Manan tion Cope 2009 Ah 14, INSURANCE PAYABLE AT END OF YEAR: RECURSIONS, VARYING Lesson 15 Relationships between Insurance Payable at Moment of Death and Payable at End of Year Reading: Actuarial Mathematics 4.4 or Models for Quantifying Risk 2nd or 3rd edition) 5.5, In practice, mortality rates are available only for integral ages. This only allows computation of insurance payable atthe end of the year. However, insurance pays at the moment of death, Hence a formula to go from the former to the latter is needed. Despite the practicality of this, this material is only tested by the SOA, not by the CAS, and ‘even the SOA hardly ever asks any questions on it ‘The formula must make an assumption about distribution of deaths within the year. The uniform distribution is the easiest to use. The formulas are then as.t) (as) and similarly for deferred insurance, For endowments, only the term part, not the pure endowment, is multiplied by is: Ken jAln + Aon For an insurance payable at the end of an mtly period, like at the end of the month of death—a truly unusual concept-—the factor would be fy. The symbol for an insurance payable atthe end of an mithly period is AY", so the equation is yee Ar ots Note that the tables you get on the exam have i/6 at 6%, 6 = 1 = d), and looking up the table, we see that at 6%, i/6 = 1.02971. , Recall that 24, and 7A, are insurances a twice the force of interest, so the former is equal to the later times 3, where the prseript 2 on i tnd é means to evaluate the function at twice th force of interest. Since dis the foree of interest, 75 = 26, Since 1 +i= €*, 21+, Therefore ‘Since this derivation is easy, you may prefer nct to bother memorizing this formula, ‘The formulas relating continuous and disorete insurances also work with varying benefits, if the variation only ‘occurs atthe ends of years. In particular, IA)en (DA), SOA MLC Study Mast eon 263, Copii 2009 ASME __15, INSURANCE: DISCRETE TO ( To relate (IA), to (1A),, however, an additional adjustment is needed to go from (JA), to (JA). ‘The latter pays a pro-rata portion ofthe year instead of the full amount, e.g, 3.5 if death occurs in the middle of the 4th year (after 3.5, year) instead of 4, Under uniform distribution of deaths, the amount not paid each year is (Diy. After factoring out f, this leads to the formula at the bottom of Actuarial Mathematics page 124: WAye= 5 se - Gj . 5a] ‘You'll have to decide whether it's worthwhile memorizing this for the rare.exam question on it, ot whether you'd rather calculate (Di)p if necessary. A similar formula for (BA), where the adjustment would be (Ti), could be written, bat the textbooks don't mention it, s0 I doubt you'd be asked about it ‘An exam question would want you to do more than multiply 4 by something, so this topic would be combined with something else. xan ISA. You are given: () Morality follows the Wustrative Lite Table. (ii) Deaths are uniformly distributed between integral ages Gi) #= 095 Calculate Ana. Answer: Since the interest rate isn't 6%, we can only use the ILT"s basic functions, We have ger = 0.00466 gaa = 0.00504 1ugan = (1 ~ 0.00466)(0.00504) = 0.00502 abe) = 1 ger ~ 449 = 1 - 0.00466 — 0.00502 = 0.99032. 1, = 0.00866 , 0.00502 ase 10) 0, Aah = 22202 ~ 9825 1.05? faa = tAba + Aed 00s = (int) 0.00899 .00899 + 0.89825 Exercises 151. Let Z be the presont-value random variable for a standard 30-year endowment insurance which pays a death benefit of 1000 at the moment of death, You are given: (® A special 30-year endowment insurance pays a death benefit of 1000 at the end of the year of death, ‘The pure endowment is 500. The actuarial present value of this endowment is 260, Gi) Another special 30-year endowment insurance pays a death benefit of 1000 at the end of the year of death, ‘The pure endowment is 750. The actuarial present value of this endowment is 320, Gii) Deaths are assumed to be uniformly distributed between integral ages. (iv) f= 005. Calculate B{Z). S04 MLC Stay Maca eon Beercises continue onthe nex page Cops ©2020 ASM EXERCISES FOR LESSON 15 265 15.2, You are given: @ GA), = 154 (iv) Deaths are uniformly distributed between integral ages. Calculate Ax. 153, You are given: (Mortality follows the Illustrative Life Table. Gi) Deaths ae uniformly distributed between integral ages. Gil) §= 006. Cateulate AS. 154, Zs the present-value random variable for a 10-year endowment for 1 on (38) with death benefits payable at the moment of death. ‘You are given: (Mortality follows the Illustrative Life Table. Gi) Deaths are uniformly distributed between integral ages. Git) i= 0.06 Calculate Var(Z). 45. [4-886:18) You are given that deaths are uniformly distributed over each year of age, CAs xpress (De = OAs in terms interes fneions “@ ®) © ©) ® 15.6, [150-F88:25] You are given: @ Ar=025 wv) ‘Assume deaths are uniformly distributed over each year of age Calculate 10004 an (A) 550 @) 551 (© 582 @) 553 (&) 554 0A MLC Stedy Mant eton _Bxerclesconunue on he nex page Copan 2009 ASM 26 ____15, INSURANCE: DISCRE TO CONTINUOUS 15.7, [150-S91:19} Which of the following are true? 1 (Aim = Dhg(10~ HwALn Th Ar=vge+Vpdse + PopsAnee MI, (7A), = £(/A), under the assumption of uniform distribution of deaths. (A) Land only @) Tana Ht only (© Mand only ) Two correct answer isnot given by (A), (8), (C), oe D). 158, [150-81-94:1] You ae given Deaths are untormly distibuted over each year of age (i) f= 0.10, Gi) 9, = 0.05, GY) devi = 0.08. Calculate An. (A) 0.103 (B) 0.108 (© oun D) 0.114 15.9 1. [180-81-94:25] You are given: ()5(40) = 0.500 Gi) (41) = 0.475 Gi) i= 0.06 Gv) Aq = 054 (0) Deaths are uniformly distributed over each year of age, Calculate Aco, (A) 0.483 ®) 0517 © 0523 ©) 0.531 15.10. [150-83-96:8] You are given: @ q=0.10 Gi) gas = 0.20 Gi) i= 012 Gv) Deaths are uniformly distributed over each year of age. Caloulate “Aly. (A) 0.206 (B) 0.209 ©) 0218 (D) 0.224 SOA MLC Study Manoa eon ‘Copyright ©2009 ASM () 1, Wana (&) 0.119 (B) 0.645 (©) 0232 [Bxerises continue on the nest page EXERCISES FOR LESSON 15 267 1511, (CAS4-82:21) The net single premium of a $10,000 20-year endowment insurance policy issued to a life age 45 is $5,880. “The insurance provides for the payment ofthe death benefit atthe end of the year of death, The effective annual rate of interest is 3%. Caloulate the net single premium for the same policy ifthe death benefit is payable at (A) Less than $5,910 (B) Atleast $5,910 but less than $5,940 (C) Atleast $5,940 but less than $5,970 (D) Atleast $5,970 (B) Cannot be determined from the information given the moment of death 15.12, [150-F96:3] You are given: 6s) = $e + vpaate+ DD Gi) (80) (ii) Deaths are uniformly distributed within each year of age. 01,2, ‘Which of the following expressions is equal to u(40)? (A) Aco, (B) Aswan, (©) Alvan ©) Adam (©) The correct answer is not given by (A), (B), (C), or (D). 15.13, [150-898:8] You are given: @ 9=0.20 Gi) ga = 0.40 i) f= 012 (iv) Deaths are uniformly distributed over each year of age. Calculate “Al (a) 0369 B) 0384 (© 0398 (p) 0.407 ©) 0414 ‘Additional released exam questions: CAS3-F06:35 Solutions 151. From (i) and (ji), 250 of pure endowment is worth 320~260 = 60, so the term insurance is worth 260~120 = 10.” At the moment of death itis worth (2P%](140) = 143.47. ‘The endowment insurance for 1000 is worth 143.47 + 4(60) = [S837 152. We apply the formula relating (ZA), to (1A), als=-(i-2)4) (024797 (15.2 ~ 0.5040664,) 15, ‘SOR MLC Sty Manual elon Cepyden 62009 ASB 268 15, INSURANCE: DISCRETE TO CONTINUOUS 15.3. The Illustrative Life Table gives i/i" = 1.02721, so We calculate i, and then plug into the formula AGP = 1.02721Ay = 1.02721(0.24905 154. We calculate first and second moments. For the first moment, we use values from the Illustrative Life Table for i/6 and rose. ! EZ) = glAse ~ 108 Aas) + 108s = 1.02971((0.14746)~(0539633(0.22892) + 053963 = 056427 In the following, 7i denotes i at twice the force of interest corresponding to i = 0.06; 2i = 1.06? ~ 1 = 0.1236, For the Second moment, we'll have to calculate °i/46 and E38. 3 BIZ?) = 5 (7Ase — Vopas "Aa) + ¥™ yop ae a Ose apn = vob = SSE = 030138 3 136, 36 Zin .06 ~ 196060 124 = L06060(0.04255 - Var(Z) = 0.31991 - 0.564277 = 1133(0.08306)) + 0.30133 = 0.31991 001509] 15.5. We expand the numerator, 186. 0.25 = Ala + (0.55 ~ Al.gq)0.40 0.03 = 0.64! ay Alay = 0.08 10004,-»m = 1000 (ote 05) + (0.55 ~ 0.05) in 03, - BGR] @ 15.7. Tis correct, since a decreasing insurance isa series of term insurances paying 10,9, Tis incorrect since v° should be v2 THis incorrect, since an addtional adjustment is needed for continuous increase in insurance, but would be correct ifthe left hand side were (1A), ® ‘SOR MLC Stay Maal dion Cops 2009 ASN EXERCISE SOLUTIONS FOR LESSON 15 Z 7 269 18.8, 1 = 208 , (0.95)(0.08) Aa=Ti9* Tio = 0.108266 ©) 15.9, We first calculate Aut = 0524420 ‘and then calculate Ago recursively. Ao = va90 + vPaoAa 0,500 ~ 0.475 3500 0.05 208 023) eon [osi7i69] (8) 15.10. We calculate the interest rate corresponding to double the force of interest. laze 2544 ‘We caleulate 2ALn using thi 0.10, (0.90,0.20) 2544 +l 54a te We calculate “A'.p based on uniform distribution of deaths. ap _ [0.2544 7 "ta = (eae aa) © 15.11, ‘This appears to be the rare case where (I) is the right answer. Without knowing mp4s, determine the vale ofthe pure endowment. Moreover, the problem does not even state that uniform strbution of deaths is assume 45412. Let's stat at 2—79, ‘Then u(79)'s second term is the discounted value of 1 at 80. This 1 is repeatedly discounted in the reoursve calculation, soit represents a pure endowment at 80. "The fict term is «payment off upon death. If benefits were paid atthe end ofthe yeas, the present value of the payment would bey, Ithey are pai atthe moment of deat, and deaths are uniformly istrbuted itis equivalent to pening a one-year continuous snuity-certin of 1, oc 2 = §. Equivalently, we know that we mukiply the payment Y by {to go from benefits pad atthe end of the yea to benefits pad atthe moment of death and = 4. Bier vay, jv conclude thatthe frst teem isa payment of 1 atthe moment of death, The recursion adds this in for every ee, s0 when we get back to 40, we have payment of 1 al the moment of death between ages 40 and 80 plus a pure endowment at age 80. The answer is (A) 15.13, We double the force of interest. The / to use is 1.12*~ 1 = 0.2544. Then 5 {= 1122309, The insurance payable at the end ofthe year of deaths an _ 02, (asyo) "Aly = Vg + VPsQusi = T5544 + asad so “Aly = 1.122399(0.362805) = [0:407211} (D) it is impossible to = In 1.2544 = 0.226657 and 0.362805, ‘SOA MIC Sty Manus)—8i eon oye 09 ASM 279 15, INSURANCE: DISCRETE TO CONTINUOUS SOA MLC Sts Sanuk eon Copy Gav? SMe Lesson 16 Annuities: Continuous, Expectation Reading: Actuarial Mathematics 5.2 ot Models for Quantifying Risk (2nd of 3rd edition) 6.1.3, 6.2.3, 6.3.3 ‘You are familiar with annuities-certain from the Financial Mathematics course, Life annuities pay only while a life is alive. ‘A continuous annuity pays a level amount continuously at @ rate of 1 per year. This is an abstraction which is useful for annuities with frequent payments, such as once a day. ‘AS with insurance, traditional actuarial notation is available only for the actuarial present value, not for the random variable itself, ‘The random variable for the present value is usually denoted in the textbooks by ¥, just as it was denoted by Z for insurance. Models for Quantifying Risk decorates ¥ and Z with the subscripts normally used for a and A, €.8., Yoo; is the random variable for the actuarial present value of a 10-year temporary life annuity on (20), ‘Standard types of annuities are: ‘Whole life annuity A continuous whole life annuity pays at a rate of 1 per annum until the death of the annuitant "Thus if the annuitant lived 7 years, the total payments would be 7’, where T does not have to be en integer, ‘The symbol for the actuarial present value of a continuous life annuity on (x) is ‘Temporary life annuity A continuous n-year temporary life annuity pays at arate of | per annum until the earliest ‘of death and time n. n is almost always an integer, although it doesn’t have to be. ‘The symbol for its actuarial present value is Gxm- Deferred life annuity A continuous n-year deferred life annuity pays at a rate of 1 per annum starting at time m ‘and continuing until the death of (x), If death occurs before time n, there ate no payments. The symbol for its actuarial present value is dy Deferred temporary life annuity An n-year deferred m-year temporary life annuity pays at arate of 1 per annum starting at time n and continuing to time n + m, assuming the annuitant is alive. ‘The symbol for its actuarial present Value is yps- n-year certain and life annuity A continuous n-year certain and life annuity pays at a rate of 1 per annum until the latest of death and time n, Thus the first n payments are certain. This annuity is a sum of an n-year annuity-certain and an n-year deferred life annuity. The symbol for its actuarial present value is 237: "The above descriptions are summarized in table 16.1 ‘An important formula from Financial Mathematics (easily derived by integration) is that for a 7-year annuity- certain, (16.1) ss, Let's discuss how to calculate ‘This formula will be used to calculate the actuarial present values of life annuit the actuarial present value for each type of annuity. SOA MLC Sty Maat eon an Copyright 200 ASM 22 16, ANNUITIES: CONTINUOUS, EXPECTATION Table 16.1: Actuarial notation for standard types of annuity Whole life annuity | 1_¢<7 a Temporary life] 1 ¢< min(T,n) c annuity Of > min(7,n) ial Deferred life 0 tsnort>T annuity L_netst Deferred 0 tsnort>T temporarylite | 1 neren+mandtsT annuity 0 T>ntm Certain andiife | 1 t max(Tn) = 16.1 Whole Life Annuity ‘A whole life annuity is an annvity-ceriain for T years. Equation (16.1) expresses it in terms of v7, a whole life insurance. Taking expected values of both sides, 06.2) Ay= 1-62, (16.3) ‘These formulas for going between whole life annuities and whole life insurances are very important for you to know. ‘They have interpretations. The second one says that the value of I right now is equal to the present value of 1 at the moment of death, plus the present value of interest on 1 between now and the moment of death, or a continuous annuity of the amount of interest (6) until death occurs, ‘The direct way of calculating the expected value is to integrate the value of the payments over the density Patlewe fun a Lf Op Prt dt (16.4) Io ‘This is not the normal way you would evaluate the expected value, Here's an artificial example of its use: Exaanut 164 A life age (2) is subject to a constant force of mortality of 0.01. This life receives an ann have determined that the present value of the annuity if the life lives 7" year is 10(1 — e~002"), Caloulate the actuarial present value of this annuity. You ‘Answar: Let ¥ be the present value random variable. From equation (164), {S0A MLC Say Mons ton Cops 2009 AS 16.1, WHOLE LIFB ANNUITY 273 (09a eaeslated from Beton {()a eloulated from current payment technique Figure 16.1: Comparison of 2 methods of calculating @ By plugging in ay, = 42% and integrating by parts, equation (16.4) is transformed into a [iva (0635) Ib ‘This is called the current payment technique. The value of the annuity is the integral of the valu of each payment () times the probability thatthe payment is made (,p,). This isthe formula you'll usually use for calculating as, assuming you can't use (16.2). ‘in fact, the current payment technique is analogous to the similar technique for calculating life expectancy, discussed on page 79 and illustrated in figure 6.3. Actuarial Mathematics mentions this in the last paragraph on page 141 (continuing into page 143), without drawing the diagrams. In subfigure 16.ta, the shaded area is the Ennvity. ‘The rectangles are the value of the payments until time ¢ (the horizontal axis) times the probability of living exactly ¢ (the vertical axis; the diference between Fr at the top of the rectangle and at the bottom of the rectangle). So ths isthe calculation based on the definition of expected value. In subfigure 16.1b, the rectangles ie the payment (the horizontal axis; since the payment is continuous, itis proportional to t, the time) times the probability of receiving the payment (the vertical axis, 1~ F() = s(0), which is the probability of survival to atleast time 1). So this is the calculation using the current payment technique. Even the current payment techniques integral has an exponential variable in it, so as with insurances, it is difficult to calculate in closed form unless the survival function is exponential or uniform. So exams will mostly give you just those two functions, ‘Bxamour 16B A life is subject to a constant force of mortality 1. The force of interest is 6. Evaluate a, Answex: By now you've memorized that A, = p/(u + 8), s0 you can apply formula (16.2) to nmediately get Sh MLE Sey Manust—Bi elton Capen 2009 ASI 21 16. ANNUITIES: CONTINUOUS, EXPECTATION ‘We'll do an example of deMoivre (uniform) later, 16.2 Temporary Life Annuity ‘The random variable for ann-year temporary life annuity is - 1 ymcra aay = —G— But v7) is an n-year endowment, so taking expected values on both sides, we get a nice relationship between the annuity and an endowment: (16.6) = baicm 16.7) Aan = ‘The integrals for evaluating the temporary life annuity are the same as for whole life, except that the upper bound is m instead of co, Both the definition and the current payment technique may be used, but you'll almost always use the latter. Exautrix 16C For (35), you are given: @ Mortality follows deMoivre’s law with » = 100, ty A Gi) 6 = 005 wt Byaluate 45.3 Auswant The easier way is to use equation (16.6). The actuarial present value of 20-year temporary insurance is 1? 00s; BL ew a : Rl-e') : and the actuarial present value of the pure endowment is 29p35 Then S08 MLC Srdy Mamual—Bh eon Copy 62005 aha 16,3, DEFERRED LIFE ANNUITY 75 "The first integral is 20(1 — e~), The second integral requires integration by parts. 5 [roo yo 20 aoveoe|” +20 [a ear 4002" + 400(1 =e" = 400-8002"! so we have 1y_ 400 80021 fyggq) = 2001 - 1) - 900 _ 500 6 65. as before. a 16.3. Deferred Life Annuity “The random varlable for an n-year deferred life annuity is yen) V Ogee = “We don’t have a similar concept for insurance, .e., one where the maximum of two times is used, so we don’t have a simple formula relating a deferred annuity to an insurance. However, a deferred annuity isthe difference-between ‘a whole life annuity and a temporary life annuity, so that the expected values obey: ‘and we can caloulate the right hand side from insurances: ‘To use the definition or the current payment technique integral, the lower bound is n instead of 0. Exanetx 16D A continuous deferred annuity pays atthe rate of 1.04 at time starting 5 years from now. You are given: oor <5 © w= ea Pa (ii) 6= 007 Calculate the actuarial present value ofthis annuity. ‘Answer: We have a fort <5 De>Y -001(5)-0021-8) = gOO4D0S for p> § SOA MLC Sty Macal—8t eon Copyah 2008 AS 216 16_ ANNUITIES: CONTINUOUS, EXPECTATION 1n the integral, we have to include the varying benefit, ITY is the present value random variable, then wu [rodeo anni, a (8 05,50 0941810) ©" O09— in 104 = (1.051271 (0.775772) = "0.05078 16.4 Other Annuities 16.4.1 n-year certain and life annuity ‘The n-year certain and life ennuity is a sum of an.#-year annuity-certain and an n-year deferred life annuity, and its actuarial present value may be calculated in this fasion 16.4.2 Accumulated value am tepresents the expected value of the accumulated value of a continuous annuity paying 1 per annum at time n, accumulated for interest and survivorship. This is trickier than the corresponding concept in financial mathematics, because of survivorship, This is the amount of a fund accumulated by depositing all annuity payments in a savings account and then at time n distributing it only (o survivors, This is higher than fm, (unless 9, = 1, and then it’s equal), In symbols ‘Thom is needed because it determines the valuation time. The symbol 5, wouldn’t make sense, nor Would yj. This is in contrast to the @ symbol, where the valuation time is always time 0, so the subscripts can be used to indicate term periods or deferral periods. We will discuss sem more when we discuss discrete annuities in seetion 17.3 on page 291. 16.4.3 Increasing and decreasing annuities As with insurances, (fa), denotes an annuity with a continuously inereasing rate of payment, paying ¢ per year at time ¢, while (/@)x pays ¢ per year in year t (which isa larger amount). (Ba);n pays n— tat time r until time n, and (Dac pays n~1+ 1 in year until time n (which is a larger amount). Evaluating these requires integration by parts even for the simplest mortality functions Exawrie 16 You are given () 4 = 0.005 Gi) 6 = 0.06 Calculate (Za), ‘Axswan: The actuarial present value is Cae [ay ‘As we know from equation (11.1) on page 185, the answer is (xg)” SOA MLC Sandy Nam edon CCopyash ©2000 asa EXERCISES FOR LESSON 16 277 Exercises 100. The force of interest, 6, is 0.05. 16.1, Mortality follows de Moivre’s law with Caleulate ds. 162, You ae given \ @ A,=03 Gi) Ayan = 04 ' Gi) §= 0.05 Ge) mps= 0.7 Caloulate ay. 163, You are given: oor ostss s= 0 s(t @ w= Calculate 2,79 benefit consisting of the refund of the net single 164. A special continuously increasing life annuity has a death y-value random variable for this annuity. The premium without interest at the moment of death. Let Z be the present ' force of mortality is a constant 0.03 and 6 = 0.05. Calculate E{Z]. 16.5, (CAS4A-F98:19] (2 points) The future lifetime: random variable T(x) has a gamma distribution with param- eins and 6. Which ofthe following san expression for 2? wa) 1-(ea)" A(t aa) © 30-()) @) 1-450" () None of AB, Cor D are comet, 166. [3+801:24] Fora disability insurance claim (i) ‘The claimant will receive payments atthe rate of 20,000 per year, payable continuously as ong a5 she remains disabled, (ii) The length ofthe payment period in years is arandom variable withthe gamma distribution with parameters a= 2and@=1. ii) Payments begin immediately (iy) 6= 0.05 Caloulate the actuarial present value of the disability payments at the time of disability (A) 36,400 (B) 37,200 (© 38,100 ©) 39,200 ©) 40,000 SOA MC Sey Masts eon Bereta cominwe onthe net pgs nytt 2008 ASN 278, 16, ANNUITIES: CONTINUOUS, EXPECTATION (46-7) {3-F02:39] Fora whole life insurance of I on (x), you ae given: (The force of morality is ust). (i) The benefits are payable at the moment of death. (ii) 6 = 006 @) 4, = 0.60 Calculate the revised actuarial present value of this insurance assuming y(t) is increased by 0.03 for all # and is decreased by 0,03. (a) 05 ®) 06 ©o7 ©) 08 ®) 09 1638. [150-F88:17] You are given: (0) Ay and a, are based on force of interest and force of mortality Js, Gi) Ay and a ate based on force of interes k+ 6 and force of mortality ses. Gil) AZ is based on foree of interest 6 and force of mortality + jy Determine A ~ A, (A) ay ® MA, © Ayaka, D) &~ opi, +60, © 6@.- a) 165.) (150-890:1, CAS4A-$92:9) ¥ is the present-value random variable for a benefit based on (x) such that: rafim 2sre1sn lin, T)>n Determine E[Y], A) dem B) Gen + aie {C) am + nde D) din + V-Day ©) nde Om + die 16.10, [CAS4A-$90:9} (2 points) You are given: o for all x > 0, a 025 Gi) 5s constant Calculate 2, (A) Less than 10.0 (B) At least 10.0, but less than 12,0 (©). At least 12.0, but less then 14.0 /(D) Atleast 14.0, but less than 16.0 ©) Atleast 160 S08 MLC ny Manotel ercses contre onthe nex page epi 2009 ASME EXERCISES FOR LESSON 16 - _ 7 16.11. @ Gi) Gil) 279 [3-FO2s4] You are given: py) =0.01,051<5 Hx(0) = 0.02, 6 = 0.06 Caleulate a. (A) 125 B) 13.0 © 134 @) 139 ® 43 16.12. [CAS4A-$93:5] (3 points) You are given the following: 6= 005 [0.05 forosx<50 M**V010 for 50 = SS ‘An alternative is to first compute digo and ds using equation (17.1): Aw _ (= 03)(1.08) 0B Asp _ (1=0.4)(1.03) rc in — 104 dso 4.03333 — 0.1(20.6) = 9.61333, 961333 - faary = 2S — ERS] o 17.2 Annuities-immediate ‘A life annuity-immediate pays 1 per year atthe end of each year as long as the annuitant is alive atthe end of the year. The random variable, if is curtate lifetime, is. on “This form is not as convenient to express in terms of insurance; we'd have to multiply by v to get vE*!, a whole life insurance. Taking expected values: vag = PAs ag = tot Ds fay = 1-4 DAs [SOA MLC Sty Mant eon pyrene 2009 Att 290 _17. ANNUITIES: DISCRETE, EXPECTATION Figure 17. ‘Timelines for temporary annuities ‘The final statement can be interpreted as follows, ‘The value of I right now is equal interest on 1 at the end of each Year if alive at that point, plus a return of the 1 with interest for the final year atthe end of the year of death, In ‘estate tax law, the former, ia, is the life estate and the later, (1 + Ax, is the remainder, ‘The above formula is even less convenient for «temporary annuity-immediate, The best we can do is translate the comesponding formula for annuities-due, equation (17.3): 1-Aen a p= Gt 0= 0+ Aon Gem +1 ~ By i La (+ Aen Hinks On = T An - ttn = ign + Acm + Atm ‘The interpretation of this is: the value of I right now is equal to interest on 1 at the end of the year if alive at that point for the next n years, plus the return of 1 with interest for that year if death occurs before the end of year n, Plus the return of 1 only if alive form years. There is no need to add interest if alive for n years, since the annuity already pays interest for year n in that case. Rather than memorizing these formulas, you're probably better off translating the annuity-due formulas as needed, Figure 17.1 compares temporary life annuities-immediate. In this igure, the valuation time is shown with a black disk, and is always 0, while “$1” indicates e payment of 1 at that time, We see that dey, has all the payments of deen except the payment at time 0, so aun = ban ~ 1 To express am in terms of ign, we have to remove a payment at 0 and add a payment at n, 80 Gem = tem ~ 1+ ns ALOE = Ae 1 C8 SOA MLC Stay Manu! ton CCopyneh 2009 nS 17.3, ACCUMULATED VALUE _ 291 Exanece 17D You are given © eaas0) = 67 Gi) Adera = 0.1 ii) @ = 0.06 Caleulate iE. Anwar: We have ayo = 6.7 +1 ~ 10, 80 Agoam = 1 ~ 0.06(7.7 — oan) = 0.538 + 0.06 00 ‘The other equation we have is Agora = Adbra + 10Eax Agra = 0.14 wE@ Substituting that into the previous equation, 0.1 + 10a = 0.538 + 0.06 i040 0438 pon 1000 = ay = G56) a "The direct way to calculate an immediate annuity is the current payment technique, which means edding up the prvinonts times hel robailes aon = SY ‘This formula can be generalized to any series of payments by; the sum then becomes Siolape Jn particular, this can be used for calculating (Za)_m and (Da)xm, which are defined the way you expect them to be. 17.3. Accumulated value ‘Accumulated values of annuities ate tricky, since the valuation dates depend on n, Here is an example of a false statement: WRONG! sen = tga — "This would be correct if the s's are replaced with a's. The problem with the equation as it stands is thatthe valuation date is n on the left side, n + 1 on the right side. Figure 17.2 compares various annuities. The annuities are lined up by age to make them comparable, with the valuation date indicated by a disk, The valuation date for Sen, is a year later than the others, which makes it necessary to multiply this by Exe, before we can do anything else. Then we see that it has a payment at age x which Sem doesn’t, and age xis n years before the valuation date, so we must accumulate this payment for n years, which ‘means dividing it by the discounting factor »,. We conclude that the corrected version of the above equation is xin = Exon S591 Copy 62008 ASH 292 17, ANNUITIES: DISCRETE, EXPECTATION st $1 st $1 Save x eel redox en-2 oxen! zene St $1 Sin rene $1 st $1 \ si st Soe -—— HO $1 st Sx ——— 8 —_ ™ rene? rene] on ened 81 $1 $1 31 st eM +t x Hl ehh rene? genet xen xen Figure 17.2: Timelines for accumulated values of annuities ‘The other annuities are valued on the same date, at age x +n. Some conclusions we draw by comparing payments and accumulating differences are: Exameue 17E You are given f Determine ,E. 42 and Sem = 41. ‘Answar: Using the second equation above, we see that Spl t hse 41d from which we conclude that .E, Exams 178 You are given: (@ Sem = 20 Gi) niBs ait) 75 SOA MLC Sty Meoua-8ih ean Cops uno ASM EXERCISES FOR LESSON 17 293 Caleulate san Answer: ‘The annuity-due that we are given pays 1 at, 1, ...,”~1, and is valued at time n, We remove 1 at time O valued at time m from the annuity-due that we are given, to get the correct payment pattern, 1 at time 0 is worth k= iy at time n, We'e left with 20 ql = £3, Then we have to correc the valuation date from time nto time Ag (1s) Bxon-t) = Be (0.75)Exenat = 0.7 6 a Ww = ae nok e074 13\f 0.7, 13 san (calles) a5 a Cangas Sacat ~ 4 Exercises 17. A special temporary 3-year life annuity-due on (30) provides payments of fin year f,¢ = 1,2,3. You are given: voaq Vay? oma. 0.85 @ qs = 0.01 poy vaya ¥ an Gi) an Gt ‘Compute the actuarial present value ofthis annuity 17.2. (150-F89s1] You are given: k| aa | ings . 1 | 1.00 | 0.33 So 2/193 | 0.24 Ae te Pe O71 a ahh ran O79 3 | 2.80 | 0.16 Tide a Pe ae © O16 BAPE. 108807 O16 4 | 3.62 | 01 Bide aPe- ae OMS ABE = 1-0 8%-007 O16 Hh Calculate a3, (A) 16 B) 18 (©) 20 @) 22 © 24 T fint (e 4 tena O91 jag O24 17RD OG 1 862109 me 1362 (20.93 O24. OIG. aN) SOA MLC Sey Manusl—8i eon ‘Bvercles conte on the nex page Copy 2008 ASH 17, ANNUITIES: DISCRETE, EXPECTATION (17.3) (SOA3-F04:11) Your company is competing to sell a life annuity-due with an actuarial present value of 500,000 to a 50-year old individual, Based on your company's experience, typical 50-year old annuitants have a complete life expectancy of 25 years. However, this individual is not as healthy as your company’s typical annuitant, and your medical experts estimate that his complete life expectancy is only 15 years, You decide to price the benefit using the issue age that produces a complete life expectancy of 15 years, You also assume: wie nt <0 For typical annuitants of all ages, mortality follows De Moivre’s Law withthe, game limiting age, «. Gi) t= 006 ho= $e Calculate the annual benefit that your company can offer o this individual, “Vy = Mn (A) 38,000 @®) 41,000 (©) 46,000 () 49,000 ®) 52,000 174, (180-891:13] You ae given: Hew 120 Gi) H=6 120 ‘Which of the following is a correct expression for an? ° vs trays 4 owen 1a eme anne fy cae vn de ” Gacne4 an = vee @ om (A) None (B) Lonly (© Tonly (D) only ©) WV only 17.8) [3-S00:5] An insurance company has agreed to make payments to a worker age x who was injured at work. (The payments are 150,000 per year, paid annually, starting immediately and continuing for the remainder of the worker's life (ii) After the first $00,000 is paid by the insurance company; the remainder witl be paid by a reinsurance company. cts. 3 4S _{O7, Os1s55 vsv000 “boon THER SOA eee oe are Calculate the actuarial present value of the payments to be made by the reinsurer. (A) Less than $0,000 (B) Atleast 50,000, but less than 100,000 (©) Atleast 100,000, bat less than 150,000 (D) At least 150,000, but less than 200,000 (&) Atleast 200,000 S08 MLC Study Mem ton -Exoreisescominue onthe next page Cnynan gaan Ah EXERCISES FOR LESSON 17 _. __ 295 17.6. [CAS4A-F93:3] (2 points) You are given the following insurance and annuity options for a single insured: 1, 10-year pure endowment insurance of 1. 2. Whole life annuity immediate with annual payments of 1 3, Whole life annuity due with annual payments of I. 4, 10-year temporary life annuity immediate with annual payments of 1. Rank the actuarial present value of each option from the largest to the smallest (A) 4>3.>2>1 B) B>2>L>d. © 224.5321 Dy 2>3>4>1 QA 32-41 17.7.) [CAS3-S04:12] You are given the following: + ‘The probability hata newborn lives to be 25 is 70% 5125) < 019 was > 0.4 © The probability that a newborn lives tobe 35 is 50% “151-05 +The following annuities-due each have an actuarial present value equal to 60,000: alife annuity-due of 7,500 0n (25) fis. 60. 000/750 a life annuity-due of 12,300 on (35) fiye - 60 OF 12.300 a life annuity-due of 9,400 on (25) that makes at most 10 payments fas: vat = 0.908 F 4-t0b ‘What isthe interest rate? tus «tas ayy VO woPes diss gh 8.0% B) 81% © 82% ©) 83% ©) 8.4% 17.8, [CAS3-F04:1] A special 30-year annuity-due on a person age 30 pays 10 for the first 10 years, 20 for the next 10 years and 30 for the last 10 years. Given: wo fixoran 420 0) Hino 1 20 49 Cro Ceo: 40) © wBn=m * tora Qo wh © Gy a | 20 Fx Geo ae ap Aye. dor 1101 A 30-4) 1 mw 0m ve Ut 40) se 40) + tow © dom Which of the following represents the actuarial present value of this annuity’? (A) 207+ 10m(1 =m) B) 10u-+20v+ 10w (©) 10u+20v-+ 10m (BP) =10u-+ 20+ 10m (BH) -20u + 200+ 10w continue onthe next page SOA MLC Sty Mans Capyabt 2000 ASM 296 17, ANNUITIES: DISCRETE, EXPECTATION 11.9. {CAS4A+$96:18} (2 points) You ate given: topo = 0.07 — anpo = 0.05 9p = 0.04 Each survivor at the age of 20 contributes an amount P to a fund so that there will be a sufficient amount at the tend of 10 years to pay $1,000 to each of the survivors'at age 30, i = 0.06, Determine P gels = why" = oo ge (A) Less than $325 0b (B) At least $325, but less than $350 in Al least $350, but less than $375 i) ®) At least $375, but less than $400 At least $400 17.10, [CASHA-F96:7] (2 points) A life, age 65, is subject to mortality as described in the following excerpt from a 3-year select-and-ultimate table: xh fogs Moar. 655,000 4,750 4,500 4,200 66 4,800 4,550 4,250 3,800 67 “4,600 4,275 3,900 3,300 68 4,300 4,000 3,500 2,800 For some interest rate j, the actuarial present value of a 2-year temporary life annuity immediate for this life ‘equals the actuarial present value of a 2-year deferred, 3-year temporary life annuity immedjate for the same life, Determine the range in which the interest rate, i, lies. (A) Less than 79% (B) Atleast 7%, but less than 89% Ges = 2) A659) GH. Atleast 84, but less than, 9% Fvtares pvserb (D) At Feast 996, but less than 10% we . 5 B) Atleast 10% SV AMIso 4 VE A800 54200104 3eog 33000" Soc sooo ‘SOA MLC Study Mal eon ‘Burclee conta on he next page {Copy @2009 ASME EXERCISES FOR LESSON 17 207 {1.11 (CASHA-F97:21) 2 points) Two lives, ages 50 and SI, were selected at their curren ages and are subject to mortality as described in the following excerpt from 93-year select and ultimate tble: = fg tom toa ton 30 5,000 4,750 4,500 4,200 51 4700 4400 4,100 3,775 52 4325 44000 3.675 3,350 53 3,600 3,275 2,950 2,625 For some annual interest ate i, the actuarial present value of @I-year deferred, 2-year temporary life annuity immediate forthe life age 50 equals the actuarial present value of a 2-year deferred, 3-year temporary life annuity immediate forthe life age Sh Determine the annual rate of interes, i a) Osa) 2) Vand aoe (A) Less than 12.0% eons ays senosy HEMEL Ze (B) Atleast 12.0%, but less than 12.5% Ge 4100 (C) Atleast 12.5%, but less than 13.0% . (D) Atleast 13.08, but less than 13.5% +143, way (B) Atleast 13.59% Seat ho de Jing one year from today, he will receive $5,000 1712. [CAS4A-899:18} (2 points) Mike is 30 years old, Begi annually for as ong as he lives. be = keforall x The present value of the annuity is $22,500, Using i = 0.10, calculate k . iA 4) sto. ean zs ~ Ary = S000 | ve] (B) Atleast 0.800, but less than 0,825, (C) Atleast 0.825, but less than 0.850 (D) Atleast 0.850, but less than 0.875 GR Atleast 0875 17.43, [CAS4-S87:18) (2 points) Which of the following is equal to sanm= oda ten Gx Gear ol SE ~ en ~ trons? ae A) mnEs nBx © wiEx (D) ~enEx (B) The correct answer is not given by (A), (B), (C), oF (D). 17.14, [150-F96:4] Which of the following is equivalent to: : gem = Gem~) 3002 Jem + (14 DAgm ~ 17 ao B) ins oO: @) sEs ®1 SOA MUC Stay Maal ton Exercises continue on the next page... apy e008 ASME Peas: he [150.598:9] You are given: Ag = 0.22 Ary2s = 0.46 Ava = 0.20 1 = 0.06 Calculate a,5. (A) 98 B) 10.1 17.16, [3-FO1:26) You ate given: @ Ap= 0.28 Gi) Ayaan = 0.40 Gil) Ayah = 0.25 0.05 Calculate asa). (A) 1.0 mua _,}7. ANNUITIES: DISCRETE, EXPECTATION hard WR Aaa Gone te Ae d Ox x14 020 (©) 104 ©) 109 pf it ous @) 120 ® 123 {0.17. ‘The numberof full years of future lifetime of (2) has the following distribution: Fallyeare of future lifetime Probabily 0 On 1 o1 2 02 3 02 4 oa i= 003 Calculateay — 0601 04 Oa Q7AS, [CAS4-F82:25] An annuity of 1, issued at age 35, is payable at the beginning of each year until age 65. ‘The annuity payments are certain forthe frst 15 years ‘You are given: Calculate the present value of the annuity. (A) Less than 18,00 m At feast 18.00, but less than 18.50 ") At least 18.50, but less than 19.00 (D) At feast 19.00, but tess than 19.50 (B) Atleast 19.50 SOA BILCSiay Marsal tion Copa @n009 ashe Assay = 11.62 Aggy = 18.13, s59 vid Benya AW re 48.69 ercses continue on the next page ( EXERCISES FOR LESSON 17 299 \ 17.19. (CAS4A-F98:18} (2 points) Mortality follows the Ilustrative Life Table. ¢ Determine ters: = wes ig» @ 61 _ (A) Less than 10.0 ‘aay = 2-08) I (B) At Teast 10.0, bt less than 10.5 (RY AcTeast 10.5, bu less than 11.0 ! (D) Atleast 11.0, but less than 11.5 (6) Atleast 1.5 206 96 47.20) [150-890:6) For » 30-year deferred, annul life annuity-due of 1 on (33), you are given: (i) Ris the net single premium for this annuity if the net single premium is refunded at the end of the year of { death for death during the deferral period. Gi) Nis the net single premium for this annuity ifthe net single premium is not refunded. Which of the following correctly expresses R ~ N? ' L Aas + soiiss 1 Aden, lds, Ms.san i 1 + in, Atiz0(4ssan— Ass) Ve pte | al = Adm) | , \ mm, (1 ssn anitss to) Oe Big - Gos > Aas sod ~ Mes dass. ' (A) None B) Lonly (©) only ©) Monly (E) The correct answer is not given by (A), (B), (C), or (D). -) 1721. {CAS4-S83:17] Smith, age 65 is evaluating three temporary life annuities, each of which provides annual i payments Fot each annuity, the last payment is at age 75. “The first annuity pays $5,000 at age 66; subsequent payments decrease by $500 per year. The present value of this annuity is $14,000. “The second annuity pays $1,000 at age 65; subsequent payments increase by $1,000 per year through age 75. “The present value ofthis annuity is $21,000. “The third annuity pays $1,000 each year beginning a age 65 and continuing to age 75. The present value of this annuity is P. Determine P. (A) Less than $4,000 : (B) At east $4,000, but less than $4,400 (C)_ Atleast $4,400, ut ess than $4,800 (D) Atleast $4,800, bat less than $5,200 (©) Atleast $5,200 SOA MLC Sealy Marlton _Exeretes continue on he nex page (Copynah 2009 ASI 300 17, ANNUITIES: DISCRETE, EXPECTATION 112, @ Gai) I present value of a 16 year temporary increasing life annuity-due providing a first payment of ‘5000 and subsequent annual payments increasing 1000 per year is 75,000, Gi) Z is the present value random variable for a 15 year temporary decreasing life annuity-immediate providing 2 first payment of 7500 and subsequent annual payments decreasing by 500 per year. Caloulate EZ). (A) Less than 24,000 (B) Atleast 24,000, but less than 26,000 (©) Atleast 26,000, but less than 28,000 (D) Atleast 28,000, but less than 30,000 (B) Atleast 30,000 P1723. (CAS3-F05:9] (Fa) and (1a) represent the standard increasing annuities, A person aged 20 buys a special five-year temporary annuity-due, with payments of 1,3, 5.7, and 9, Given: @ ton = 3.41 i) aap = 3.04 Gil) Cayo, = 8.05 iv) Ua)ygq) = 7.17 Calculate the net single premium, (A) Less than 18.0 om ‘At least 18.0, but less than 18.5 (C) Atleast 18.5, but less than 19.0 () Atleast 19.0, but Jess than 19.5 © Atleast 19.5 Vat elaw ay Yeo. 17.24. You are given 4 aay ~ Gin) Gt - Veal Ooms We 1Ohaa ya aa Gil) §= 0.05 hehes Gv) ope = 095 Cateutate 5,45 aA E whe aca? Ae orl wie v 17.25, You ae given @ Seqq = 12 Sasa Qadil f wre (i) ae =0.75 (ii) 9B, = 0.78 Sucw Calculate s.. “a Abe v6) wle 4 wo OG SOA MLC Stuy anus con Exercises continue on the next page. Copp 62009 ASH EXERCISES FOR LESSON 17. 301 ‘The following exercise involves more equation manipulation than a modern exam would have, so it may be skipped. 217.26, (4-886:23] Simplity 1 1 SilonAln toy win tenant #9) “a (B) © ©) me 17.27, [150-F89:28] You are given that sq = dem/nEs Which of the following are true? Lhe masts = Veen wx TL Agm ~Agien = 4 0B TL Span ~ sem 1d 11 only (B) Land IT only (© Mand Mionly (D) J, Wand OT (F) The correct answer is not given by (A), (B). (C), oF (). 17,28, (150-$91:1] You ate given: (9B, = 040 ; Gi) taas=7 Ge Haw y ae) Ve ae Sarvap inte + WiG troha Caloulate (A) 12.0 @) 125 fof © no hoa SOA MLC Stedy Manon!—Bi eon Beretes cone onthe nex page Copy 2000 ASM 17. ANNUITIES: DISCRETE, EXPECTATION 17.291 (CAS4A-597:2] 2 points) John (age (2)) and Paul (age ()) are coworkers Who are injured in a common occurrence. John’s first-year medical expenses are $200,000, with subsequent years’ expenses increased by 12% annual inflation, Paul’ first-year indemnity payment i $50,000, with subsequent years’ payments increased by 79% ‘annual inflation. Their expected mortality is deseribed in the following table: Expected Mortality for: John Paul dows bye 0 1,000 1,000 1800900 2 700 800 900) 650) 4 450500 John receives no indemnity payments and Paul receives no medical benefits. Payments are made at the end.of ‘each year the claimant survives. surer’s expected total payment at the end of the third year afler the accident (f = 3), (A) Less than $175,000 (B) Atleast $175,000, but less than $190,000 . soneott st") 0, (©) Atleast $190,000, but less than $205,000 290.0001") G41 soOcoLt (D) Atleast $205,000, but less than $220,000 ®) Atleast $220,000 17.30. "A life annuity-immediate provides payments of 10,8, 6, and 4 in the fist 4 years, followed by payments of 2 per year thereafter, You ore given 0.26 0.30 2 OA Gi) Ae Gil) ads (iy) Agh=08 spr = 0.98 Calculate the actuarial present value of the annuity. 17.31. You are given @ Argrq = 084 Gi) Aasam = 0.69 Gil) 1= 0.02 Calculate jgés.r 17.32. (150-F87:1) You are given: (198s = 0.35 Gi) agg = 5.6 Gi) (= 010 Calculate Ady) (A) 0.05 (B) 0.10 (C) O15 (D) 0.20 (B) 0.25 SA MLC Sty Mania ion Exercises continue on the next page Copyghc2009 ASK EXERCISES FOR LESSON 17 17,33. (4-F86:27) Simplify [se ~ HD dase (A) dain B) dan ~ 4: (C) diicm + Ve ©) d@m- 9) (B) diiggen ~ VPs [150-188:30] You are given: 303 +dyPxim: say = 4.0 drm omy ey ae 100 Ate 1 AG ay 5.0 Sag) nta 9 oxen haere) X 0.94. wia> © fein > An = heer Caloulate Abra (A) 024 (B) 034 (©) 044 ©) 0.54 ©) 064 1735, [150-81-94:19} Consider the following present-value random variables, where K isthe curcte future life- time of (3): Yeign K20 gafiem o Gaga) Gas She E © tapas = 0.87 4 _ asa) © dysa3) = 9.868 / © iss = 4.392 1 LI fos i Caleulate Ady eo ‘Less than 0.16 }) At least 0.16, but less than 0.32. (© Atleast 0.32, but less than 0.48 (D) _Atiesst 0.48, but les than 0.64 ©) Atleast 0.64 The following exercise is probably too theoretical for a modem exam, and may be skipped. 1747, (150-197:6) You are given: © G denoves the value ofthe current payments integral expression for Jn, evaluated by a trapezoidal ap- proximation using a singe trapezoid over each year of age. (1) H denotes the exact value of ac, evaluated by the same interest and morality as used to determine G. Determine lity se IG ~ Hl. (A 0 ®t (© i-6 os (©) § Additional released exam questions: CAS3-S05:5, SOA M-F06:4, SOA MLC-S07:24, CAS3L-F08:20, CASSL-S09:13 Solutions ATA. The value is 0.99. , (0.990.985) 14 Tgg@)* Tog) = 1 + 1.903846 + 2.70474 608585 17.2, We add up the probability of living exactly & years times dg. ix = 0.33(1.00) + 0.24(1.93) + 0.16(2.80) + (1 ~0.33 ~ 0.24 ~ 0.16)(3.62) = [B86] @) ‘The last probability is not 0.11, since ifthe insured survives 3 years he gets 4 payments regardless of whether or not he dies in the fourth yea, 17.3, Since the complete life expectancy for typical annuitants is 25, under deMoivre this must be half of the ‘maximum lifetime, which is therefore 50, and w = 100, For a life expectancy of 15, you'd make the issue age 100-{2515) = 70 so that maximum future lifetime would be twice expectancy, We've mentioned that for deMoivre's [ive ifs easier to calculate insurance and then to use formula (17.1) to convert it to an annuity-due, The density is 1% t fn 36 2 108 ny epmight 200 aS EXERCISE SOLUTIONS FOR LESSON 17 307 3607 “The benefit is then $982 = [52,297], (E) IIA, Inagy = Ine-@* = -(u + 6). The correct expression for nis ws | should not have a square on 6. Il is only an approximation, TV is no good because of the missing negative sign in the denominator. Since i,m = 1 and a7 = eH, Ms correct. (D) 175. The insurer pays 150,000 for 3 years followed by 50,000 in the fourth year. The reinsurer pays 100,000 in the fourth year followed by 150,000 afterwards, but the worker cannot survive past the ft year (he sixth payment). ‘The actuavial present value of the payments made by the reinsurer is 100,000 3B, + 150,000 4B, + 150,000 sy aEx 0.296296 aBy 0.197531 or? = 27, 0.13168: sly = Tags = 0131687 [APV of payments = 100,000(0.296296) + 150,000(0.197531) + 150,000¢0.131687) = [79,012] (B) 176. The annuity!due provides one more payment than the whole life annuity immediate, which provides more peyinents than 2 10-year annuity. The pure endowment provide te last payments, only 1st duration 10. (E) 177. Translating the fist 2 bullets into actosral symbols, (25) = 0:7 and s(35) = 0.5, So opas = f= §- Then wwe have 3 annuities-due: se, = $0,000 1000 ag, = $0,000 a5 = T2300 = 4878 60,000 asta = gp 7 6383 Now we use the fact thatthe sum of a term and deferred annuity is a whole life annuity: isan + VoPastns ths io) 8 = 6.383 + v!9(2\4.878) OA MLC Sey Manus 8th eon Copy 2009 ASH 308 17. ANNUITIES: DISCRETE, EXPECTATION @ 178. We have 10u for the firs 10 payments and 30m for the last 10. The middle 10 are harder. We decompose 030 = dhoan + 0a Ho {n10+ nadaorat rotor 1+ ago, +m nf Big nba Qe So we need 20(v~ wmv) forthe present value of te middle 10 payments, ‘The total actuarial present value is then 101+ 30m + 20(v~ u ~ mw) = PANO 42051 Tomi), (D) 179, We need ioExy gg 2 WP. (0.041 hm * 10600 ~ \0.06)\ 05% ‘The answer is 1000(0.372263) = [372,263] (C) 17.10, ‘The equation can be set up as 0.372263, 47500 + 45007" 1200" + 3800v4 + 3300v5 ‘where we've multiplied ;pjy through by fs) = 5000, I don’t see how to solve this quartic equation (after dividing ‘out v) with the caleulators allowed atthe time of this exam other than by rial and error; you're not expected to know the quartic equation, and it's pretty messy (o use anyway! Plugging in 79, 8%, and 9% shows that the solution is between 8% and 9%. (C) 17.11, This time, the equation is 4500v? 4 4200v" _ 377503 + 3350v4 + 262505 ‘5000 4700 Which is “only” a cubic, Tal and error demonstrates thet the interest rate that solves this is higher than 13.5%. (&) 1 Wonder what the CAS had in mind in these two questions (17.10 and this one), whether they really expected students {0 plug in four possibilities or whether there is some shortcut that Tam not aware of. With modern calculators like the BA-II, however, you can solve for arate of return. TAX a, = BE? = 4.5, We have SOK MLC Stsy Magna ean Copia 62009 nS EXERCISE SOLUTIONS FOR LESSON 17 309 TTAB. icon = aE snd 80 the fits fraction is 8. ‘un + sEsdevn = y, $0 the last 2 summands are lower than a, by the present value of the nth payment, »Ex So the expression reduces to ay-Gx-nEd = Ex B) 17.14, Wo use the formula am = 42. gm = tem — 1+ Es =F = Agn) -i4 inks inEz| (8) ign + (1+ 0Agm = 141-14 inBe =| 1735, First le’s celoulate Ayzm. To get the endowment, we take whole life, remove the discounted value of insurance after 25 years, and add in a pure endowment. 22 ~ 0.20(0.46) + 0.20 = 0.328 Ag = ‘Then we convert it into an annuity-due. 10,328 dasa = apenas = 18 Finally we convert that into an annuity-immediate by removing the initial payment of I and adding « payment at the end of the 25th year. agg) = 1.872 -1+0.20=[5072] (B) 1746. 28 ~ 025(0.40) +0.25 = 0.43 1-043 Seal = 0.95/1.05, gg = 11.97- 140.25 = [E22] B) Act = = 1197 1TAT. There is 2 0.6 probability of three payments and a 0.4 probability of four payments. oxy = 069 + 0.40 (0.11 —a1.03°) + 0411 - (171.0375) O05 (0.084858) + 040.1151) _ eras 0.08 47.18. This isa life annuity of | for 30 years, plus an annuity certain for 15 years, minus a temporary life annuity for 15 years (the intersection of the first 2 annuities), or 18,13 + 11.94 ~ 11.62 =[18.45 | (B) TAS. We use digg = oF es ns + dy, dns and 04s at 69% can be read right off the Mlustrative Life Table. 1 -(1/1.06') 0.06/1.06 Aggy = 0s ns + rm = (039994,7.2170) + 7.801692 801692 ay = © SOA MLC Sty Manul—8h eice Ceprsen 2009 Att 310 17, ANNUITIES: DISCRETE, EXPECTATION 17.20. The annuity with a refund is R= Asa + soiss Pu Ass R= The diference is then ji sossAssmm As -jtiys Can be expressed as sais = Bs ~ Bag, Asso ~ Ass a Putting the last two expressions together gets I, The other expressions don’t work, I is obviously mistaken, II is \wrong because there is no general way to convert a term insurance into d times an annuity. (C) 17.21, We double the frst annuity and add a payment of 11,000 at age 65, so its total val is 2(14,000)+ 11,000 = 39,000. Adding the second annuity to this we get a level series of payments of 12,000 from 65 to 75 inclusive, and {he total value is 39,000 +- 21,000 = 60,000. Dividing by 12 gets us P, or [$:5000]. (D) 17.22, " One way to do this is to divide the increasing annuity by 1000 to obtain a seties of payments 5, 6,.., 20 worth 75. Subtract this from 21 times a level annuity-due for 16 years worth 6+ 1 = 7 to oblain payments of 16,15,....1 worth (21)(7)~75 = 72. Now remove the payment of 16 to obtein 72~ 16 = 56. Multiply this by 500 to Bet the desired decreasing annuity, 500 x 56 = [2,000 (D) One of the rare cases where the answer is atthe border of range. 17.23, The initial payment of 1 has present value 1. ‘To get payments at times 1, 2, 3, and 4, we are best off working with immediate annuities; the due ones have payments at times 0, 1, 2, 3. By taking ? increasing annuities- immediate, we get payments of 2, 4, 6, 8 at times 1, 2,3, 4. We add to that e level annuity of 1 to get the desired payments of 3, 5,7, 9. So the answer is 1+ 2la)na + m9 = 1+2(0.17) +3.04 = [FESR] wy 17.24, First we caleulate 19) “4 far Oe + ra ~ Bry 1,05(1~(1/1.05)"9) 318 ay = 13 + 8,107822 ~ ary agrq = 7.107822 ‘We adjust this to get aq and then accumulate it 10 years, 4ei0) = Germ — 1+ 0B nbs = 22%, -ossias = 7.107822 — 1 + 0.583218 = 6.691039 = 6091089 Sei = 0583018 530A MLC Stody Manual efiton Copa 2009 ASN EXERCISE SOLUTIONS FOR LESSON 17 _ aut 417.25, ‘The difference between Sq and sg is thatthe later does not have the intial payment of 1, and thatthe latter is evaluated at the end of year 9 while the former is evaluated at the end of year 10. “We first move back from the end of year 10 tothe end of year 9. " of.) = ss et limite inital pment, sy snes [0a 17.26. This problem is modified version of problem 5.35 in Actuarial Mathematics. Ifyou want to reason tis itiply numerator and denominator by v. In other words, put double ‘out (not so easy!), the frst thing you'd do is mul ots overall the s's. Now the expression inside parentheses isa 10-year insurance which pays a life annuity starting ethos 0 Tn other words, isa life annuity, except that if you die after 10 years you get nothing, So it's & 10-year temporary life annuity minus a 10-year pure endowment of 10-year annuity-cerain, Dividing by Sym, the answer is). * ew well doit algebraically, Les temporarily ignore J. We write ou the insurances a a Disna-uden = re * . 7 = Seapegonal a 5 S eaedncitn % 2 ra & vitro ~ P10 IO, WaPo dase 1Pedesk-tVa “the inst equality is because the sum of an annuity-duecertzin with k payments if one survives k years ks 105 8 life annuity-due for 10 years to anyone who survives 10 years or less. ‘We now use Syp) = Yap) = Zig to obtain the inal answer: vier = Yost _ Micra _ Y0Ps A. = Vio am 1op0"” etm) 1727. Tis uve because the difference in the deferred insurances is the probability of dying in the n+ 1)st year sinnes the value of 1 atthe end ofthat year, oF dey times *! 4p. The second factor is the same #8 Vn. “Tig uve because the difference in the endowments is f one survives more than n years. ‘Then the payment of 1 is made one year late. So the difference is yp times the difference between "and v™", which is aps(v* yl) = Peo =¥) = Ed. SOA MLC Sed Manual #0 eon opr 200 AS 32 17. ANNUITIES: DISCRETE, EXPECTATION I is false because the two s's are evaluated at diferent times, The fists is evaluated at time n-+ 1, whereas Ceesseond sis enfuated at tine, Thus, in adiionl othe diferencia he vl ofthe payments, which ‘one must divide sum by Eyay. A corect equality would be 5 Son Exn (A) 17.28, TO + tole STOOL) + (soyae + 0B) = 15(0.40) + (7 + 0.40) = [is © 17.29. There are two years inflation by the third year. The answer is 200,000(1.12740.6) + 50,000(1.07° 40.65) = [T8737] «B) 17.30. The timeline of payments is 10 8 6 4 2 2 2 2 aaa. 0 1 2 3 4 5 6 7 8 ‘The easier way to-do this is to express the annuity a as 45 100.5) 2(0a) 3 + 2a ‘This combination is easily seen to have the appropriate payments: Year 112; 3};4{s5]6 Wa | 10 10[ 10] 10} 10 -2da).3 | -2| -4] -6] -8| -10 2ay 2] 2] 2] 2| 2{ 2 ‘Sum eae 8a ear eae ‘We calculate the annuities. First we must back out d. Agh = 08 = py? 28 "098 = 0.960224 1 - 0.960224 = 0.039776 Now we calculate the anny 074 0.039776 ~ i 2 ay = 18.60442 ~ 1 = 17,60442 Ags = Ar AgdAns + Ap} {$04 MLC Suny Manat—Bi eion Copyright e208 ash EXERCISE SOLUTIONS FOR LESSON 17 pois = 0,26 ~ 0.8(0.30) + 0.8 = 0.82 _ 1-082 45) = 0.030776 agg) ip — 1+ AH = 4.525399 ~ 1 + 0.8 = 4.325399 ‘Themover's a= 10(4.325399) — 2(12) + 2(17.60442) ‘An alternative solution is to express the annuity as a= 12aq5) ~ 200) q5) + Pale 2A 14,07902, and itis then seen that you get the same answer, = 4.525399 ‘You can caloulte, sing Aves and Ags that gf 1731. sass = Asam ~ Has.to 1 =Ags79 _ 1 = Aasz0, d d 102 = (Ga) sme ~ Asam 1.02)0.15) so = es 17.32. Aggy = 5.641 = 66 Ado =1~(4)s6—035 = [605] 417.33. The first summand pays an annuity-due of d with £+ I payments if one survives exactly k years, 0 0.99 Yoonlrosey 5 oo 1 = Tis aos) ~ Toe a9 (b — 21)°(0.0888889 - 0.166667") = (b — 21)*(0.0611111) = (b~21)*@.0611111) 16.9548, 25.0452 = 0,0888889 SOK MLC Sty Mansa eon Copish 009 ASN 804 MLCSuuy Aesth aan Cop 2000 nat fa

You might also like